Tag Archives: chondoituyen

ĐỀ THI CHỌN ĐỘI TUYỂN QUỐC GIA CỦA CÁC TỈNH, THÀNH

ĐỀ THI CHỌN ĐỘI TUYỂN TRƯỜNG PHỔ THÔNG NĂNG KHIẾU

ĐỀ THI CHỌN ĐỘI TUYỂN HSG QUỐC GIA CỦA TRƯỜNG PTNK NĂM 2008 – 2009 – Toán Việt (toanviet.net)

ĐỀ THI CHỌN ĐỘI TUYỂN HSG QUỐC GIA CỦA TRƯỜNG PTNK NĂM 2009 – 2010 – Toán Việt (toanviet.net)

ĐỀ THI CHỌN ĐỘI TUYỂN HSG QUỐC GIA CỦA TRƯỜNG PTNK 2010 – 2011 – Toán Việt (toanviet.net)

ĐỀ THI CHỌN ĐỘI TUYỂN HSG QUỐC GIA CỦA TRƯỜNG PTNK NĂM 2011 – 2012 – Toán Việt (toanviet.net)

ĐỀ THI CHỌN ĐỘI TUYỂN QUỐC GIA CỦA TRƯỜNG PTNK NĂM 2013 – 2014 – Toán Việt (toanviet.net)

ĐỀ THI CHỌN ĐỘI TUYỂN HSG QUỐC GIA CỦA TRƯỜNG PTNK NĂM 2014 – 2015 – Toán Việt (toanviet.net)

Đáp án thi chọn đội tuyển Toán trường PTNK năm 2015 – Toán Việt (toanviet.net)

Đáp án đề thi chọn đội tuyển trường Phổ thông Năng khiếu thi HSG QG năm 2016 – Toán Việt (toanviet.net)

ĐỀ THI CHỌN ĐỘI TUYỂN HSG QUỐC GIA CỦA TRƯỜNG PTNK NĂM 2017 – 2018 – Toán Việt (toanviet.net)

ĐỀ THI CHỌN ĐỘI TUYỂN HSG QUỐC GIA CỦA TRƯỜNG PTNK NĂM 2018 – 2019 – Toán Việt (toanviet.net)

ĐỀ THI CHỌN ĐỘI TUYỂN HSG QUỐC GIA CỦA TRƯỜNG PTNK NĂM 2019 – 2020 – Toán Việt (toanviet.net)

Đáp án đề thi chọn đội tuyển trường PTNK năm 2020 – Toán Việt (toanviet.net)

Đề thi và đáp án chọn đội tuyển toán trường PTNK năm 2021 – Toán Việt (toanviet.net)

ĐỀ THI CHỌN ĐỘI TUYỂN CÁC TỈNH THÀNH KHÁC

Đề thi và đáp án kì thi chọn đội tuyển thi Quốc gia trường Phổ thông Năng khiếu năm học 2018 – 2019

ĐỀ THI

Ngày thi thứ nhất

Bài 1. Cho số nguyên $a>1$. Tìm giá trị lớn nhất của số thực $d$ sao cho tồn tại một cấp số cộng có công sai $d$, số hạng đầu tiên là $a$ và có đúng hai trong các số $a^2, a^3, a^4, a^5$ là những số hạng của cấp số cộng đó.

Bài 2. Cho $n$ số thực $x_1, x_2, \ldots, x_n$. Với mỗi $i \in{1,2, \ldots, n}$, gọi $a_i$ là số các chỉ số $j$ mà $\left|x_i-x_j\right| \leq 1$ và $b_i$ là số các chỉ số $j$ mà $\left|x_i-x_j\right| \leq 2$ ( $i$ và $j$ có thể bằng).

(a) Chứng minh rằng tồn tại $i$ để $b_i \leq 3 a_i$.

(b) Gọi $A$ là số cặp $(i, j)$ có thứ tự mà $\left|x_i-x_j\right| \leq 1$ và $B$ là số cặp $(i, j)$ có thứ tự mà $\left|x_i-x_j\right| \leq 2$ ( $i$ và $j$ có thể bằng nhau). Chứng minh rằng $B \leq 3 A$.

Bài 3. Cho $p$ là số tự nhiên. Xét phương trình nghiệm nguyên $x^3+x+p=y^2$.

(a) Tìm số nguyên tố $p$ nhỏ nhất dạng $4 k+1$ sao cho phương trình có nghiệm.

(b) Chứng minh rằng nếu $p$ là số chính phương thì phương trình trên có nghiệm nguyên dương.

Bài 4. Cho tam giác $A B C$ nhọn nội tiếp đường tròn $(O)$ với $B, C$ cố định và $A$ di động trên $(O)$. $D$ là trung điểm $B C$. Trên $A B$ lấy các điểm $M, P$ và trên $A C$ lấy các điểm $N, Q$ sao cho $D A=D P=D Q$, dồng thời $D M \perp A C, D N \perp A B$.

(a) Chứng minh rằng các điểm $M, N, P, Q$ cùng thuộc một đường tròn $(\mathcal{C})$ và $(\mathcal{C})$ luôn đi qua một điểm cố định.

(b) Chứng minh rằng tâm của $(\mathcal{C})$ luôn thuộc một đường tròn cố định.

 

Ngày thi thứ hai

Bài 5. Cho số thực $a \neq 0$. Tìm giới hạn của dãy số $\left(u_n\right)$ thoả mãn:

$\quad\quad\quad\quad\quad\quad\quad\quad u_1=0, u_{n+1}\left(u_n+a\right)=a+1, \forall n \in \mathbb{N}^*$

Bài 6. Tìm tất cả các hàm số $f: \mathbb{R}^{+} \rightarrow \mathbb{R}^{+}$thoả mãn điều kiện:

$\quad\quad\quad\quad f\left(x f\left(y^2\right)-y f\left(x^2\right)\right)=(y-x) f(x y) \forall x, y \in \mathbb{R}^{+}, x>y$

Bài 7. Cho $n=2018.2019$. Gọi $A$ là tập hợp các bộ $\left(a_1, a_2, \ldots, a_n\right)$ có thứ tự thoả mãn điều kiện $a_i \in{0,1} \forall i \in{1,2, \ldots, n}$ và $\sum_{i=1}^n a_i=2018^2$. Có bao nhiêu bộ $\left(a_1, a_2, \ldots, a_n\right)$ từ $A$ để $\sum_{i=1}^k a_i \geq \frac{a}{2}$ và $\sum_{i=n-k+1}^n a_i \geq \frac{k}{2} \forall k \in{1,2, \ldots, n}$ ?

Bài 8. Đường tròn $(\mathcal{C})$ tâm $I$ nội tiếp tam giác $A B C$ và tiếp xúc với các cạnh $A B, A C$ tại $E, F$. $A M, A N$ là các đường phân giác trong, phân giác ngoài của góc $\angle B A C(M, N$ nằm trên $B C)$. Gọi $d_M, d_N$ lần lượt là các tiếp tuyến của $(\mathcal{C})$ qua $M, N$ và khác $B C$.

(a) Chứng minh rằng $d_M, d_N, E F$ đồng quy tại điểm $D$.

(b) Lấy trên $A B, A C$ các điểm $P, Q$ thoả mãn $D P|A C, D Q| A B$. Gọi $R, S$ là trung điểm của $D E, D F$. Chứng minh rằng $I$ thuộc đường thẳng qua các trực tâm của hai tam giác $D P S, D Q R$.

 

LỜI GIẢI

Ngày thi thứ nhất

Bài 1. Cho số nguyên $a>1$. Tìm giá trị lớn nhất của số thực $d$ sao cho tồn tại một cấp số cộng có công sai $d$, số hạng đầu tiên là $a$ và có đúng hai trong các số $a^2, a^3, a^4, a^5$ là những số hạng của cấp số cônng đó.

Lời giải: Trước hết, ta chứng minh rằng $d=a^3-a$ thoả mãn điều kiện. Thật vậy, xét cấp số cộng có số hạng đầu là $a$ và công sai là $d=a^3-a$ thì

$\quad\quad\quad\quad\quad\quad\quad\quad\quad\quad \left\{\begin{array}{l}a^3=a+\left(a^3-a\right) \\ a^5=a+\left(a^3-a\right)\left(a^2+1\right)\end{array} .\right.$

Do đó $a^3, a^5$ cùng thuộc cấp số cộng có công sai $d=a^3-a$.

Giả sử rằng tồn tại giá trị $d>a^3-a$ thoả mãn điều kiện bài toán. Khi đó:

$\quad\quad\quad\quad\quad\quad\quad\quad\quad\quad a+d>a+a^3-a=a^3$

Dẫn đến hai số hạng thuộc cấp số cộng phải là $a^4$ và $a^5$. Lại để ý rằng $a>1$ nên có $a<a^4<a^5$, kết hợp lại thì phải tồn tại hai số nguyên dương $k<l$ sao cho:

$\quad\quad\quad\quad\quad\quad\quad\quad\quad\quad \left\{\begin{array}{l}a^4=a+k d \\ a^5=a+l d\end{array}\right.$

Từ đó $a(a+k d)=a+l d$ hay $d(l-a k)=a^2-a>0$.

Chú ý rằng ta có $d>0$ nên $l-a k>0$, hơn nữa $l-a k \in \mathbb{Z}$ nên $l-a k \geq 1$. Điều này dẫn đến $a^2-a \geq d>a^3-a$, vô lý do $a>1$.

Vậy giá trị lớn nhất của $d$ là $\max d=a^3-a$.

Bài 2. Cho $n$ số thực $x_1, x_2, \ldots, x_n$. Với mỗi $i \in{1,2, \ldots, n}$, gọi $a_i$ là số các chỉ số $j$ mà $\left|x_i-x_j\right| \leq 1$ và $b_i$ là số các chỉ số $j$ mà $\left|x_i-x_j\right| \leq 2(i$ và $j$ có thể bằng nhau).

(a) Chứng minh rằng tồn tại $i$ dể $b_i \leq 3 a_i$.

(b) Gọi $A$ là số cặp $(i, j)$ có thứ tự mà $\left|x_i-x_j\right| \leq 1$ và $B$ là số cặp $(i, j)$ có thứ tự mà $\left|x_i-x_j\right| \leq 2$ ( $i$ và $j$ có thể bằng nhau). Chứng minh rằng $B \leq 3 A$.

Lời giải . (a) Không mất tính tổng quát, giả sử $x_1 \leq x_2 \leq \ldots \leq x_n$.

Xét $k=\max [a_1, a_2, \ldots, a_n]$ và $a_i=k$, khi đó tồn tại $k$ số trong dãy là:

$\quad\quad\quad\quad x_u \leq x_{u+1} \leq \ldots \leq x_i \leq \ldots \leq x_v \text { với }\left|x_u-x_i\right|,\left|x_v-x_i\right| \leq 1 .$

Ngoài ra vì tính lớn nhất của $k$ nên $\left|x_{u-1}-x_i\right|>1,\left|x_{v+1}-x_i\right|>1$.

Trong $\left[x_u, x_v\right]$, có đúng $k$ chỉ số $j$ để $\left|x_j-x_i\right| \leq 1<2$. Còn trước $x_u$, xét hai số $x_r, x_s$ sao cho $x_r \leq x_s$ và $\left|x_r-x_i\right| \leq 2,\left|x_s-x_i\right| \leq 2$ thì:

$\quad\quad\quad\quad \left|x_r-x_s\right|=x_s-x_r=\left(x_i-x_r\right)-\left(x_i-x_s\right)<2-1=1$

nên sẽ có không quá $k$ số $j$ để $\left|x_j-x_i\right| \leq 2$ vì nếu ngược lại, sẽ có nhiều hơn $k$ số liên tiếp trong dãy cách nhau không quá 1 đơn vị, mâu thuẫn với tính lớn nhất của $k$. Tương tự với các số sau $x_v$, vì thế nên $b_i \leq 3 k$ kéo theo $b_i \leq 3 a_i$.

(b) Ta sẽ chứng minh bằng quy nạp theo $n$.

Với $n=1$ rõ ràng $A=B=1$ nên khẳng định hiển nhiên đúng. Giả sử kết quả đúng với $n \geq 1$, ta sẽ chứng minh nó cũng đúng với $n+1$.

Xét dãy số thực $T=\left(x_1, x_2, \ldots, x_{n+1}\right)$ bất kỳ và giả sử $x_1 \leq x_2 \leq \ldots \leq x_{n+1}$. Ký hiệu $A_T, B_T$ là số cặp có thứ tự các chỉ số $(i, j)$ tương ứng với định nghĩa của đề bài. Giả sử $k \geq 1$ là số lượng lớn nhất các số của $T$ được chứa trong một đoạn độ dài bằng 2 nào đó.

Gọi $x_i$ là số cuối cùng của dãy mà trong đoạn $\left[x_i-1, x_i+1\right]$ có chứa đúng $k$ số (kể cả $x_i$ ). Gọi $T^{\prime}$ là dãy mới sau khi bỏ $x_i$ đi. Khi đó, số lượng các số thuộc $T^{\prime}$ có trong $\left[x_i-1, x_i+1\right]$ là $k-1$, ngoài ra $x_i$ đã bị bỏ đi thuộc về đúng $2 k-1$ cặp của $A_T$.

Do đó: $A_T=A_{T^{\prime}}+2 k-1$.

Ta viết lại như sau

$\quad\quad\quad\quad \left[x_i-2 ; x_i+2\right]=\left[x_i-2 ; x_i-1\right] \cup\left[x_i-1 ; x_i+1\right] \cup\left[x_i+1 ; x_i+2\right]$

Trừ đoạn ở giữa thì hai đoạn đầu và cuối chứa tối đa $k$ phần tử của $T$. Hơn nữa, do định nghĩa số $x_i$ nên trong đoạn $\left[x_i+1 ; x_i+2\right]$ có tối đa $k-1$ phần tử của $T$. Từ đó có tối đa:

$\quad\quad\quad\quad\quad\quad\quad\quad\quad\quad\quad\quad 2(k-1)+k=3 k-2$

phần tử của $T$ (không tính $x_i$ ) thuộc $\left[x_i-2 ; x_i+2\right]$. Dẫn đến:

$\quad\quad\quad\quad\quad\quad B_T \leq 2(3 k-2)+1+B_{T^{\prime}}=3(2 k-1)+B_{T^{\prime}}$

Áp dụng giả thiết quy nạp, ta có $B_{T^{\prime}}<3 A_{T^{\prime}}$ nên từ các điều trên thì:

$\quad\quad\quad\quad B_T \leq 3(2 k-1)+B_{T^{\prime}}<3(2 k-1)+3 A_{T^{\prime}}=3\left(A_{T^{\prime}}+2 k-1\right)=3 A_T .$

Theo nguyên lý quy nạp, bài toán cũng đúng với $n+1$.

Vậy bài toán được chứng minh hoàn toàn.

Nhận xét. Bài toán này thật ra liên quan đến phương pháp xác suất trong tổ hợp, có thể xem tại quyển “The Probabilistic Method” của GS. Noga Alon. Ta xét một lời giải khác như sau:

(a) Chọn $i$ sao cho số các chỉ số $j$ để $\left|x_i-x_j\right| \leq 1$ là lớn nhất. Khi đó, số lượng chỉ số $j$ sao cho $x_j \in\left(x_i+1, x_i+2\right]$ tối đa là $a_i$, vì nếu không thì tồn tại $j$ để $a_j>a_i$. Tương tự, số lượng chỉ số $j$ sao cho $x_j \in\left[x_i-2, x_i-1\right)$ tối đa là $a_i$.

Chú ý rằng với các chỉ số $j$ để $\left|x_i-x_j\right| \leq 2$ thì ta có điều sau:

$\quad\quad\quad\quad x_j \in\left[x_i-2, x_i-1\right) \cup\left(x_i-1, x_i+1\right) \cup\left(x_i+1, x_i+2\right]$

Số lượng các chỉ số đó chính là $b_i$, dẫn đến $b_i \leq a_i+a_i+a_i=3 a_i$. Hơn nữa, nếu đẳng thức xảy ra, ta phải có mỗi đoạn (hay nửa khoảng) ở phân hoạch trên chứa chính xác $a_i$ chỉ số $j$ của $x_j$.

(b) Bài toán hiển nhiên đúng với $n=1$. Giả sử rằng tồn tại $n>1$ để kết luận không đúng, ta chọn $n$ nhỏ nhất. Ta cũng chọn $i$ sao cho $a_i$ lớn nhất.

Gọi $A^{\prime}, B^{\prime}$ tương ứng là số cặp chỉ số $(k, l)$ mà $\left|x_k-x_l\right| \leq 1$ và $\left|x_k-x_l\right| \leq 2$, trong đó $1 \leq k, l \leq n$ và $k, l \neq i$. Vì $n$ là phản ví dụ nhỏ nhất nên $B^{\prime} \leq 3 A^{\prime}$.

Các cặp chỉ số $(k, l)$ mà $k=i$ hoặc $l=i$ và $\left|x_k-x_l\right| \leq 1$ đều phải có dạng $(k, i)$ hoặc $(i, k)$ trong đó $k \neq i$ và $(i, i)$. Có tổng cộng $2\left(a_i-1\right)+1$ cặp như thế nên $A=A^{\prime}+2\left(a_i-1\right)+1$.

Tương tự thì $B=B^{\prime}+2\left(b_i-1\right)+1$. Do đó nếu $b_i \leq 3 a_i-1$ thì:

$\quad\quad\quad\quad B=B^{\prime}+2 b_i-1 \leq 3 A^{\prime}+2\left(3 a_i-1\right)-1=3\left(A^{\prime}+2 a_i-1\right)=3 A$

Điều này trái với việc $n$ là phản ví dụ nhỏ nhất. Do đó $b_i \geq 3 a_i$. Theo ý (a) thì $b_i \leq 3 a_i$, từ đây phải có $b_i=3 a_i$. Hơn nữa, số lượng chỉ số $j$ để thỏa mãn $x_j \in\left[x_i-2, x_i-1\right)$ hoặc $x_j \in\left(x_i+1, x_i+2\right]$ dều phải bằng $a_i$.

Với mỗi $j, j^{\prime}$ sao cho $x_j, x_{j^{\prime}} \in\left[x_i-2, x_i-1\right)$, ta có $\left|x_j-x_{j^{\prime}}\right|<1$, dẫn đến $a_j \geq a_i$. Mặt khác $a_i$ là lớn nhất có thể nên $a_j=a_i$. Tương tự, với mỗi $j$ sao cho $x_j \in\left(x_i+1, x_i+2\right]$ thì $a_j=a_i$. Như vậy với mọi $j$ sao cho $1<\left|x_i-x_j\right| \leq 2$ thì $a_j=a_i$. Cũng với cách chọn chỉ số $j$ đó, lập luận tương tự như những ý trên, ta cũng phải có $b_j=3 a_j$.

Xây dựng đồ thị $\mathcal{G}$ với các đỉnh được đánh số là $1,2, \ldots, n$ sao cho cặp đỉnh $(k, l)$ kề nhau khi và chỉ khi $1<\left|x_k-x_l\right| \leq 2$. Những lập luận trên cho thấy mọi đỉnh $j$ mà tồn tại một đường đi từ $i$ đến $j$ đều phải thỏa mãn $a_j=a_i$ và $b_j=3 a_j$. Gọi $\mathcal{X}$ là tập hợp tất cả các đỉnh $j$ sao cho tồn tại một đường đi từ $i$ dến $j$ trong $\mathcal{G}$. Đặt $\mathcal{Y}={1,2, \ldots, n} \backslash \mathcal{X}(\mathcal{Y}$ có thể rỗng $)$.

Bây giờ, gọi $A_y, B_y$ tương ứng là số cặp chỉ số $(k, l)$ có tính thứ tự, có thể bằng nhau mà $\left|x_k-x_l\right| \leq 1$ và $\left|x_k-x_l\right| \leq 2$, trong đó $k, l \in \mathcal{Y}$. Chú ý rằng $A_{\mathcal{Y}}=B_{\mathcal{Y}}=0$ nếu $\mathcal{Y}=\emptyset$. Bởi $n$ là phản ví dụ nhỏ nhất, ta phải có $B_{\mathcal{Y}} \leq 3 A_{\mathcal{Y}}$. Ta gọi $a_{y, k}$ và $b_{y, k}$ tương ứng là số chỉ số $j \in \mathcal{Y}$ mà $\left|x_j-x_k\right| \leq 1$ và $\left|x_j-x_k\right| \leq 2$. Định nghĩa tương tự $a_{\mathcal{X}, k}$ và $b_{\mathcal{X}, k}$.

Với mọi $k \in \mathcal{Y}$, dễ thấy $k$ không kề bất cứ đỉnh nào trong $\mathcal{X}$, vì vậy ta có được $b_{\mathcal{X}, k}=0$ và $b_k=b_{\mathcal{Y}, k}+a_{\mathcal{X}, k}$. Từ đây dẫn đến đẳng thức sau:

$\quad\quad\quad\quad\quad\quad B=\sum_{k \in \mathcal{X}} b_k+\sum_{k \in \mathcal{Y}} b_k=3 \sum_{k \in \mathcal{X}} a_k+\sum_{k \in \mathcal{Y}}\left(b_{y, k}+a_{\mathcal{X}, k}\right)$

Ta đồng thời có $\sum_{k \in \mathcal{Y}} b_{y, k}=B_{\mathcal{Y}} \leq 3 A_{\mathcal{Y}}$. Hơn nữa, ta cũng có được:

$\quad\quad A=\sum_{k \in \mathcal{X}} a_k+\sum_{k \in \mathcal{Y}} a_k=\sum_{k \in \mathcal{X}} a_k+\sum_{k \in \mathcal{Y}}\left(a_{\mathcal{Y}, k}+a_{\mathcal{X}, k}\right)=\sum_{k \in \mathbb{X}} a_k+A_{\mathcal{Y}}+\sum_{k \in \mathcal{Y}} a_{\mathcal{X}, k}$

Do đó:

$\quad\quad\quad\quad B \leq 3 A_{\mathcal{Y}}+\sum_{k \in \mathcal{Y}} a_{\mathcal{X}, k}+3 \sum_{k \in \mathcal{X}} a_k \leq 3\left(A_{\mathcal{Y}}+\sum_{k \in \mathcal{Y}} a_{\mathcal{X}, k}+\sum_{k \in \mathcal{X}} a_k\right)=3 A$

Điều này dẫn đến giả sử phản chứng là sai.

Vì vậy, với mọi số nguyên dương $n$, ta phải có $B \leq 3 A$. Bài toán kết thúc.

Bài 3. Cho $p$ là số tự nhiên. Xét phương trình nghiệm nguyên

$\quad\quad\quad\quad\quad\quad\quad\quad\quad\quad\quad\quad x^3+x+p=y^2 .$

(a) Tìm số nguyên tố $p$ nhỏ nhất dạng $4 k+1$ sao cho phương trình có nghiệm.

(b) Chứng minh rằng nếu $p$ là số chính phương thì phương trình trên có nghiệm nguyên dương.

Lời giải. (a) Các số nguyên tố có dạng $4 k+1$ là $5,13,17, \ldots$

Trước hết, ta thấy với $p=13$ thì $x^3+x+13=y^2$ có nghiệm là $(x ; y)=(4 ; 9)$. Ta sẽ chứng minh rằng phương trình $x^3+x+5=y^2$ không có nghiệm nguyên. Xét modulo 4. Có các khả năng sau xảy ra:

  • Khi $x$ chia 4 dư $0,1,2,3$, vế trái chia 4 lần lượt dư $1,3,3,3$.
  • Khi $y$ chia 4 dư $0,1,2,3$, vế phải chia 4 lần lượt dư $0,1,0,1$.

Do đó $y$ phải lẻ và $4 \mid x$. Viết biểu thức đã cho thành:

$\quad\quad\quad\quad\quad\quad\quad\quad\quad\quad (x+3)\left(x^2-3 x+10\right)=y^2+5^2$

Do $x+3 \equiv 3(\bmod 4)$ nên $x+3$ có ước nguyên tố $q \equiv 3(\bmod 4)$. Ta biết rằng với $a, b \in \mathbb{Z}$ thì $a^2+b^2$ chia hết cho số nguyên tố $q \equiv 3(\bmod 4)$ khi và chỉ khi $q \mid a$ và $q \mid b$. Từ đó thì $q \mid 5$ hay $q=5$, mâu thuẫn.

Vậy $p=13$ là số nguyên tố nhỏ nhất cần tìm.

(b) Trước hết, ta giới thiệu kết quả sau (còn gọi là định lý 4 số):

Bổ Đề. Với các số nguyên dương $a, b, c, d$ thoả mãn $a b=c d$ thì tồn tại các số nguyên dương $x, y, z, t$ sao cho $a=x y, b=z t, c=x z, d=y t$.

Chứng minh bổ đề. Đặt $k=\operatorname{gcd}(a, c)$ và viết $a=k a_1, c=k c_1$ thì rõ ràng $\operatorname{gcd}\left(a_1, c_1\right)=1$. Thay vào đề bài, ta có

$\quad\quad\quad\quad\quad\quad\quad\quad\quad\quad k a_1 b=k c_1 d \text { hay } a_1 b=c_1 d .$

Từ đây chú ý $a_1 \mid c_1 d$, nên $a_1 \mid d$, đặt $d=a_1 \ell$. Thay vào thì có $b=\ell c_1$. Từ đó, ta chọn $x=k, y=a_1, z=c_1, t=\ell$ thì có ngay điều phải chứng minh.

Quay lại bài toán, do $p$ là số chính phương nên đặt $p=a^2, a \in \mathbb{Z}$. Ta viết lại phương trình thành dạng:

$\quad\quad\quad\quad\quad\quad\quad\quad x^3+x+a^2=y^2 \text { hay } x\left(x^2+1\right)=(y-a)(y+a) .$

Áp dụng kết quả trên vào bài toán, ta thấy tồn tại các số nguyên dương $m, n, p, q$ để $x=m n, x^2+1=p q, y+a=m p, y-a=n q$. Từ đó:

$\quad\quad\quad\quad\quad\quad\quad\quad\quad\quad (m n)^2+1=p q \text { và } m p-n q=2 a \text {. }$

Xét dãy số $\left(u_n\right)$ xác định bởi $u_0=0, u_1=1, u_{n+2}=\alpha u_{n+1}+u_n$, trong đó $\alpha$ là hằng số mà ta sẽ chọn sau. Rõ ràng với mọi $n$ thì

$\quad\quad\quad\quad\quad\quad\quad\quad u_n^2-u_{n+1} u_{n-1}=(-1)^{n-1}\left(u_1^2-u_2 u_0\right)=(-1)^{n-1} .$

Khi đó, với $n$ chẵn thì $u_n^2-u_{n+1} u_{n-1}=-1$. Chọn $m n=u_{2 k}$. Ta có:

$\quad\quad\quad\quad\quad\quad\quad u_2=\alpha, u_3=\alpha^2+1, u_4=\alpha\left(\alpha^2+2\right), u_5=\alpha^4+3 \alpha^2+1$

Chọn $p=u_3, q=u_5, m n=u_4$ thì rõ ràng $(m n)^2+1=p q$. Bây giờ ta chỉ cần có được

$\quad\quad\quad\quad\quad\quad m u_3-n u_5=2 a \text { hay } m\left(\alpha^2+1\right)-n\left(\alpha^4+3 \alpha^2+1\right)=2 a .$

Từ đây chọn $\alpha=4 a^2$ và viết $m=2 a\left(\alpha^2+2\right), n=2 a\left(\alpha^4+3 \alpha^2+1\right)$ thì đẳng thức trên sẽ thoả mãn, vì

$\quad\quad\quad\quad\quad\quad\quad\quad \left(\alpha^2+1\right)\left(\alpha^2+2\right)-\left(\alpha^4+3 \alpha^2+1\right)=1 .$

Vậy phương trình có một cặp nghiệm cụ thể là

$\quad\quad (x, y)=\left(4 a^2\left(16 a^4+2\right), 2 a\left(16 a^4+2\right)\left(16 a^4+1\right)-a\right) \text { với } a=\sqrt{p} \in \mathbb{Z}^{+} .$

Bài 4. Cho tam giác $A B C$ nhọn nội tiếp đường tròn $(O)$ với $B, C$ cố định và $A$ di động trên $(O)$. $D$ là trung điểm $B C$. Trên $A B$ lấy các điểm $M, P$ và trên $A C$ lấy các điểm $N, Q$ sao cho $D A=D P=D Q$, dồng thời $D M \perp A C, D N \perp A B$.

(a) Chứng minh rằng các điểm $M, N, P, Q$ cùng thuộc một đường tròn $(\mathcal{C})$ và (C) luôn đi qua một điểm cố định.

(b) Chứng minh rằng tâm của $(\mathcal{C})$ luôn thuộc một đường tròn cố định.

Lời giải . (a) Dễ thấy tam giác $A M Q$ cân tại $M$ nên

$\quad\quad \angle D M Q=\angle D M A=90^{\circ}-\angle A=\frac{180^{\circ}-2 \angle A}{2}=\frac{180^{\circ}-\angle P D Q}{2}=\angle D P Q$

Do đó tứ giác $M P D Q$ nội tiếp. Chứng minh tương tự, ta có tứ giác $Q N D P$ nội tiếp nên $M, N, P, Q$ cùng thuộc một đường tròn $(\mathcal{C})$, và $(\mathcal{C})$ luôn đi qua điểm $D$ cố định.

(b) Gọi $K B, K C$ là hai tiếp tuyến của $(O)$. Ta có $D, K, O$ thẳng hàng, lại có:

$\quad\quad\quad\quad\quad \angle B K O=90^{\circ}-\angle B O K=90^{\circ}-\angle B A C=\angle B M D$

Từ đó tứ giác $B D K M$ nội tiếp. Để ý rằng $K D \perp B C$ nên $K M \perp A B$, hơn nữa $D N \perp A B$ nên $K M | D N$. Tương tự thì $K N | D M$. Do đó $D M K N$ là hình bình hành hay $D K, M N$ có $J$ là trung điểm chung.

Gọi $I$ là tâm của $(\mathcal{C})$ thì $I J \perp M N$ và $J L | A D$. Chú ý rằng $D$ là tâm $(A P Q)$ và cũng là trực tâm tam giác $A M N$ nên $P Q, M N$ là hai đường đối song. Đồng thời nếu $L$ là trung điểm $A D$ thì $J L$ vuông góc với đường nối hai chân đường cao từ $M, N$ của tam giác $A M N$ nên $J L \perp P Q$. Lại có $D P=D Q$ và $I P=I Q$ nên $I D \perp P Q$, do đó $J L | D I$.

Từ đây $I D L J$ là hình bình hành và $I L, D J$ có $T$ là trung điểm chung cố định. Xét phép vị tự tâm $D$ tỉ số $\frac{1}{2}$ hợp với phép đối xứng tâm $T$ thì $A \mapsto I$. Do $A$ thuộc đường tròn $(O)$ cố định nên $I$ cũng thuộc đường tròn cố định là ảnh của $(O)$ qua hợp các phép biến hình trên. Bài toán kết thúc.

Nhận xét. Bài toán này còn một hướng tiếp cận bản chất hơn như sau. Nếu gọi $A^{\prime}$ là điểm đối xứng của $A$ qua $D$ thì $K, A^{\prime}$ là hai điểm liên hợp đẳng giác trong tam giác $A B C$, từ đó đường tròn $(\mathcal{C})$ chính là đường tròn đi qua các hình chiếu của $K, A^{\prime}$ trên các cạnh tam giác $A B C$, dồng thời $I$ là trung diểm $K A^{\prime}$.

Dưới đây là một bài toán tương tự: Cho tam giác nhọn $A B C$ nội tiếp đường tròn $(O)$ có $B C$ cố định và $A$ di dộng trên $(O)$. Gọi $H$ là trực tâm tam giác và lấy điểm $E, F$ thuộc $A B, A C$ theo thứ tự đó sao cho $H$ là trung điểm $E F$.

  1. Chứng minh rằng tâm của đường tròn $(A E F)$ luôn thuộc một đường tròn cố định. Đặt là $\omega$.
  2. Giả sử $\omega$ cắt lại $(O)$ tại các điểm $X, Y$. Chứng minh rằng $X, Y, O$ thẳng hàng.

 

Ngày thi thứ hai

Bài 5. Cho số thực $a \neq 0$. Dãy số $\left(u_n\right)$ thoả mãn:

$\quad\quad\quad\quad\quad\quad\quad\quad\quad\quad u_1=0, u_{n+1}\left(u_n+a\right)=a+1 \forall n \in \mathbb{N}^*$

Tìm giới hạn của dãy số $\left(u_n\right)$.

Lời giải: Đặt $x_{n+1}=(a+1) y_n$ và $y_{n+1}=x_n+a y_n$. Ta có:

$\quad\quad\quad\quad\quad\quad\quad y_{n+2}=x_{n+1}+a y_{n+1}=a y_{n+1}+(a+1) y_n$

Đồng thời $u_n=\frac{x_n}{y_n}$. Để ý rằng $u_1=0, u_2=\frac{a+1}{a}$. Chọn $y_1=1, y_2=a$. Từ đó:

$\quad\quad\quad\quad\quad\quad\quad\quad\quad\quad y_n=\frac{(a+1)^n-(-1)^n}{a+2} \forall n \geq 1$

Công thức trên chỉ xác định với $a \neq-2$ nên xét trường hợp $a=-2$, ta có dãy

$\quad\quad\quad\quad\quad\quad\quad\quad\quad\quad \left\{\begin{array}{l}u_1=0, \\ u_{n+1}=\frac{1}{2-u_n}, n \geq 1\end{array} .\right.$

Bằng quy nạp, ta chứng minh được $u_n \in[0 ; 1)$ nên:

$\quad\quad\quad\quad\quad\quad\quad u_{n+1}-u_n=\frac{1}{2-u_n}-u_n=\frac{\left(u_n-1\right)^2}{2-u_n}>0$

Dãy $\left(u_n\right)$ tăng và bị chặn trên bởi 1 nên có giới hạn hữu hạn là $L \in(0,1)$. Giải phương trình giới hạn, ta có được $L=\frac{1}{2-L}$. Khi đó thì $L=1$.

Tiếp theo, xét $a \neq-2$, ta có:

$\quad\quad\quad\quad u_n=\frac{x_n}{y_n}=\frac{(a+1) y_{n-1}}{y_n}=\frac{(a+1)^n+(a+1)(-1)^n}{(a+1)^n-(-1)^n} \forall n \in \mathbb{N}^*$

Đặt $-(a+1)=b \in{-1 ; 1}$, ta viết lại thành:

$\quad\quad\quad\quad\quad\quad\quad\quad\quad\quad u_n=\frac{b^n-b}{b^n-1} \forall n \geq 1$

Có các khả năng sau xảy ra:

  • Nếu $b>1$ hoặc $b<-1$, tương ứng là $a<-2$ hoặc $a>0$, thì $\lim u_n=1$.
  • Nếu $-1<b<1$, tương ứng là $-2<a<0$, thì $\lim u_n=b=-(a+1)$.

Vậy ta có kết luận sau trong các trường hợp của $a$ :

  • Nếu $a \in(-2 ; 0)$ thì $\lim u_n=-(a+1)$.
  • Nếu $a \notin(-2 ; 0)$ thì $\lim u_n=-1$.

Bài 6. Tìm tất cả các hàm số $f: \mathbb{R}^{+} \rightarrow \mathbb{R}^{+}$thoả mãn diều kiện:

$\quad\quad\quad\quad\quad\quad f\left(x f\left(y^2\right)-y f\left(x^2\right)\right)=(y-x) f(x y) \forall x, y \in \mathbb{R}^{+}, x<y .$

Lời giải . Theo giả thiết thì với mọi $y>x>0$, ta đều có

$\quad\quad\quad\quad\quad\quad\quad\quad x f\left(y^2\right)-y f\left(x^2\right)>0 \Rightarrow \frac{f\left(y^2\right)}{f\left(x^2\right)}>\frac{y}{x}>1 .$

Do đó,

$\quad\quad\quad\quad\quad\quad\quad\quad y^2>x^2 \Leftrightarrow y>x \Leftrightarrow f\left(y^2\right)>f\left(x^2\right)$

nên hàm $f$ dã cho đồng biến trên $\mathbb{R}^{+}$. Trong đề bài, thay $y=x+1$, ta có

$\quad\quad\quad\quad\quad\quad\quad f\left(x f\left((x+1)^2\right)-(x+1) f\left(x^2\right)\right)=f(x(x+1))$

hay

$\quad\quad\quad\quad\quad\quad\quad\quad x f\left((x+1)^2\right)-(x+1) f\left(x^2\right)=x(x+1) $

$\quad\quad\quad\quad\quad\quad\quad\quad \Leftrightarrow \frac{f\left((x+1)^2\right)}{x+1}=\frac{f\left(x^2\right)}{x}+1, \forall x>0$

Thực hiện thao tác này nhiều lần, ta có

$\quad\quad\quad\quad\quad\quad\quad\quad \frac{f\left((x+n)^2\right)}{x+n}=\frac{f\left(x^2\right)}{x}+n, \forall x>0, n \in \mathbb{Z}^{+}$

hay

$\quad\quad\quad\quad\quad\quad\quad\quad x f\left((x+n)^2\right)-(x+n) f\left(x^2\right)=n x(x+n) .$

Trong dề bài, thay $y=x+n$, ta có

$\quad\quad\quad\quad\quad\quad f\left(x f\left((x+n)^2\right)-(x+n) f\left(x^2\right)\right)=n f(x(x+n)) $

$\quad\quad\quad\quad\quad\quad \Leftrightarrow f(n x(x+n))=n f(x(x+n)) .$

Với mọi $n \in \mathbb{Z}^{+}, y>0$, ta luôn chọn được $x>0$ để $x(x+n)=y$ nên ta có

$\quad\quad\quad\quad\quad\quad\quad\quad\quad\quad f(n y)=n f(y), \forall n \in \mathbb{Z}^{+}, y \in \mathbb{R}^{+} .$

Đặt $f(1)=a>0$, với mọi $n \in \mathbb{Z}^{+}$, cho $y=\frac{1}{n}$, suy ra

$\quad\quad\quad\quad\quad\quad\quad\quad\quad\quad f(1)=n f\left(\frac{1}{n}\right) \Rightarrow f\left(\frac{1}{n}\right)=\frac{a}{n} .$

Do đó,

$\quad\quad\quad\quad\quad\quad\quad\quad f\left(\frac{n}{m}\right)=n f\left(\frac{1}{m}\right)=\frac{n}{m} a, \forall m, n \in \mathbb{Z}^{+}$

hay $f(x)=a x, \forall x \in \mathbb{Q}^{+}$. Với mọi số thực $x_0>0$, chọn hai dãy số hữu tỷ $\left(a_n\right),\left(b_n\right)$ sao cho $a_n<x_0<b_n$ và $\lim a_n=\lim b_n=x_0$. Rõ ràng

$\quad\quad\quad\quad\quad\quad f\left(a_n\right)<f\left(x_0\right)<f\left(b_n\right) \Rightarrow a \cdot a_n<f\left(x_0\right)<a \cdot b_n,$

nên cho $n \rightarrow+\infty$, ta có $f\left(x_0\right)=a x_0$. Do đó, với mọi số thực $x>0$ thì $f(x)=a x$. Thay vào biểu thức đã cho, ta có

$\quad\quad\quad\quad\quad \left\{\begin{array}{l}f\left(x f\left(y^2\right)-y f\left(x^2\right)\right)=a^2\left(x y^2-x^2 y\right)=a^2(y-x) x y \\ (y-x) f(x y)=a(y-x) x y\end{array}\right.$

nên $a=1$. Vậy tất cả các hàm số cần tìm là $f(x)=x, \forall x>0$.

Nhận xét. Có một điều đáng chú ý ở bài toán này là việc từ giả thiết, ta phải ngầm hiểu rằng $x f\left(y^2\right)-y f\left(x^2\right)>0$ với mọi cặp số dương $x<y$. Ta có thể thêm tường minh điều kiện đó vào đề bài cho rõ. Tuy nhiên, nếu thêm theo kiểu như sau thì sẽ có một chút vấn đề phát sinh:

Tìm tất cả các hàm số $f: \mathbb{R}^{+} \rightarrow \mathbb{R}^{+}$thoả mãn với mọi cặp số dương $x<y$, nếu $x f\left(y^2\right)-y f\left(x^2\right)>0$ thì

$\quad\quad\quad\quad\quad\quad\quad\quad\quad\quad f()=(y-x) f(x y) \forall x, y \in \mathbb{R}^{+}, x<y .$

Khi đó, ta có thể nhận thêm một hàm số thỏa mãn nữa là $f(x)=\sqrt{x}$. Lý do là vì với mọi cặp số $y>x>0$, ta đều có $x f\left(y^2\right)-y f\left(x^2\right)=0$, mà vì thế, điều kiện “nếu” ở trên là sai nên mệnh đề kéo theo là đúng.

Bài 7. Cho $n=2018.2019$. Gọi $A$ là tập hợp các bộ $\left(a_1, a_2, \ldots, a_n\right)$ có thứ tự thoả mãn điều kiện $a_i \in{0,1} \forall i \in{1,2, \ldots, n}$ và $\sum_{i=1}^n a_i=2018^2$.

Có bao nhiêu bộ $\left(a_1, a_2, \ldots, a_n\right)$ từ $A$ dể:

$\quad\quad\quad\quad\quad \sum_{i=1}^k a_i \geq \frac{a}{2} \text { và } \sum_{i=n-k+1}^n a_i \geq \frac{k}{2} \forall k \in{1,2, \ldots, n}$

Lời giải. Ta giải bài toán tổng quát khi thay 2018 bởi $m \in \mathbb{Z}^{+}$. Bài toán đã cho tương đương với bài toán sau:

Trong hệ trục tọa độ Oxy, xét lưới điểm nguyên trong hình chũ nhật có đỉnh dưới bên trái là $O(0 ; 0)$ và dỉnh trên bên phải là $A\left(m^2 ; m\right)$. Dặt $B(m ; m)$ và $C\left(m^2-m ; 0\right)$, hỏi có bao nhiêu đương đi tù̀ $O \rightarrow A$ sao cho mỗi bước, ta đi sang phải hoặc lên trên 1 đơn vị, gọi là đương đi đơn, và không vượt lên trên $O B$ cũng nhu không xuống dưới $A C$ ?

Ở đây, các số $0 ; 1$ tương ứng với các bước đi lên trên, các bước đi sang phải; còn điều kiện tổng $k$ số đầu và tổng $k$ số cuối không nhỏ hơn $\frac{k}{2}$ tương ứng với số lượng bước đi lên không vượt quá số lượng bước đi sang phải. Để thuận tiện, ta gọi đường đi cắt $d$ nếu nó có các phần nằm về cả hai phía của $d$. Trước hết, ta sẽ chứng minh bổ đề sau:

Bổ Đề. Số đường đi đơn từ $O \rightarrow A(m ; n)$, có cắt đường thẳng $y=x$, là $C_{m+n}^{m+1}$.

Thật vậy, Xét đường thẳng $(d): y=x+1$, rõ ràng các đường đi đơn cắt $y=x$ dều sẽ có điểm chung với đường thẳng $(d)$ này. Tại các điểm chung đó, ta thực hiện đối xứng trục để được một đường đi mới xuất phát từ $O \rightarrow A^{\prime}(n-1, m+1)$.

Trong hình trên, đường cũ là đứt nét, còn đường mới là liền nét. Rõ ràng phép đối xứng trục trên là song ánh, biến các đường cần tìm (cắt $y=x$ ), thành các đường từ $O \rightarrow A^{\prime}$; do đó, số lượng đường cần tìm là $C_{m+n}^{n-1}$.

Trở lại bài toán,

Số đường đi đơn từ $O \rightarrow A\left(m^2 ; m\right)$ là $C_{m^2+m}^m$ vì nó bằng số cách chọn $m$ lần đi lên trong tổng số $m^2+m$ lần di chuyển, trong đó số đường đi cắt $O B$ bằng số đường đi cắt $A C$ và bằng $C_{m^2+m}^{m-1}$ (theo bổ đề).

Do đó, ta chỉ cần tìm số đường đi cắt cả $O B, A C$ với ý tưởng đối xứng hai lần đã dùng để chứng minh bổ đề.

Đầu tiên, ta thực hiện đối xứng qua đường thẳng $y=x+1$; khi đó, các đường đi đơn sẽ xuất phát từ $O \rightarrow A^{\prime}\left(m-1 ; m^2+1\right)$. Do các đường ban đầu còn vượt qua $A C$ nên các đường mới phải cắt thêm $y=x+m^2-m+3$. Tiếp tục đối xứng qua đường thẳng này, ta đưa về đếm số đường đi đơn từ $O \rightarrow A^{\prime \prime}\left(m-2, m^2+2\right)$. Suy ra số đường đi trong trường hợp này là $C_{m^2+m}^{m-2}$. Vậy theo nguyên lý bù trừ, kết quả cần tìm sẽ là

$\quad\quad\quad\quad\quad\quad\quad\quad\quad\quad C_{m^2+m}^m-2 C_{m^2+m}^{m-1}+C_{m^2+m}^{m-2} .$

Thay $m=2018$, ta có số lượng đường đi, cũng chính là số bộ thỏa mãn đề bài.

Nhận xét. Dưới đây là một số kết quả tương tự về đường đi đơn trong đề bài

$1$. Số đường đi đơn từ $(0 ; 0) \rightarrow(m ; n)$ mà không có điểm chung với $y=x$ là

$\quad\quad\quad\quad\quad\quad\quad\quad\quad\quad\quad\quad \frac{m-n}{m+n} C_{m+n}^m .$

$2$. Số đường đi đơn từ $(0 ; 0) \rightarrow(m ; n)$ mà không vượt qua $y=x$ là

$\quad\quad\quad\quad\quad\quad\quad\quad\quad\quad\quad C_{m+n}^n-C_{m+n}^{n-1} .$

$3$. Số đường đi gồm $n$ bước mà không vượt $y=x$ là

$\quad\quad\quad\quad\quad\quad\quad\quad\quad \sum_{i=n / 2}^n \frac{n !(2 i+1-n)}{(i+1) !(n-i) !}=C_n^{[n / 2]} .$

$4$. Số đường đi đơn từ $(0 ; 0) \rightarrow(m ; n)$ mà không có điểm chung với $y=x+t$ là

$\quad\quad\quad\quad\quad\quad\quad\quad\quad\quad\quad\quad C_{m+n}^n-C_{m+n}^{m-t}$.

Bạn đọc có thể dùng phương pháp tương tự trên để giải quyết các bài toán này.

Bài 8. Đường tròn $(\mathcal{C})$ tâm $I$ nội tiếp tam giác $A B C$ và tiếp xúc với các cạnh $A B, A C$ tại $E, F$. $A M, A N$ là các đường phân giác trong, phân giác ngoài của góc $\angle B A C(M, N$ nằm trên $B C)$. Gọi $d_M, d_N$ lần lượt là các tiếp tuyến của $(\mathcal{C})$ qua $M, N$ và khác $B C$.

(a) Chứng minh rằng $d_M, d_N, E F$ dồng quy tại điểm $D$.

(b) Lấy trên $A B, A C$ các điểm $P, Q$ thoả mãn $D P|A C, D Q| A B$. Gọi $R, S$ là trung điểm của $D E, D F$. Chứng minh rằng $I$ thuộc đường thẳng qua các trực tâm của hai tam giác $D P S, D Q R$.

Lời giải. (a) Gọi $X, Y$ lần lượt là tiếp điểm của tiếp tuyến thứ hai kẻ từ $M$ dến $(I)$ và $D^{\prime}$ là tiếp điểm của $(I)$ trên $B C$. Gọi $K$ là trung điểm $E F$.

Xét trong đường tròn $(I)$ thì $E F$ là đường đối cực của $A$ và $K \in E F$ nên đối cực của $K$ sẽ đi qua $A$, mà $N A \perp I A$ nên $N A$ chính là đường đối cực của $K$.

Đường đối cực của $K$ đi qua $N$ nên đối cực của $N$, là $D^{\prime} Y$, sẽ đi qua $K$. Dễ thấy rằng $A M$ là trục đối xứng của tứ giác $D^{\prime} X E F$ nên suy ra $D^{\prime} X | E F$. Xét $D^{\prime}(E F, X Y)$, ta có có $D^{\prime} Y$ đi qua trung điểm của $E F$ và $D^{\prime} X | E F$ nên

$\quad\quad\quad\quad\quad\quad\quad\quad\quad\quad D^{\prime}(E F, X Y)=-1$

hay tứ giác $E X F Y$ điều hòa. Suy ra $M X, N Y, E F$ đồng quy. Ngoài ra ta cũng có $X, Y, A$ thẳng hàng.

(b) Dễ thấy các tam giác $P E D$ và $D Q F$ là các tam giác cân. Gọi $H_1, H_2$ lần lượt là trực tâm của tam giác $\triangle D P S, \triangle D Q R$. Ta có

$\quad\quad\quad\quad\quad\quad\quad\quad \angle P H_1 S=\angle P D F=\angle A F E=\angle P E S$

nên $E P S H_1$ là tứ giác nội tiếp. Suy ra $R H_1 \cdot R P=R S \cdot R E$. Ngoài ra,

$\quad\quad\quad\quad\quad\quad\quad\quad\quad\quad K A \cdot K I=K E \cdot K F$

nên

$\quad\quad\quad\quad\quad\quad\quad\quad\quad \frac{R P}{K A} \cdot \frac{R H_1}{K I}=\frac{R E}{K E} \cdot \frac{R S}{K F} .$

Theo định lý Thales thì $\frac{R P}{K A}=\frac{R E}{K E}$ nên $\frac{R H_1}{K I}=\frac{R S}{K F}$, mà

$\quad\quad\quad\quad\quad\quad\quad R S=R D-S D=\frac{D E-D F}{2}=\frac{E F}{2}=K F$

Suy ra $R H_1=K I$, mà $R H_1 | K I$ (do cùng vuông góc với $E F$ ) nên $I K R H_1$ là hình chữ nhật, kéo theo $I H_1 | E F$. Một cách tương tự, ta có $I H_2 | E F$ vậy nên đường thẳng $H_1 H_2$ đi qua $I$.

Nhận xét. Trong câu a, tính chất $A, X, Y$ thẳng hàng của bài toán cũng đúng khi thay $M, N$ là chân các đường phân giác bởi cặp điểm liên hợp điều hòa bất kỳ với $B, C$. Điều này có được nhờ tính chất của các đường đối cực (hoặc có thể chứng minh nhờ việc sử dụng phép chiếu trực giao các chùm điều hòa).

 

 

 

 

 

 

 

 

 

 

 

 

 

 

 

 

 

 

 

 

 

 

 

 

 

 

 

 

 

 

 

 

 

 

 

 

 

 

 

 

 

 

 

 

 

 

 

 

 

 

 

 

 

 

 

 

 

 

 

 

 

 

 

 

 

 

 

 

 

 

 

 

 

 

 

 

 

 

 

 

 

 

 

 

 

 

 

 

 

 

 

 

 

 

 

 

 

 

 

 

 

 

 

 

 

 

 

 

 

 

 

 

 

 

 

 

 

 

 

 

 

 

 

 

Đề thi và đáp án kì thi chọn đội tuyển thi Quốc gia trường Phổ thông Năng khiếu năm học 2019 – 2020

ĐỀ THI

Ngày thi thứ nhất

Bài 1. Số thực $\alpha$ được gọi là điểm tụ của dãy số $\left(u_n\right)$ nếu tồn tại ít nhất một dãy con của $\left(u_n\right)$ có hội tụ đến $\alpha$.

(a) Hãy chỉ ra một dãy số có vô hạn điểm tụ.

(b) Chứng minh rằng nếu dãy số có mọi dãy con hội tụ thì nó cũng hội tụ.

(c) Gọi $S$ là tập hợp tất cả các số chính phương dương. Dãy số $\left(a_n\right)$ xác định bởi $a_n=\frac{1}{n}$ nếu $n \in S$ và $a_n=\frac{1}{n^2}$ nếu $n \notin S$.

Đặt $b_n=\sum_{k=1}^n a_k$. Xét tính hội tụ của các dãy số $\left(a_n\right)$ và $\left(b_n\right)$.

Bài 2. Tìm tất cả các hợp số dương $n$ sao cho $\sigma(n) \equiv 2(\bmod \varphi(n))$, trong đó ký hiệu $\sigma(n), \varphi(n)$ là hàm tổng các ước của $n$ và hàm Euler.

Bài 3. Tìm tất cả các hàm số $f: \mathbb{R} \rightarrow \mathbb{R}$ thỏa mãn

$\quad\quad\quad\quad\quad f(f(x)+y)+f(x) f(f(y))=x f(y)+x+y, \forall x, y \in \mathbb{R} .$

Bài 4. Cho tam giác $A B C$ không cân nội tiếp trong đường tròn $(O)$ với $B C$ cố định và $A$ thay đổi trên cung lớn $B C$. Các đường tròn bàng tiếp góc $A, B, C$ lần lượt tiếp xúc với $B C, C A, A B$ tại $D, E, F$. Gọi $L, M, N$ lần lượt là giao điểm khác $A, B, C$ của $(A B E),(A C F) ;(B C F),(B A D) ;(C A D),(C B E)$.

(a) Chứng minh rằng $A L$ luôn đi qua điểm cố định khi $A$ thay đổi.

(b) Gọi $K, I, J$ lần lượt là trung điểm của $A D, B E, C F$. Chứng minh rằng $K L, I M, J N$ dồng quy.

Ngày thi thứ hai

Bài 5. Cho $a, b, c$ là các số thực dương thỏa mãn $8\left(a^2+b^2+c^2\right)=9(a b+b c+c a)$.

Tìm giá trị lớn nhất và giá trị nhỏ nhất của biểu thức

$\quad\quad\quad\quad\quad\quad\quad\quad\quad\quad T=\frac{a+b}{c}+\frac{b+c}{a}+\frac{c+a}{b} .$

Bài 6. Tìm tất cả các hàm số $f: \mathbb{Z}^{+} \rightarrow \mathbb{Z}^{+}$thỏa mãn đồng thời các điều kiện sau

$\quad\quad$ i) $m f(m)+n f(n)+2 m f(n)$ là số chính phương với mọi $m, n$;

$\quad\quad$ ii) $f(m n)=f(m) f(n)$ với mọi $m, n$ nguyên dương;

$\quad\quad$  iii) Với mọi số nguyên tố $p, f(p)$ không chia hết cho $p^2$.

Bài 7. Một trường phổ thông có $n$ học sinh. Các học sinh tham gia vào tổng cộng $m$ câu lạc bộ là $A_1, A_2, \ldots, A_m$.

(a) Chứng minh rằng nếu mỗi câu lạc bộ có 4 học sinh và hai học sinh bất kỳ tham gia chung nhất một câu lạc bộ thì $m \leq \frac{n(n-1)}{12}$.

(b) Giả sử tồn tại $k>0$ sao cho hai câu lạc bộ bất kỳ có chung nhau $k$ thành viên và tồn tại một câu lạc bộ $A_t$ có $k$ thành viên. Chứng minh rằng $m \leq n$

Bài 8. Cho tam giác $A B C$ nội tiếp đường tròn $(O)$. Đường tròn nội tiếp $(I)$ tiếp xúc với các cạnh $B C, C A, A B$ lần lượt tại $D, E, F$. Gọi $J$ là tâm bàng tiếp góc $A$ của tam giác $A B C$ và $H$ là hình chiếu của $D$ lên $E F$.

(a) Chứng minh rằng giao điểm của $A H, J D$ thì thuộc đường thẳng $O I$.

(b) Giả sử $D H$ cắt lại $(I)$ ở $K$ và $I K$ cắt lại đường tròn ngoại tiếp $(I E F)$ ở $L$. Chứng minh rằng $A D, L H$ cắt nhau tại một điểm nằm trên $(I E F)$.

 

LỜI GIẢI

Ngày thi thứ nhất

Bài 1. Số thực $\alpha$ được gọi là điểm tụ của dãy số $\left(u_n\right)$ nếu tồn tại ít nhất một dãy con của $\left(u_n\right)$ có hội tụ đến $\alpha$.

(a) Hãy chỉ ra một dãy số có vô hạn điểm tụ.

(b) Chứng minh rằng nếu một dãy số có mọi dãy con hội tụ thì nó cũng hội tụ.

(c) Gọi $S$ là tập hợp tất cả các số chính phương dương. Dãy số $\left(a_n\right)$ xác định bởi $a_n=\frac{1}{n}$ nếu $n \in S$ và $a_n=\frac{1}{n^2}$ nếu $n \notin S$.

Đặt $b_n=\sum_{k=1}^n a_k$. Xét tính hội tụ của các dãy số $\left(a_n\right)$ và $\left(b_n\right)$.

Lời giải. (a) Ta sẽ chỉ ra dãy số mà mỗi số nguyên dương xuất hiện vô hạn lần trong đó. Chẳng hạn $\left(u_n\right)$ : là

$\quad\quad\quad\quad\quad\quad\quad\quad\quad\quad 1,2,1,2,3,1,2,3,4,1,2,3,4,5, \ldots$

với $u_n=1$ nếu $n \in S$ và $u_{n+1}=u_n+1$ nếu $n \notin S$, trong đó $S$ là tập hợp các số có dạng $\frac{m(m+1)}{2}$ như $1,3,6,10,15, \ldots$ Khi đó, với mỗi số nguyên dương $m \in \mathbb{Z}^{+}$thì ta luôn có thể trích ra một dãy con vô hạn của $\left(u_n\right)$ có tất cả các phần tử đều bằng $m$, tức là hội tụ về $m$.

(b) Do mỗi dãy số là dãy con của chính nó nên rõ ràng khẳng định của bài toán là đúng.

(c) Ta có $0 \leq a_n \leq \frac{1}{n}$ với mọi $n$ nên theo nguyên lí kẹp, ta suy ra $\lim a_n=0$. Nhận xét rằng $b_n$ là dãy tăng. Ta có

$\quad\quad\quad\quad\quad b_{n^2}=\sum_{i=1}^{n^2} a_i =\sum_{i \in S, i=1}^{n^2} a_i+\sum_{i \notin S, i=1}^{n^2} a_i=\sum_{i \in S, i=1}^{n^2} \frac{1}{i}+\sum_{i \notin S, i=1}^{n^2} \frac{1}{i^2} $

$\quad\quad\quad\quad\quad\quad\quad\quad\quad\quad\quad =\left(1+\frac{1}{2^2}+\cdots+\frac{1}{n^2}\right)+\sum_{i \notin S, i=1}^{n^2} \frac{1}{i^2} $

$\quad\quad\quad\quad\quad\quad\quad\quad\quad\quad\quad <\left(1+\frac{1}{2^2}+\cdots+\frac{1}{n^2}\right)+\left(\sum_{i=1}^{n^2} \frac{1}{i^2}\right) $

$\quad\quad\quad\quad\quad\quad\quad\quad\quad\quad\quad =\left(1+\frac{1}{2^2}+\cdots+\frac{1}{n^2}\right)+\left(1+\frac{1}{2^2}+\cdots+\frac{1}{n^4}\right)$

Vì dãy $u_n=1+\frac{1}{2^2}+\cdots+\frac{1}{n^2}<1+\frac{1}{1 \cdot 2}+\frac{1}{2 \cdot 3}+\cdots+\frac{1}{(n-1) n}=2-\frac{1}{n}<2$ là bị chặn trên nên từ đánh giá đã xây dựng được, ta có $b_{n^2}$ cũng bị chặn trên. Kết hợp với $b_n$ là dãy tăng, ta suy ra bản thân dãy $b_n$ cũng bị chặn trên nên nó hội tụ.

Bài 2. Tìm tất cả các hợp số dương $n$ sao cho

$\quad\quad\quad\quad\quad\quad\quad\quad\quad\quad \sigma(n) \equiv 2 \quad(\bmod \varphi(n)),$

trong đó ký hiệu $\sigma(n), \varphi(n)$ là hàm tổng các ước của $n$ và hàm Euler.

Lời giải . Giả sử $p$ là một ước nguyên tố lẻ của $n$. Nếu $v_p(n)>1$ thì theo công thức của hàm Euler, ta có $p \mid \varphi(n)$, mà $n \cdot \sigma(n)-2$ chia hết cho $\varphi(n)$, tức là cũng chia hết cho $p$ nên kéo theo $p \mid 2$, vô lý. Suy ra $v_p(n)=1$ với mọi $p \mid n$.

Đặt $n=2^k \cdot p_1 p_2 \ldots p_t$ với $k \geq 0$ và $p_1<p_2<\ldots<p_t$ là các số nguyên tố phân biệt. Theo công thức tính các hàm, ta có

$\quad\quad\quad\quad\quad\quad\quad \varphi(n)=2^{k-1}\left(p_1-1\right)\left(p_2-1\right) \ldots\left(p_t-1\right)$

$\quad\quad\quad\quad\quad\quad\sigma(n)=\left(2^{k+1}-1\right)\left(p_1+1\right)\left(p_2+1\right) \ldots\left(p_t+1\right) .$

Đánh giá lũy thừa 2 trong các số trên, ta có

$\quad\quad\quad\quad\quad\quad v_2(\varphi(n)) \geq k-1+t \text { và } v_2(n \cdot \sigma(n)) \geq k+t .$

Do đó từ $\varphi(n) \mid n \cdot \sigma(n)-2$, ta suy ra $1 \geq k-1+t$ nên $k+t \leq 2$. Ta xét các trường hợp sau

  • Nếu $t=0$ thì $n=2^k$ là hợp số nên $k=2, n=4$, thử trực tiếp ta thấy thỏa.
  • Nếu $t=1$ thì $n=2 p$ nên $\varphi(n)=p-1, \sigma(n)=3(p+1)$ và đưa về

$\quad\quad\quad\quad\quad\quad\quad\quad\quad\quad p-1 \mid 6 p(p+1)-2$

Chú ý rằng

$\quad\quad\quad\quad\quad\quad 6 p(p+1)-2=6 p^2+6 p-2=(p-1)(6 p+12)-10$

nên $p-1 \mid 10$. Từ đó ta tìm được $p=3, p=11$ tương ứng với $n=6, n=22$.

  • Nếu $t=2$ thì $k=0$, ta có $n=p_1 p_2$ nên

$\quad\quad\quad\quad \varphi(n)=\left(p_1-1\right)\left(p_2-1\right) \text { và } \sigma(n)=\left(p_1+1\right)\left(p_2+1\right)$

đưa về

$\quad\quad\quad\quad \left(p_1-1\right)\left(p_2-1\right) \mid\left(p_1+1\right)\left(p_2+1\right)-2 .$

Điều này không thể xảy ra vì $\left(p_1-1\right)\left(p_2-1\right)$ chia hết cho 4 trong khi biểu thức còn lại chia 4 dư 2 . Do đó, trường hợp này không có số $n$ thỏa mãn.

Vậy tất cả các số cần tìm là $4,6,22$.

Nhận xét. Chú ý rằng mọi số nguyên tố đều thỏa mãn yêu cầu của đề bài.

Bài 3. Tìm tất cả các hàm số $f: \mathbb{R} \rightarrow \mathbb{R}$ thỏa mãn

$\quad\quad\quad\quad\quad\quad f(f(x)+y)+f(x) f(f(y))=x f(y)+x+y$

với mọi số thực $x, y$.

Lời giải. Thay $x=y=0$ vào phương trình đề cho, ta có

$\quad\quad\quad\quad\quad\quad\quad\quad\quad\quad f(f(0))+f(0) f(f(0))=0 .$

suy ra $f(f(0))=0$ hoặc $f(0)=-1$. Ta xét các trường hợp sau:

  1. Nếu $f(f(0))=0$. Thay $y=0$, vào phương trình dề cho, ta có $f(f(x))=$ $x f(0)+x, \forall x \in \mathbb{R}$ Thay $x=f(0)$ và sử dụng $f(f(0))=0$, ta được $f(0)=$ $[f(0)]^2+f(0)$, hay $f(0)=0$. Do đó $f(f(x))=x$ với mọi $x \in \mathbb{R}$. Thay vào phương trình đề bài, ta có

$\quad\quad\quad\quad\quad\quad f(f(x)+y)+y f(x)=x f(y)+x+y, \forall x, y \in \mathbb{R} .$

Thay $y$ bởi $f(y)$ và sử dụng tính đối xứng của vế trái, ta được

$\quad\quad\quad\quad\quad\quad f(f(x)+f(y))+f(x) f(y)=x y+x+f(y)=x y+y+f(x) .$

Do đó $f(x)-x=f(y)-y$ với mọi $x, y \in \mathbb{R}$, hay $f(x)=x+c$. Thử lại, ta có $c=0$.

  1. Nếu $f(0)=-1$. Thay $y=0$ vào phương trình đề cho, ta có $f(f(x))+$ $f(x) f(-1)=0, \forall x \in \mathbb{R}$. Từ đây suy ra $f(f(-1))=-[f(-1)]^2$. Thay $x=0$ vào phương trình đề cho, ta có $f(y-1)-f(f(y))=y, \forall y \in \mathbb{R}$. Kết hợp các đẳng thức trên lại, ta có

$\quad\quad\quad\quad\quad\quad\quad\quad f(x-1)+f(x) f(-1)=x, \forall x \in \mathbb{R} .$

Thay $y=-1$ vào phương trình đề cho và sử dụng $f(f(-1))=-[f(-1)]^2$, ta lại có

$\quad\quad\quad\quad\quad\quad f(f(x)-1)-f(x)[f(-1)]^2=x f(-1)+x-1, \forall x \in \mathbb{R} .$

Mặt khác, ta cũng có

$\quad\quad\quad\quad\quad\quad\quad\quad f(-1) f(f(x))+f(x)[f(-1)]^2=0, \forall x \in \mathbb{R} .$

Cộng vế theo vế hai biểu thức trên lại, ta có

$\quad\quad\quad\quad\quad\quad\quad\quad\quad\quad f(x)=[1+f(-1)] x+1, \forall x \in \mathbb{R} .$

Thử lại, ta thấy không thỏa mãn.

Vậy phương trình có nghiệm hàm duy nhất là $f(x)=x$.

Bài 4. Cho tam giác $A B C$ không cân nội tiếp trong đường tròn $(O)$ với $B C$ cố định và $A$ thay đổi trên cung lớn $B C$. Các đường tròn bàng tiếp góc $A, B, C$ lần lượt tiếp xúc với các cạnh $B C, C A, A B$ tại $D, E, F$. Gọi $L, M, N$ lần lượt là giao điểm khác $A, B, C$ của các cặp đường tròn

$\quad\quad\quad\quad (A B E),(A C F) ;(B C F),(B A D) ;(C A D),(C B E) .$

(a) Chứng minh rằng $A L$ luôn đi qua điểm cố định khi $A$ thay đổi.

(b) Gọi $K, I, J$ lần lượt là trung điểm của $A D, B E, C F$. Chứng minh rằng $K L, I M, J N$ đồng quy.

Lời giải . (a) Đặt $B C=a, C A=b, A B=c$ và $p$ là nửa chu vi thì theo tính chất tiếp điểm bàng tiếp, ta có $B F=C E=p-a$.

Bằng biến đổi góc, ta có được $\triangle L B F \sim \triangle L E C(g . g)$, mà $B F=C E$ nên hai tam giác này bằng nhau. Suy ra $L B=L E, L C=L F$ nên $L$ là trung điểm cung $B E$ của đường tròn $(A B E)$ và cũng là trung diểm cung $C F$ của $(A C F)$.

Từ đó ta có $A L$ là phân giác góc $B A C$ hay $A L$ luôn đi qua trung điểm cung nhỏ $B C$ của $(O)$, là điểm cố định.

(b) Để ý rằng vai trò của $M, N, L$ là bình đẳng trong tam giác $A B C$. Do đó, từ câu a, một cách tương tự, ta có $M, N$ thuộc phân giác góc $B, C$ nên cũng lần lượt là trung điểm các cung nhỏ của các đường tròn tương ứng. Suy ra $M, K, N$ thẳng hàng (cùng thuộc trung trực của đoạn $A D$ ); tương tự với các bộ ba $N, I, L$ và $L, J, M$. Cuối cùng, ta thấy rằng

$\quad\quad\quad\quad\quad\quad\quad \frac{K M}{K N}=\frac{A K \cdot \tan \angle M A K}{A K \cdot \tan \angle N A K}=\frac{\tan (B / 2)}{\tan (C / 2)} .$

Tương tự với các tỷ số khác. Đến đây, áp dụng định lý Ceva cho tam giác $M N L$, ta có các đoạn thẳng $L K, I M, J N$ dồng quy.

Nhận xét. Một cách khác cho câu a như sau: Xét phép nghịch đảo đối xứng với phương tích $k=A B \cdot A C$ và trục đối xứng là phân giác góc $A$. Ta có $E \rightarrow E^{\prime} \in$ $A C, F \rightarrow F^{\prime} \in A B$ sao cho $A E \cdot A E^{\prime}=A F \cdot A F^{\prime}=k$. Ta tính được

$\quad\quad\quad\quad A E^{\prime}=\frac{b c}{p-c} \rightarrow B E^{\prime}=\frac{c(p-a)}{p-c} \rightarrow \frac{E^{\prime} B}{E^{\prime} A}=\frac{p-a}{b} .$

Tương tự thì $\frac{F^{\prime} C}{F^{\prime} A}=\frac{p-a}{c}$. Áp dụng định lý Ceva cho tam giác $A B C$ thì $C E^{\prime}, B F^{\prime}$ và phân giác góc $A$ đồng quy.

Lại có qua phép nghịch đối xứng trên thì phân giác giữ nguyên,

$\quad\quad\quad\quad\quad\quad\quad\quad\quad\quad (A B E) \rightarrow C F^{\prime},(A C F) \rightarrow B E^{\prime}$

nên ta có $L$ thuộc phân giác góc $A$.

 

Ngày thi thứ hai

Bài 5. Cho $a, b, c$ là các số thực dương thỏa mãn $8\left(a^2+b^2+c^2\right)=9(a b+b c+c a)$. Tìm giá trị lớn nhất và giá trị nhỏ nhất của biểu thức

$\quad\quad\quad\quad\quad\quad\quad\quad\quad\quad T=\frac{a+b}{c}+\frac{b+c}{a}+\frac{c+a}{b} .$

Lời giải . Do tính thuần nhất đối xứng của các biến nên chuẩn hóa

$\quad\quad\quad\quad\quad a b+b c+c a=8 \rightarrow a^2+b^2+c^2=9 \rightarrow a+b+c=5 .$

Ta có $P+3=(a+b+c)\left(\frac{1}{a}+\frac{1}{b}+\frac{1}{c}\right)=\frac{40}{a b c}$ nên ta đưa về tìm min, max của $T=a b c$ trong điều kiện

$\quad\quad\quad\quad\quad\quad\quad\quad\quad\quad\quad \left\{\begin{array}{l}a+b+c=5 \\ a b+b c+c a=8\end{array}\right.$

Chú ý rằng $b+c=5-a, b c=8-a(b+c)=8-a(5-a)$ nên từ đánh giá quen thuộc $(b+c)^2 \geq 4 b c$, ta có

$\quad\quad\quad\quad\quad\quad (5-a)^2 \geq 4\left(8-5 a+a^2\right) \Leftrightarrow 1 \leq a \leq \frac{7}{3} .$

Suy ra $T=a b c=a\left(8+a^2-5 a\right)=f(a)$. Đến đây khảo sát hàm số này trên miền $\left[1 ; \frac{7}{3}\right]$, ta được $\min T=4, \max T=\frac{112}{27}$ nên $\min P=\frac{93}{14}$, $\max P=7$. Từ đó, ta thu được kết luận như sau

  • Giá trị lớn nhất của $P$ là 7 , đạt được chẳng hạn khi $(a, b, c)=(2,2,1)$.
  • Giá trị nhỏ nhất của $P$ là $\frac{93}{14}$, đạt được chẳng hạn khi $(a, b, c)=\left(\frac{7}{3}, \frac{4}{3}, \frac{4}{3}\right)$.

Bài 6. Tìm tất cả các hàm số $f: \mathbb{Z}^{+} \rightarrow \mathbb{Z}^{+}$thỏa mãn đồng thời các điều kiện sau đây

$\quad\quad$ i) $m f(m)+n f(n)+2 m f(n)$ là số chính phương với mọi $m, n$;

$\quad\quad$  ii) $f(m n)=f(m) f(n)$ với mọi $m, n$ nguyên dương;

$\quad\quad$  iii) Với mọi số nguyên tố $p, f(p)$ không chia hết cho $p^2$.

Lời giải . Thay $m=n=1$ vào ii), ta suy ra $f(1)=f(1)^2$ nên $f(1)=1$. Thay $m=n$ vào i), ta suy ra $4 m f(m)$ là số chính phương với mọi $m \in \mathbb{Z}^{+}$nên $m f(m)$ cũng là số chính phương với mọi $m \in \mathbb{Z}^{+}$.

Với $p$ là số nguyên tố, vì $p f(p)$ là số chính phương nên $p \mid f(p)$ và ta đặt $f(p)=k^2 p$, với $k$ là số nguyên dương nào đó. Thay $m=p, n=1$ vào i), ta suy ra $p f(p)+1+2 p$ là số chính phương, hay $k^2 p^2+2 p+1$ là số chính phương.

Vì $k^2 p^2+2 p+1>(k p)^2$ nên

$\quad\quad\quad\quad\quad\quad k^2 p^2+2 p+1 \geq(k p+1)^2=k^2 p^2+2 k p+1 .$

Do đó $2 p \geq 2 k p$ nên ta phải có $k=1$.

Vì thế nên $f(p)=p$ với mọi số nguyên tố $p$. Sử dụng điều kiện ii), hàm $f$ nhân tính, và cũng vì mọi số nguyên dương bất kỳ đều có thể viết dưới dạng tích của các số nguyên tố nên ta có được $f(n)=n$ với mọi $n \in \mathbb{Z}^{+}$.

Thử lại ta thấy hàm số này thỏa mãn các ràng buộc của đề bài.

Nhận xét. Trên thực tế, ta có thể bỏ bớt diều kiện ii), iii) đi mà bài toán gốc vẫn có thể giải quyết được. Cụ thể như sau:

Chứng minh rằng nếu hàm số $f: \mathbb{Z}^{+} \rightarrow \mathbb{Z}^{+}$thỏa mãn $f(1)=1$ và với mọi $m, n \in \mathbb{Z}^{+}$, ta có $m f(m)+n f(n)+2 m f(n)$ là số chính phương thì $f(n)=n, \forall n \in \mathbb{Z}^{+}$.

Bài 7. Một trường phổ thông có $n$ học sinh. Các học sinh tham gia vào tổng cộng $m$ câu lạc bộ là $A_1, A_2, \ldots, A_m$.

(a) Chứng minh rằng nếu mỗi câu lạc bộ có 4 học sinh và hai học sinh bất kỳ tham gia chung nhất một câu lạc bộ thì $m \leq \frac{n(n-1)}{12}$.

(b) Giả sử tồn tại $k>0$ sao cho hai câu lạc bộ bất kỳ có chung nhau $k$ thành viên và tồn tại một câu lạc bộ $A_t$ có $k$ thành viên. Chứng minh rằng $m \leq n$

Lời giải . (a) Gọi $S$ là số bộ $({A, B}, C)$ mà trong đó học sinh $A, B$ cùng tham gia vào câu lạc bộ $C$. Ta thực đếm $S$ bằng hai cách

  1. Chọn câu lạc bộ trước, có $m$ cách, chọn cặp học sinh cùng tham gia vào đó có $C_4^2=6$ cách nên $S=6 \mathrm{~m}$.
  2. Chọn cặp học sinh trước, có $C_n^2$ cách, chọn câu lạc bộ mà hai học sinh đó cùng tham gia, có không quá 1 cách nên $S \leq C_n^2$.

Từ đó suy ra

$\quad\quad\quad\quad\quad\quad\quad\quad\quad 6 m \leq C_n^2 \Leftrightarrow m \leq \frac{n(n-1)}{12} .$

(b) Xét câu lạc bộ $X$ nào đó có $k$ thành viên. Xét $m-1$ câu lạc bộ còn lại thì theo giả thiết, rõ ràng các câu lạc bộ này đều có chứa $k$ thành viên trên của câu lạc bộ $X$. Từ đó suy ra $m-1$ câu lạc bộ còn lại đôi một không có thành viên chung.

Xét $n-k$ học sinh còn lại của trường thì rõ ràng một học sinh thuộc tối đa một câu lạc bộ (trong số các câu lạc bộ còn lại), suy ra số câu lạc bộ còn lại không vượt quá $n-k$ nên suy ra $m \leq n-k+1 \leq n$. Ta có điều phải chứng minh.

Nhận xét. Ý b của bài toán khá hiển nhiên, nhưng thực ra nó là một “phiên bản dễ” của bất đẳng thức Fisher sau đây:

Cho $A_1, A_2, \ldots, A_m$ là các tập con của tập ${1,2, \ldots, n}$ sao cho hai tập con bất kỳ có chung nhau đúng $k$ (với $k$ là số nguyên cố định nào đó không vượt quá n). Khi đó $m \leq n$.

Tuy nhiên, chứng minh sơ cấp cho kết quả này quả thực rất khó. Cách phổ biến nhất là dùng đại số tuyến tính. Cụ thể là:

Ta đặt tương ứng mỗi tập $A_i$ với một vector $v_i$ trong $\mathbb{F}_2^n$ như sau

$\quad\quad\quad\quad\quad\quad\quad\quad v_{i j}=\left\{\begin{array}{l}1 \text { nếu } j \in A_i \\ 0 \text { nếu } j \notin A_i\end{array} .\right.$

Chú ý rằng $\left|A_i \cap A_j\right|=k$ với mọi $i \neq j$. Bởi vậy, các vector $v_1, \ldots, v_m$ là các vector trong $\mathbb{R}^n$. Mặt khác, ta có số chiều của $\mathbb{R}^n$ là $n$. Do đó, trong bước tiếp theo chúng ta chỉ cần chứng minh $v_1, \ldots, v_m$ độc lập tuyến tính trong không gian $\mathbb{R}^n$.

Giả sử phản chứng rằng tồn tại các hệ số $\alpha_1, \ldots \alpha_m$ không đồng nhất bằng không sao cho $\sum_{i=1}^m \alpha_i v_i=0$. Do đó, ta có

$\quad\quad\quad\quad\quad\quad 0 =\left|\sum_{i=1}^m \alpha_i v_i\right|^2=\left\langle\sum_{i=1}^m \alpha_i v_i, \sum_{i=1}^m \alpha_i v_i\right\rangle $

$\quad\quad\quad\quad\quad\quad\quad =\sum_{i=1}^m \alpha_i^2\left|v_i\right|^2+\sum_{1 \leq i \neq j \leq m} \alpha_i \alpha_j\left\langle v_i, v_j\right\rangle$

Mặt khác, $\left|v_i\right|^2=\left|A_i\right|$, và $\left\langle v_i, v_j\right\rangle=\left|A_i \cap A_j\right|$. Bởi vậy,

$\quad\quad\quad\quad 0=\sum_{i=1}^m \alpha_i^2\left|v_i\right|^2+\sum_{i \neq j} k \alpha_i \alpha_j=\sum_{i=1}^m \alpha_i^2\left(\left|A_i\right|-k\right)+k \sum_{1 \leq i, j \leq m} \alpha_i \alpha_j$

Ta thấy rằng $\sum_{1 \leq i, j \leq m} \alpha_i \alpha_j=\left(\sum_{1 \leq i \leq m} \alpha_i\right)^2$, nên $0=\sum_{i=1}^m \alpha_i^2\left(\left|A_i\right|-k\right)+k\left(\sum_{1 \leq i, j \leq m} \alpha_i\right)^2$. Vì $\left|A_i\right| \geq k$ và có nhiều nhất một tập với chính xác $k$ phần tử, nên $\alpha_1=\cdots=\alpha_m=0$.

Điều này mâu thuẫn với giả thiết, hay các vector $v_1, \ldots, v_m$ là độc lập tuyến tính. Như vậy ta sẽ có $m \leq n$.

Bài 8. Cho tam giác $A B C$ nội tiếp đường tròn $(O)$. Đường tròn nội tiếp $(I)$ tiếp xúc với các cạnh $B C, C A, A B$ lần lượt tại $D, E, F$. Gọi $J$ là tâm bàng tiếp góc $A$ của tam giác $A B C$ và $H$ là hình chiếu của $D$ lên $E F$.

(a) Chứng minh rằng giao điểm của $A H, J D$ thì thuộc đường thẳng $O I$.

(b) Giả sử $D H$ cắt lại $(I)$ ở $K$ và $I K$ cắt lại đường tròn ngoại tiếp $(I E F)$ ơ $L$. Chứng minh rằng $A D, L H$ cắt nhau tại một diểm nằm trên $(I E F)$.

Lời giải. (a) Ta có bổ đề sau:

Bổ ĐỀ. $O I$ là đường thẳng Euler của tam giác $D E F$.

Bổ đề này quen thuộc và có thể chứng minh bằng cách hướng như sau (chi tiết xin dành cho bạn đọc).

  1. Sử dụng phép nghịch đảo tâm $I$, phương tích $r^2$ biến $(O)$ thành đường tròn Euler của $D E F$ nên có các tâm thẳng hàng.
  2. Sử dụng phép vị tự bằng cách gọi thêm trung điểm các cung nhỏ $B C, C A, A B$ của $(O)$.

Khi đó, gọi $T$ là giao điểm của $I O$ và $H D$ thì rõ ràng $T$ là trực tâm của tam giác $D E F$. Gọi $M$ là trung điểm cung nhỏ $B C$ của $(O)$ thì dễ thấy $M$ là trung điểm $I J$.

Bằng biến đổi góc, ta có $\triangle T E F \sim \triangle I B C$, mà $T H, I D$ là hai đường cao tương ứng nên $\frac{T H}{I D}=\frac{E F}{B C}$. Mặt khác, $\triangle I E F \sim \triangle M B C$ nên

$\quad\quad\quad\quad\quad\quad\quad\quad \frac{E F}{B C}=\frac{I E}{M C}=\frac{2 I E}{2 M I}=\frac{2 I E}{I J}$

suy ra $\frac{T H}{I D}=\frac{2 I E}{I J}$. Do đó

$\quad\quad\quad\quad\quad\quad T H \cdot I J=2 I D^2=2 I N \cdot I A=T D \cdot I A$

(vì $I, T$ lần lượt là tâm ngoại tiếp và trực tâm tam giác $D E F$ ) nên $\frac{T H}{T D}=\frac{I A}{I J}$. Cuối cùng, vì $H D | A J$ (cùng vuông góc với $E F$ ) nên theo định lý Talet thì $A H, J D, T I$ đồng quy hay nói cách khác, $A H, J D$ cắt nhau trên $O I$.

(b) Giả sử $A D$ cắt lại $(I)$ tại $G$. Ta cần chứng minh rằng $G, H, L$ thẳng hàng.

Do $D K | A I$ nên

$\quad\quad\quad\quad\quad\quad\quad\quad\quad\quad\angle A G L=\angle A I L=\angle A I K=\angle D K I$

suy ra $\angle D G L=\angle D K L$. Vì thế nên $D G K L$ là tứ giác nội tiếp. Do đó, $L G$ là trục đẳng phương của $(L K D),(I E F)$. Lại có

nên suy ra $H$ thuộc trục đẳng phương của hai đường tròn này, tức là $H \in L G$. Từ đó ta có điều phải chứng minh.

Nhận xét. Liên quan đến ý b, có một bài toán khá thú vị với nội dung như sau:

Trung tuyến đỉnh $D$ của tam giác $D E F$ cắt $(I)$ ở $L$. Chứng minh rằng trục đẳng phương của $(L B F),(L C E)$ đi qua giao điểm của $J D$ và đường thẳng qua $A$, vuông góc với $A I$.

 

 

 

 

 

 

 

 

 

 

 

 

 

 

 

 

 

 

 

 

 

 

 

 

 

 

 

 

 

 

 

 

 

 

 

 

 

 

 

 

 

 

 

 

 

 

 

 

 

 

 

 

 

 

 

 

 

 

 

 

 

 

 

 

 

 

 

 

 

 

 

 

 

 

 

 

 

 

 

 

 

 

 

 

 

 

 

 

 

 

 

 

 

 

 

 

 

 

 

 

Đề thi và đáp án kì thi chọn đội tuyển thi Quốc gia trường Phổ thông Năng khiếu năm học 2014 – 2015

ĐỀ THI

 

Ngày thi thứ nhất

Bài 1. Cho $a, b, c>0$ thỏa mãn điều kiện $(a+1)(b+1)(c+1)=1+4 a b c$. Chứng mình rằng ta có bất đẳng thức $a+b+c \leq 1+a b c$.

Bài 2. Cho tập hợp $A=[n^3-4 n+15 \mid n \in \mathbb{N}]$. Tìm tất cả các phần tử $a \in A$ thỏa mãn đồng thời hai điều kiện sau đây:

$\quad (i)\quad a$ là số chẵn.

$\quad (ii)$ Nếu $a_1, a_2$ là các ước số của $\frac{a}{2}$ với $a_1, a_2>1$ thì $\operatorname{gcd}\left(a_1, a_2\right)>1$.

Bài 3. Tìm tất cả các hàm số $f: \mathbb{N}^* \rightarrow \mathbb{N}^*$ thỏa mãn:

$\quad\quad\quad\quad\quad\quad\quad\quad\quad\quad f\left(\frac{f(n)}{n}\right)=n^2 \forall n \in \mathbb{N}^*$

Bài 4. Cho tam giác $A B C$ nội tiếp $(O)$, có $B, C$ cố định và $A$ thay đổi trên $(O)$. Ký hiệu $(I)$ là đường tròn nội tiếp tam giác $A B C$. Gọi $\left(O_1\right)$ là đường tròn qua $A, B$ và tiếp xúc với đường tròn $(I)$ tại $E$. Gọi $\left(O_2\right)$ là đường tròn qua $A, C$ và tiếp xúc với đường tròn $(I)$ tại $F$. Đường phân giác trong của góc $\angle A E B$ cắt $\left(O_1\right)$ tại $M$ và đường phân giác trong của góc $\angle A F C$ cắt $\left(O_2\right)$ tại $N$.

(a) Chứng minh rằng tứ giác $E F M N$ nội tiếp.

(b) Gọi $J$ là giao điểm của $E M$ và $F N$. Chứng minh rằng đường thẳng $I J$ luôn đi qua một điểm cố định.

Ngày thi thứ hai

Bài 5. Cho dãy số $\left(x_n\right)$ bởi $x_0=1, x_1=2014$ và $x_{n+1}=\sqrt[3]{x_n x_{n-1}^2} \forall n \in \mathbb{N}^*$.

(a) Chứng minh rằng dãy số $\left(x_n\right)$ có giới hạn hữu hạn và tìm giới hạn đó.

(a) Với mỗi $n \geq 2$, hãy tìm số nguyên dương $k$ nhỏ nhất sao cho $a=x_n^k$ là một số nguyên. Chứng minh rằng khi đó $a$ không thể viết được dưới dạng tổng các lũy thừa bậc ba của hai số tự nhiên.

Bài 6. Cho $X$ là tập hợp gồm 19 phần tử.

(a) Chứng minh rằng tồn tại ít nhất 2600 tập con 7 phần tử của $X$ sao cho với hai tập con $A, B$ bất kỳ trong số 2600 tập con đó, ta có $|A \cap B| \leq 5$.

(b) Xét một họ $\Omega$ gồm $k$ tập con có 7 phần tử của $X$. Một tập $A \subset X$ được gọi là một cận trên của $\Omega$ nếu như $|A|=8$ và tồn tại một tập con $F$ của họ $\Omega$ sao cho $F \subset A$. Gọi $d$ là số tập con cận trên của họ $\Omega$. Chứng minh rằng $d \geq \frac{3}{2} k$.

Bài 7. Cho tam giác $A B C$ không cân. Gọi $I$ là trung điểm $B C$. Đường tròn $(I)$ tâm $I$ đi qua $A$ cắt $A B, A C$ lần lượt tại $M, N$. Giả sử $M I, N I$ cắt $(I)$ tại $P, Q$. Gọi $K$ là giao điểm của $P Q$ với tiếp tuyến tại $A$ của $(I)$. Chứng minh rằng $K$ thuộc đường thẳng $B C$.

Bài 8. Tìm số nguyên dương $n$ lớn nhất thỏa mãn các điều kiện sau:

$\quad (i)\quad n$ không chia hết cho 3 .

$\quad (ii)$ Bảng vuông $n \times n$ không thể được phủ kín bằng 1 quân tetramino $1 \times 4$ và các quân trimino $1 \times 3$. Trong phép phủ, các quân tetramino và trimino được phép quay dọc nhưng không được phép chườm lên nhau hoặc nằm ra ngoài bảng vuông.

LỜI GIẢI

Bài 1. Cho $a, b, c>0$ thỏa mãn điều kiện $(a+1)(b+1)(c+1)=1+4 a b c$. Chứng minh rằng ta có bất đẳng thức
$\quad\quad\quad\quad\quad\quad\quad\quad\quad\quad a+b+c \leq 1+a b c .$
Lời giải. Điều kiện đã cho viết thành $a b+b c+c a+a+b+c=3 a b c$. Chia hai vế cho $a b c$ rồi đặt $a=\frac{1}{x}, b=\frac{1}{y}, c=\frac{1}{z}$, ta có $x y+y z+z x+x+y+z=3$.
Bất đẳng thức đã cho có thể viết thành
$\quad\quad\quad\quad\quad\quad x y+y z+z x-x y z \leq 1 \text { hay } x+y+z+x y z \geq 2 \text {. }$
Theo bất đẳng thức Schur thì
$\quad\quad\quad\quad\quad\quad (x+y+z)^3+9 x y z \geq 4(x y+y z+z x)(x+y+z) .$
Đặt $m=x+y+z, n=x y+y z+z x$ thì $m+n=3$ và
$\quad\quad\quad\quad\quad\quad\quad\quad\quad\quad x y z \geq \frac{4 m n-m^3}{9} .$
Ta sẽ chứng minh rằng
$\quad\quad\quad\quad\quad\quad m+\frac{4 m n-m^3}{9} \geq 2 \Leftrightarrow m^3+4 m^2-21 m+18 \leq 0$
hay $(m-2)\left(m^2+6 m-9\right) \leq 0$. Chú ý rằng $m^2 \geq 3 n$ nên
$\quad\quad\quad\quad\quad\quad\quad\quad m^2 \geq 3(3-m) \Leftrightarrow m^3+3 m \geq 9$

Do đó $m^2+6 m-9 \geq 0$. Ta xét các trường hợp

  1. Nếu $m>2$ thì $x+y+z>2$ nên hiển nhiên bất đẳng thức cần chứng minh là đúng.
  2. Nếu $m \leq 2$ thì $m-2 \leq 0$ nên ta cũng có $(m-2)\left(m^2+6 m-9\right) \leq 0$.

Vậy trong mọi trường hợp, ta luôn có điều phải chứng minh.

Bài 2. Cho tập hợp $A=[n^3-4 n+15 \mid n \in \mathbb{N}]$. Tìm tất cả các phần tử $a \in A$ thỏa mãn đồng thời hai điều kiện sau đây:

$\quad (i)\quad a$ là số chẵn.

$\quad (ii)$ Nếu $a_1, a_2$ là các ước số của $\frac{a}{2}$ với $a_1, a_2>1$ thì $\operatorname{gcd}\left(a_1, a_2\right)>1$.

Lời giải. Ta thấy rằng $a=n^3-4 n+15$ chẵn nên $n^3+15$ chẵn hay $n$ lẻ. Đặt $n=2 k+1$ với $k \in \mathbb{N}$. Ta có

$\quad\quad\quad\quad\quad\quad a=n^3-4 n+15 =(n+3)\left(n^3-3 n+15\right) $

$\quad\quad\quad\quad\quad\quad\quad\quad\quad\quad\quad\quad\quad\quad =(2 k+4)\left(4 k^2-2 k+3\right)$

nên $\frac{a}{2}=(k+2)\left(4 k^2-2 k+3\right)$. Điều kiện ii) cho thấy rằng $\frac{a}{2}$ phải là lũy thừa của một số nguyên tố, vì nếu nó có hai ước nguyên tố trở lên, đặt là $p, q$ thì chọn $x=p, y=q$, ta có $x, y>1$ nhưng $\operatorname{gcd}(x, y)=1$, không thỏa mãn.

Vì $\left(4 k^2-2 k+3\right)-(k+2)=4 k^2-3 k+1>0$ với mọi $k \in \mathbb{N}$. Do đó, ta phải có $k+2 \mid 4 k^2-2 k+3$. Suy ra

$\quad\quad\quad\quad\quad\quad\quad\quad\quad\quad\frac{4 k^2-2 k+3}{k+2}=4 k-10+\frac{23}{k+2} \in \mathbb{Z} .$

Do đó $k+2 \in{1,23}$ vì $k+2>0$. Ta xét các trường hợp

  1. Nếu $k+2=1$ thì $k=-1$ hay $n=2 k+1=-1<0$, không thỏa mãn.
  2. Nếu $k+2=23$ thì $k=21$ hay $n=43$, tính được $\frac{a}{2}=3 \cdot 5^2 \cdot 23^2$, cũng không thỏa mãn.

Vậy không tồn tại số $a$ nào thỏa mãn.

Bài 3. Tìm tất cả các hàm số $f: \mathbb{N}^* \rightarrow \mathbb{N}^*$ thỏa mãn:

$\quad\quad\quad\quad\quad\quad\quad\quad\quad\quad f\left(\frac{f(n)}{n}\right)=n^2 \forall n \in \mathbb{N}^* .$

Lời giải. Với $n \in \mathbb{N} *$, ta thấy rằng nếu $n=1$ thì $f(f(1))=1$.

Nếu $n>1$ thì gọi $p$ là một ước nguyên tố bất kỳ của $n$.

Vì $\frac{f(n)}{n} \in \mathbb{N} *$ nên $n \mid f(n)$. Đặt $a=v_p(n), b=v_p(f(n))$ thì trước hết, ta có $a \leq b$.

Từ $f\left(\frac{f(n)}{n}\right)=n^2$, ta suy ra rằng $\frac{f(n)}{n} \mid n^2$ hay $f(n) \mid n^3$, tức là $b \leq 3 a$.

Trong biểu thức đã cho, thay $n \rightarrow \frac{f(n)}{n}$ thì

$\quad\quad\quad\quad\quad\quad f\left(\frac{f\left(\frac{f(n)}{n}\right)}{\frac{f(n)}{n}}\right)=\left(\frac{f(n)}{n}\right)^2 \Leftrightarrow f\left(\frac{n^3}{f(n)}\right)=\left(\frac{f(n)}{n}\right)^2$

Do đó, ta phải có

$\quad\quad\quad\quad\quad\quad\quad\quad\quad\left(\frac{f(n)}{n}\right)^2\left|\frac{n^3}{f(n)} \Leftrightarrow f^3(n)\right| n^5 \text { nên } 3 b \leq 5 a \text {. }$

Sau đó lại tiếp tục thay $n$ trong biểu thức đã cho bởi $\frac{n^5}{f^3(n)}$ và cứ như thế, ta xây dựng được hai dãy hệ số của $a, b$ như sau

$\quad\quad\quad\quad\quad\quad\quad\quad\quad\quad u_0=v_0=1, u_1=3, v_1=1 \text { và } $

$\quad\quad\quad\quad\quad\quad\quad\quad u_{k+1}=2 u_{k-1}+v_k, v_{k+1}=2 v_{k-1}+u_k \text { với } k \geq 1 .$

Khi đó

$\quad\quad\quad\quad\quad\quad\quad\quad\quad\quad\quad\quad\frac{v_{2 k}}{u_{2 k}} \leq \frac{b}{a} \leq \frac{u_{2 k+1}}{v_{2 k+1}} .$

Biến đổi công thức của hai dãy, ta có $u_{n+2}=5 u_n-4 u_{n-2}, v_{n+2}=5 v_n-4 v_{n-2}$ và cả hai dãy đều có phương trình đạạc trưng là $t^2-5 t+4=0$. Ngoài ra, dãy chăn và dãy lẻ trong mỗi dãy đều độc lập với nhau.

Ta có $u_0=1, u_2=3, v_0=1, v_2=5$ nên

$\quad\quad\quad\quad\quad\quad\quad\quad u_{2 k}=\frac{13+2 \cdot 16^k}{15}, v_{2 k}=\frac{11+4 \cdot 16^k}{15}, k \geq 1 .$

Từ đó, dễ dàng tính được $\lim \frac{u_{2 k+1}}{v_{2 k+1}}=2$.

Một cách tương tự, ta tính được $\lim \frac{u_{2 k}}{v_{2 k}}=\frac{1}{2}$. Do đó, số $\frac{b}{a}$ bị kẹp ở giữa và là số nguyên nên chỉ có thể là $\frac{b}{a}=2 \Leftrightarrow b=2 a$.

Rõ ràng tập hợp ước nguyên tố của $n$ và $f(n)$ là giống nhau. Hơn nữa, với một ước nguyên tố cụ thể thì số mũ trong $f(n)$ gấp đôi số mũ trong $n$. Suy ra $f(n)=$ $n^2, \forall n>1$.

Tiếp theo, giả sử $f(1)=n>1$ thì ta có $f(f(1))=1$ nên $f(n)=1$, mâu thuẫn. Vì thế nên chỉ có thể $f(1)=1$.

Vậy tất cả các hàm thỏa mãn là $f(n)=n^2, \forall n \in \mathbb{N}^*$.

Bài 4. Cho tam giác $A B C$ nội tiếp $(O)$, có $B, C$ cố định và $A$ thay đổi trên $(O)$. Ký hiệu $(I)$ là đường tròn nội tiếp tam giác $A B C$. Gọi $\left(O_1\right)$ là đường tròn qua $A, B$ và tiếp xúc với đường tròn $(I)$ tại $E$. Gọi $\left(O_2\right)$ là đường tròn qua $A, C$ và tiếp xúc với đường tròn $(I)$ tại $F$. Đường phân giác trong của góc $\angle A E B$ cắt $\left(O_1\right)$ tại $M$ và đường phân giác trong của góc $\angle A F C$ cắt $\left(O_2\right)$ tại $N$.

(a) Chứng minh rằng tứ giác $E F M N$ nội tiếp.

(b) Gọi $J$ là giao điểm của $E M$ và $F N$. Chứng minh rằng đường thẳng $I J$ luôn đi qua một điểm cố định.

Lời giải. (a) Trước hết, ta thấy rằng $O_1, I, E$ thẳng hàng và $O_2, I, F$ thẳng hàng. Vì $M$ là trung điểm cung $A B$ của $\left(O_1\right)$ nên $O_1 M$ là trung trực của $A B$, suy ra $O \in O_1 M$. Tương tự, ta cũng có $O \in O_1 N$.

Gọi $P, Q$ lần lượt là tiếp điểm của $(I)$ với $A B, A C$.

Vì $I P | O_1 M$ (cùng vuông góc với $A B$ ) nên $\angle M O_1 E=\angle P I E$. Hơn nữa, các tam giác $O_1 M E, I P E$ đều cân với đỉnh là $O_1, I$ nên suy ra chúng đồng dạng, tức là $\angle I E P=\angle O_1 E M$ hay $E, P, M$ thẳng hàng. Tương tự thì $F, Q, N$ cũng thẳng hàng.

Vì ta đã có $E, F, P, Q$ cùng thuộc đường tròn $(I)$ nên để có $E, F, M, N$ cùng thuộc một đường tròn thì $\angle E M N=\angle E F N=\angle E P Q$ hay $M N | P Q$.

Mặt khác, $A I \perp P Q$ nên ta cần có $A I \perp M N$.

Thật vậy, sử dụng phương tích với đường tròn $(I)$ ta có

$\quad\quad\quad\quad\quad\quad M A^2-N A^2=M P \cdot M E-N Q \cdot N F=M I^2-N I^2$

nên theo định lý bốn điểm thì $A I \perp M N$, từ đó ta có điều phải chứng minh.

(b) Vì $P Q|M N, O M| I P$ nên dễ dàng có $\angle I P Q=\angle O M N$. Tương tự $\angle I P Q=$ $\angle O N M$.

Do đó, hai tam giác $I P Q, O M N$ đồng dạng với nhau, tức là

$\quad\quad\quad\quad\quad\quad\quad\quad\quad\quad\quad\quad\frac{I P}{O M}=\frac{P Q}{M N} \text {. }$

Ngoài ra,

$\quad\quad\quad\quad\quad\quad\quad\quad\quad\quad\quad\quad\frac{J P}{J M}=\frac{I P}{O M}$

kết hợp với $\angle J P I=\angle J M O$, ta có hai tam giác $J P I, J M O$ đồng dạng, dẫn đến

$\quad\quad\quad\quad\quad\quad\quad\quad\quad\quad\quad\quad\angle P J I=\angle M J O \text {. }$

Từ đây suy ra $I, J, O$ thẳng hàng hay $I J$ luôn đi qua điểm $O$ cố định.

Ngày thi thứ hai

Bài 5. Cho dãy số $\left(x_n\right)$ bởi $x_0=1, x_1=2014$ và $x_{n+1}=\sqrt[3]{x_n x_{n-1}^2} \forall n \in \mathbb{N}^*$.

(a) Chứng minh rằng dãy số $\left(x_n\right)$ có giới hạn hữu hạn và tìm giới hạn đó.

(b) Với mỗi $n \geq 2$, hãy tìm số nguyên dương $k$ nhỏ nhất sao cho $a=x_n^k$ là một số nguyên. Chứng minh rằng khi đó $a$ không thể viết được dưới dạng tổng các lũy thừa bậc ba của hai số tự nhiên.

Lời giải. (a) Đặt $u_n=\log _{2014}\left(x_n\right)$ thì ta thu được dãy $\left(u_n\right)$ như sau

$\quad\quad\quad\quad\quad\quad\quad\quad\quad\quad\left\{\begin{array}{c}u_0=0, u_1=1 \\ u_{n+1}=\frac{1}{3} u_n+\frac{2}{3} u_{n-1}\end{array}\right.$

Từ đó tìm được

$\quad\quad\quad\quad\quad\quad\quad\quad\quad\quad u_n=\frac{3}{5}-\frac{3}{5} \cdot\left(\frac{-2}{3}\right)^n$

Suy ra $\lim _{n \rightarrow+\infty} u_n=\frac{3}{5}$ nên ta có được

$\quad\quad\quad\quad\quad\quad\quad\quad \lim _{n \rightarrow+\infty} x_n=\lim _{n \rightarrow+\infty}\left(2014^{u_n}\right)=2014^{3 / 5}$

(b) Ta thấy rằng để có $\left(x_n\right)^k$ là một số nguyên thì $\frac{3 k\left(3^n-(-2)^n\right)}{5 \cdot 3^n} \in \mathbb{Z}$ nguyên. Ta xét các trường hợp

  1. Nếu $n$ lẻ thì $3^n-(-2)^n=3^n+2^n: 5$. Vì $\operatorname{gcd}\left(\frac{3^n+2^n}{5}, 3^n\right)=1$ nên ta được $3^n \mid 3 k$ nên $k$ nhỏ nhất thỏa mãn điều này là $k=3^{n-1}$.
  2. Nếu $n$ chẵn thì $3^n-2^n \equiv(-2)^n-2^n=0(\bmod 5)$ và tương tự, ta cũng tìm được $k=3^{n-1}$.

Do đó số $k$ nhỏ nhất cần tìm là $k=3^{n-1}$. Tiếp theo, ta sẽ chứng minh rằng phương trình sau không có nghiệm tự nhiên

$\quad\quad\quad\quad\quad\quad\quad a^3+b^3=2014^n \Leftrightarrow(a+b)\left(a^2-a b+b^2\right)=2014^n$

Gọi $n_0$ số nguyên dương nhỏ nhất sao cho tồn tại $a, b \in \mathbb{Z}^{+}$để $a^3+b^3=2014^{n_0}$. Dễ thấy $n_0=1$ không thỏa mãn nên ta chỉ xét $n_0 \geq 2$. Ta xét các trường hợp

  1. Nếu $\operatorname{gcd}\left(a+b, a^2-a b+b^2\right)=1$ thì dễ thấy $(a-b)^2 \geq 1$. Khi đó

$\quad\quad\quad\quad\quad\quad\quad\quad\quad a^2-a b+b^2 \geq a+b>\sqrt{a^2-a b+b^2} .$

Vì $2014=2 \cdot 19 \cdot 53$ nên chỉ có thể xảy ra

$\quad\quad\quad\quad\quad\quad\quad\quad\quad\quad a+b=19^{n_0}, a^2-a b+b^2=106^{n_0} .$

Ngoài ra $(a+b)^2 \leq 4\left(a^2-a b+b^2\right)$ nên ta phải có $361^{n_0} \leq 4 \cdot 106^{n_0}$. Đánh giá này sai khi $n_0 \geq 2$ nên trường hợp này không thỏa mãn.

$2$. Nếu $\operatorname{gcd}\left(a+b, a^2-a b+b^2\right)>1$ thì chẳng hạn $a+b=2^x u, a^2-a b+b^2=2^y v$ với $\quad\quad\quad\quad\quad\quad\quad\quad\operatorname{gcd}(u, 2)=\operatorname{gcd}(v, 2)=1 .$

Các trường hợp còn lại chứng minh tương tự. Ngoài ra

$\quad\quad\quad\quad\quad\quad\quad\quad\quad\quad u v=1007^{n_0}, x+y=n_0 .$

Chú ý rằng $(a+b)^2-\left(a^2-a b+b^2\right)=3 a b$ nên $3 a b$ cũng chẵn, tức là cả hai số $a, b$ đều chẵn (vì nếu không thì $a^3+b^3$ lẻ).

Từ đây dễ dàng chứng minh được $3 v_2(a)=3 v_2(b)=n_0$, ta đưa về $x^{\prime 3}+y^{\prime 3}=$ $1007^{n_0}$. Cứ như thế, ta được 2014 $|a, 2014| b$ nên phương trình sau cũng có nghiệm nguyên dương

$\quad\quad\quad\quad\quad\quad\quad\quad\quad\quad \left(\frac{a}{2014}\right)^3+\left(\frac{b}{2014}\right)^3=2014^{n_0-3} .$

Điều này mâu thuẫn với các chọn $n_0$ nên phương trình trên vô nghiệm. Các trường hợp còn lại tương tự.

Ta có điều phải chứng minh.

Bài 6. Cho $X$ là tập hợp gồm 19 phần tử.

(a) Chứng minh rằng tồn tại ít nhất 2600 tập con 7 phần tử của $X$ sao cho với hai tập con $A, B$ bất kỳ trong số 2600 tập con đó, ta có $|A \cap B| \leq 5$.

(b) Xét một họ $\Omega$ gồm $k$ tập con có 7 phần tử của $X$. Một tập $A \subset X$ được gọi là một cận trên của $\Omega$ nếu như $|A|=8$ và tồn tại một tập con $F$ của họ $\Omega$ sao cho $F \subset A$. Gọi $d$ là số tập con cận trên của họ $\Omega$. Chứng minh rằng $d \geq \frac{3}{2} k$.

Lời giải. (a) Không mất tính tổng quát, ta có thể giả sử $X$ là tập hợp 19 số nguyên dương đầu tiên. Gọi $X(k)$ là tập hợp tất cả các tập con có 7 phần tử của $X$ và tổng các phần tử của nó chia 19 dư $k$.

Khi đó, dễ thấy rằng $|X(0)|+|X(1)|+\cdots+|X(18)|$ chính là số tập con có 7 phần tử tùy ý của $X$ và là $C_{19}^7$.

Ta thấy rằng hai tập hợp $A, B \in X(k)$ tùy ý đều thỏa mãn đề bài.

Thật vậy,

Giả sử $|A \cap B|=6$ (không thể có $|A \cap B|=7$ vì khi đó hai tập hợp trùng nhau).

Đặt $A=[a_1, a_2, a_3, a_4, a_5, a_6, x]$, $B=[a_1, a_2, a_3, a_4, a_5, a_6, y]$ thì

$\quad\quad\quad\quad\quad\quad\quad\quad\quad\quad \sum_{i=1}^6 a_i+x \equiv \sum_{i=1}^6 a_i+y \equiv k \quad(\bmod 19)$

nên $x \equiv y(\bmod 19)$. Suy ra $x=y$, mâu thuẫn. Đến đây, dễ thấy rằng

Ta có điều phải chứng minh.

(b) Xét một tập hợp $F$ thuộc họ $\Omega$. Vì $|X \backslash F|=19-7=12$ nên có tất cả 12 tập hợp $A \subset X$ với $|A|=8$ và $F \subset A$.

Ngược lại, ứng với một tập hợp $A$ là một cận trên của họ $\Omega$, có không quá 8 tập $F$ trong họ $\Omega$ sao cho $F \subset A$. Do đó $d \geq \frac{12}{8} k$ hay $d \geq \frac{3}{2} k$.

Đẳng thức xảy ra khi họ $\Omega$ là tập hợp tất cả các tập con có 7 phần tử của $X$.

Bài 7. Cho tam giác $A B C$ không cân. Gọi $I$ là trung điểm $B C$. Đường tròn (I) tâm $I$ đi qua $A$ cắt $A B, A C$ lần lượt tại $M, N$. Giả sử $M I, N I$ cắt $(I)$ tại $P, Q$. Gọi $K$ là giao điểm của $P Q$ với tiếp tuyến tại $A$ của $(I)$. Chứng minh rằng $K$ thuộc đường thẳng $B C$.

Lời giải. Không mất tính tổng quát, giả sử $A B<A C$.

Kẻ đường kính $A J$ của đường tròn $(I)$. Khi đó, dễ thấy tứ giác $A B J C$ và $A N J Q$ là các hình bình hành nên $J B|A C, J Q| A N$ dẫn đến $J, Q, B$ thẳng hàng. Tương tự $J, P, C$ thẳng hàng.

Gọi $H$ là hình chiếu của $A$ lên $B C$ thì tứ giác $A Q B H$ nội tiếp.

Suy ra

$\quad\quad\quad\quad\quad\quad \angle Q H B=\angle Q A B=\angle Q A M=\angle Q P M=\angle Q P I$

nên tứ giác $P Q H I$ cũng nội tiếp. Gọi $(O)$ là đường tròn ngoại tiếp tam giác $A B C$ thì dễ thấy đường tròn $(A H I)$ tiếp xúc với $(O)$ tại $A$.

Xét ba đường tròn $(O),(A H I),(P Q H I)$ thì

  • Trục đẳng phương của $(O),(A H I)$ là tiếp tuyến của $(O)$ tại $A$.
  • Trục đẳng phương của $(O),(P Q H I)$ là $P Q$.
  • Trục đẳng phương của $(P Q H I),(A H I)$ là $H I$.

Do đó, $K$ chính là tâm đẳng phương của ba đường tròn nên $K \in H I$ hay $K, B, C$ thẳng hàng.

Bài 8. Tìm số nguyên dương $n$ lớn nhất thỏa mãn các điều kiện sau:

$\quad (i)$ n không chia hết cho 3 .

$\quad (ii)$ Bảng vuông $n \times n$ không thể được phủ kín bằng 1 quân tetramino $1 \times 4$ và các quân trimino $1 \times 3$. Trong phép phủ, các quân tetramino và trimino được phép quay dọc nhưng không được phép chườm lên nhau hoặc nằm ra ngoài bảng vuông.

Lời giải. Ta sẽ chứng minh $n=5$ là giá trị lớn nhất cần tìm.

Ta nhận thấy rằng nếu $n=3 k+1, k \geq 1$ thì ta luôn phủ được bảng vuông $n \times n$ bằng cách phủ hàng đầu tiên bằng 1 quân tetramino kích thước $1 \times 4$ (ta sẽ gọi tắt là tetramino) và $k-1$ quân trimino kích thước $1 \times 3$ (ta sẽ gọi tắt là trimino). Các cột còn lại có chiều dài $3 k$ có thể phủ được bằng các quân trimino (xoay dọc lại).

Ta chứng minh rằng nếu $n=3 k+2, k \geq 2$ thì bảng vuông $n \times n$ cũng phủ được. Cách phủ với $n=8$ được minh họa như sau

Dễ dàng thấy rằng với $k \geq 3$ thì ta có thể thu được cách phủ cho bảng vuông $n \times n$ bằng cách phủ phần hình vuông $8 \times 8$ ở góc trên bên trái như trên, phần còn lại gồm 1 hình chữ nhật kích thước $3(k-2) \times(3 k+2)$ và 1 hình chữ nhật kích thước $8 \times 3(k-2)$ phủ được bằng các quân trimino.

Bây giờ ta chứng minh bảng vuông $5 \times 5$ không thể phủ được bằng 1 quân tetramino và 7 quân trimino.

Trước hết ta chứng minh bổ đề: Nếu bảng vuông $5 \times 5$ có thể phủ được bằng một hình vuông $1 \times 1$, ta gọi là unomino và 8 quân trimino thì quân unomino $1 \times 1$ phải phủ ô trung tâm.

Thật vậy,

Ta đánh số các ô của bảng vuông $5 \times 5$ như hình vẽ

Ta thấy rằng một quân trimino luôn phủ đúng một ô mang số 1 , một ô mang số 2 và một ô mang số 3 . Vì số các số 2 bằng 9 , còn số các số 1 và 3 bằng 8 nên nếu phép phủ ở đề bài thực hiện được thì quân unomino phải phủ một ô mang số 2 .

Mặt khác, ta có thể đánh số bảng vuông $5 \times 5$ bằng một cách khác

Các tính chất nói ở trên vẫn đúng cho cách đánh số này, tuy nhiên ở đây số số 1 là 9 , còn số số 2 và 3 là 8 . Do đó, một lần nữa ta kết luận quân unomino phải phủ một ô mang số 1 .

Giao hai điều kiện cần nói trên lại, ta thấy với một cách phủ hợp lệ thì quân unomino phải phủ ô trung tâm.

Quay trở lại với vấn đề phủ bảng vuông $5 \times 5$ bằng 1 quân tetramino và 7 quân trimino. Nếu tồn tại một cách phủ như thế thì cắt quân tetramino thành 1 quân unomino và 1 quân trimino, ta thu được một phép phủ bảng vuông $5 \times 5$ bằng 1 quân unomino và 8 quân trimino.

Theo bổ đề thì quân unomino phải nằm ở ô trung tâm, nghĩa là một đầu của quân tetramino phải nằm ở ô trung tâm, mâu thuẫn (vì khi đó quân tetramino sẽ bị lòi ra ngoài bảng vuông).

Với những lý luận ở trên, ta kết luận $n=5$ là giá trị lớn nhất cần tìm.

 

 

 

 

 

 

 

 

 

 

 

 

 

 

 

 

 

 

 

 

 

 

 

 

 

 

 

 

 

 

 

 

 

 

 

 

 

 

 

 

 

 

 

 

 

 

 

 

 

 

 

 

 

 

 

 

 

 

 

 

 

 

 

 

 

 

 

 

 

 

 

 

 

 

 

 

 

 

 

 

 

 

 

 

 

Đề thi và đáp án kì thi chọn đội tuyển thi Quốc gia trường Phổ thông Năng khiếu năm học 2013 – 2014

ĐỀ THI

Ngày thi thứ nhất

Bài 1. Tìm tất cả các hàm số $f: \mathbb{R} \rightarrow \mathbb{R}$ thoả mãn:

$\quad\quad\quad\quad\quad\quad\quad\quad\quad\quad f\left(x^3+y+f(y)\right)=2 y+x^2 f(x) \forall x, y \in \mathbb{R}$

Bài 2. Cho dãy số $\left(u_n\right)$ thoả mãn $u_1=2013, u_{n+1}=u_n^3-4 u_n^2+5 u_n \forall n \in \mathbb{N}^*$. Tìm tất cả các số nguyên tố $p$ là ước của $u_{2014}+2009$ và $p \equiv 3(\bmod 4)$.

Bài 3. Trong một hội nghị khoa học có 5000 đại biểu tham dự, trong đó có một nhóm gồm 280 người là ban tổ chức. Giả sử rằng mỗi thành viên của ban tổ chức thì quen tất cả thành viên khác của hội nghị. Một uỷ ban gồm một số đại biểu được gọi là uỷ ban làm việc nếu tất cả các thành viên trong đó đều quen nhau, và được gọi là uỷ ban thách thức nếu không có hai thành viên nào của uỷ ban quen nhau. Chứng minh rằng ta có thể phân hoạch các đại biểu (kể cả ban tổ chức) thành các ủy ban mà mỗi ủy ban gồm 5 đại biểu, sao cho các uỷ ban này hoặc là làm việc, hoặc là thách thức.

Bài 4. Cho tam giác $A B C$ có $B, C$ cố định còn $A$ di động sao cho $A B=A C$ và $\angle B A C>60^{\circ}$. Đường thẳng đối xứng với $B C$ qua $A B$ cắt $A C$ tại $P$. Trên đoạn $P C$ lấy điểm $M$ sao cho $P M=P B$. Gọi $N$ là giao điểm của $A B$ và phân giác ngoài của góc $\angle B C A$. Chứng minh rằng $M N$ luôn đi qua một điểm cố định.

Ngày thi thứ hai

Bài 5. Cho 2014 số thực $x_1, x_2, \ldots, x_{2014}$ thoả mãn các điều kiện:

(i) $x_1+x_2+\cdots+x_{2014}=0$,

(ii) $x_1^2+x_2^2+\cdots+x_{2014}^2=2014$.

Tìm giá trị lớn nhất của biểu thức $P=x_1 x_2 \cdots x_{2014}$.

Bài 6. Cho dãy số $\left(u_n\right)$ xác định bởi $u_1=1, u_{n+1}=\frac{u_n}{\sqrt{u_n^2+1}+\sqrt{2}}, \forall n \in \mathbb{N}^*$. Tính $\lim \frac{u_{n+1}}{u_n}$

Bài 7. Cho $n$ là số nguyên dương và $A$ là tập con khác rỗng của $X={1,2, \ldots, n}$.

(a) Tính giá trị của tổng $S(A)=\sum_{E \subset X}(-1)^{|E \cap A|}$ trong đó $E$ lấy trên tất cả các tập hợp con của $X$, kể cả tập hợp rỗng.

(b) Cho $m \in \mathbb{N}^*$. Xét $m$ tập con khác rỗng của $X$ là $A_1, A_2, \ldots, A_m$ và $m$ số nguyên khác 0 là $a_1, a_2, \ldots, a_m$ sao cho $a_1+a_2+\cdots+a_m<0$. Chứng minh rằng tồn tại $E \subset X$ sao cho $\sum_{i=1}^m(-1)^{\left|E \cap A_i\right|} a_i>0$.

Bài 8. Cho tam giác $A B C$ nhọn có $H$ là trực tâm. $P$ là điểm di động bên trong tam giác $A B C$ sao cho $\angle B P C=\angle B H C$. Đường thẳng qua $B$ vuông góc $A B$ cắt $P C$ tại $M$, đường thẳng qua $C$ vuông góc $A C$ cắt $P B$ tại $N$. Gọi $I$ là trung điểm của $M N$. Chứng minh rằng $I$ luôn thuộc một đường thẳng cố định.

 

LỜI GIẢI

Ngày thi thứ nhất

Bài 1 . Tìm tất cả các hàm số $f: \mathbb{R} \rightarrow \mathbb{R}$ thoả mãn:

$\quad\quad\quad\quad\quad\quad f\left(x^3+y+f(y)\right)=2 y+x^2 f(x), \forall x, y \in \mathbb{R} .$

Lời giải. Trong phương trình đã cho, thay $x=y=0$, ta có $f(f(0))=0$.

Lại thay $y=0$ thì

$\quad\quad\quad\quad\quad\quad\quad\quad\quad\quad f\left(f^3+f(0)\right)=x^2 f(x), \forall x .$

Thay $y=f(0)$ thì

$\quad\quad\quad\quad\quad\quad\quad\quad\quad\quad f\left(x^3+f(0)\right)=2 f(0)+x^2 f(x) .$

Từ đây suy ra $f(0)=0$. Thay $y=0$ vào đẳng thức đã cho ta được $f\left(x^3\right)=x^2 f(x)$. Do đó ta có

$\quad\quad\quad f\left(x^3+y+f(y)\right)=2 y+f\left(x^3\right) \text { hay } f(x+y+f(y))=2 y+f(x)\quad\quad\quad(*).$

Thay $y$ bởi $-y$, ta được

$\quad\quad\quad\quad\quad\quad\quad\quad\quad\quad f(x-y+f(-y))=-2 y+f(x) .$

Với $x$ bất kì, ta lấy $2 y=f(x)$ ta được $f(x-y+f(-y))=0$ suy ra $x-y+f(-y)=0$. Do đó, ta được $f(-x)=f(-y+f(-y))=-2 y=-f(x)$. Từ đây suy ra

$\quad\quad\quad\quad\quad\quad\quad\quad\quad\quad f(x+f(y)+f(f(y)))=2 f(y)+f(x)$

$\quad\quad\quad\quad\quad\quad\quad\quad\quad\quad f(x+f(y)+f(f(y)))=2 f(y)+f(x) .$

Trong $(*)$ thay $x=-y$ ta được $f(f(y))=2 y+f(-y)=2 y-f(y)$, kết hợp với đẳng thức trên, ta được

$f(x+2 y)=2 f(y)+f(x) .$

Đến đây cho $x=0$ ta được $f(2 y)=2 f(y)$ nên ta được $f(x+y)=f(x)+f(y)$, tức là $f(x)$ cộng tính. Đến đây ta sẽ tính $f\left((x+1)^3+(x-1)^3\right)$ theo hai cách như sau

  • $f\left((x+1)^3+(x-1)^3\right)=f\left(2 x^3+6 x\right)=2 x^2 f(x)+6 f(x)$.
  • $f\left((x+1)^3+(x-1)^3\right)=(x+1)^2 f(x+1)+(x-1)^2 f(x-1)=(x+1)^2(f(x)+$ $f(1))+(x-1)^2(f(x)-f(1))=2 x^2 f(x)+2 f(x)+4 x f(1)$.

So sánh hai đẳng thức trên, ta được $f(x)=x f(1)=a x$ với mọi $x$. Thử lại ta được $a=1, a=-2$.

Vậy các hàm cần tìm là $f(x)=x, f(x)=-2 x$.

Bài 2. Cho dãy số $\left(u_n\right)$ thoả mãn $u_1=2013, u_{n+1}=u_n^3-4 u_n^2+5 u_n \forall n \in \mathbb{N}^*$.

Tìm tất cả các số nguyên tố $p$ là ước của $u_{2014}+2009$ và $p \equiv 3(\bmod 4)$.

Lời giải. Ta có

$\quad\quad\quad\quad\quad\quad\quad\quad u_{n+1}-2 =\left(u_n-2\right)\left(u_{n-1}-1\right)^2 $

$\quad\quad\quad\quad\quad\quad\quad\quad\quad\quad\quad\quad =\left(u_{n-2}-1\right)^2\left(u_{n-1}-1\right)^2\left(u_{n-2}-2\right) $

$\quad\quad\quad\quad\quad\quad\quad\quad\quad\quad\quad\quad =\left(u_{n-1}-1\right)^2\left(u_{n-2}-1\right)^2 \cdots\left(u_2-1\right)^2\left(u_1-2\right) .$

Do đó

$\quad\quad\quad\quad\quad u_{2014}+2009=2011\left[\left(u_{2013}-1\right)^2\left(u_{2012}-1\right)^2 \cdots\left(u_2-1\right)^2+1\right] .$

Gọi $B$ là biểu thức trong dấu ngoặc vuông thứ hai. Ta có bổ đề quen thuộc là nếu $a^2+b^2$ chia hết cho số nguyên tố $p=4 k+3$ thì $a, b$ cùng chia hết cho $p$. Từ đây suy ra số $B$ có dạng $a^2+1$ nên nó sẽ không có ước nguyên tố dạng $4 k+3$.

Vậy $u_{2014}+9$ chỉ có một ước nguyên tố $p \equiv 3(\bmod 4)$ duy nhất là 2011 .

Bài 3. Trong một hội nghị khoa học có 5000 đại biểu tham dự, trong đó có một nhóm gồm 280 người là ban tổ chức. Giả sử rằng mỗi thành viên của ban tổ chức thì quen tất cả thành viên khác của hội nghị. Một uỷ ban gồm một số đại biểu được gọi là uỷ ban làm việc nếu tất cả các thành viên trong đó đều quen nhau, và được gọi là uỷ ban thách thức nếu không có hai thành viên nào của uỷ ban quen nhau. Chứng minh rằng ta có thể phân hoạch các đại biểu (kể cả ban tổ chức) thành các ủy ban mà mỗi ủy ban gồm 5 đại biểu, sao cho các uỷ ban này hoặc là làm việc, hoặc là thách thức.

Lời giải. Trước hết, ta chứng minh bổ đề sau

ĐỊNH LÝ RAMSEY Với $s, t$ là các số nguyên dương, gọi $R(s, t)$ là số đỉnh ít nhất cần có của một graph để trong đó luôn tồn tại một tập độc lập $s$ đỉnh hoặc một graph con đầy đủ $t$ đỉnh. Khi đó

$\quad\quad\quad\quad\quad\quad\quad\quad\quad\quad\quad\quad R(s, t) \leq C_{s+t-2}^{s-1}\quad\quad\quad(*) .$

Chứng minh. Ta sẽ chứng minh rằng

$\quad\quad\quad\quad\quad\quad\quad\quad R(s, t) \leq R(s-1, t)+R(s, t-1) .$

Để ý rằng với $s=1$ hoặc $t=1$ thì $R(s, t)=1$. Do đó, nếu chứng minh được đánh giá này thì chỉ cần dùng tính chất của tam giác Pascal để có

$\quad\quad\quad\quad\quad\quad\quad\quad R(s, t) \leq C_{s+t-3}^{s-2}+C_{s+t-3}^{s-1}=C_{s+t-2}^{s-1} .$

Đặt $n$ là vế phải của $(*)$ và xét graph $G$ có $n$ đỉnh. Xét $v \in G$ thì

  1. Nếu như có ít nhất $R(s, t-1)$ đỉnh kề với $v$. Khi đó, theo định nghĩa thì trong tập đỉnh đó, sẽ luôn có một tập độc lập $s$ đỉnh hoặc một graph con đầy đủ $t-1$ đỉnh, ghép thêm đỉnh $v$ vào thì thỏa mãn điều kiện của $R(s, t)$.
  2. Nếu như có ít nhất $R(s-1, t)$ đỉnh không kề với $v$. Tương tự trên, trong tập đỉnh đó, cũng sẽ có một một graph con đầy đủ $t$ đỉnh hoặc tập độc lập $s-1$ đỉnh, ghép thêm đỉnh $v$ vào thì thỏa mãn điều kiện của $R(s, t)$.

Từ đó, ta thấy graph $G$ này thỏa mãn điều kiện của $R(s, t)$ nên theo tính nhỏ nhất thì $R(s, t) \leq n$.

Trở lại bài toán,

Xét graph đơn vô hướng $G=(V, E)$ đại diện cho hội nghị khoa học đã nêu, trong đó $V$ là tập hợp các đại biểu và hai đỉnh được nối nhau nếu hai đại biểu tương ứng quen nhau. Ta gọi $T$ là tập hợp đỉnh biểu diễn cho thành viên của ban tổ chức.

Khi đó một ủy ban gồm 5 thành viên là đại diện nếu như đó là một graph đầy đủ, còn đó là thách thức nếu đó là graph không có cạnh. Ta gọi các graph con như thế là graph con “chuẩn”.

Trong các đỉnh $V \backslash T$, ta xóa dần dần các graph con chuẩn đến khi không thực hiện được nữa. Ta gọi tập hợp còn lại là $S$. Ta sẽ chứng minh rằng $S \cup T$ có thể phân hoạch thành các graph con chuẩn như trên.

Theo định lý Ramsey, rõ ràng $|S| \leq C_8^4=70$. Xét một đỉnh $v \in S$ thì giả thiết, $v$ kề với cả 280 đỉnh của $T$ nên ta chọn ra trong đó 4 đỉnh để ghép với $v$ tạo thành một graph con “chuẩn”. Cứ như thế thực hiện cho đến hết các phần tử của $S$, còn lại bao nhiêu phần tử trong $T$ thì chia đều ra thành các graph con “chuẩn” là được.

Bài toán được giải quyết.

Nhận xét. Bài toán gốc của đề thi bị sai và có thể chỉ ra phản ví dụ. Nội dung cụ thể của nó như sau: Trong một hội nghi khoa học có 5000 đại biểu tham dư, mỗi một đại biểu biết ít nhất một thứ tiếng. Một uỷ ban gồm một số đại biểu được gọi là uỷ ban làm việc nếu tất cả các thành viên uỷ ban đều biết chung một thứ tiếng, và được gọi là uỷ ban thách thức nếu không có hai thành viên nào của uỷ ban biết chung một thứ tiếng. Uỷ ban có thể gồm 1 thành viên, khi đó gọi là uỷ ban làm việc hay thách thức cũng được. Chứng minh rằng ta có thể chia các đai biều thành đúng 100 uỷ ban, mỗi đại biểu thuộc đúng 1 uỷ ban, sao cho các uỷ ban này hoặc là uỷ ban làm việc, hoặc là uỷ ban thách thúc.

Ban biên tập đã bổ sung giả thiết như bài toán vừa giải ở trên và cố gắng giữ nguyên các phát biểu còn lại.

Bài 4. Cho tam giác $A B C$ có $B, C$ cố định còn $A$ di động sao cho $A B=A C$ và $\angle B A C>60^{\circ}$. Đường thẳng đối xứng với $B C$ qua $A B$ cắt $A C$ tại $P$. Trên đoạn $P C$ lấy điểm $M$ sao cho $P M=P B$. Gọi $N$ là giao điểm của $A B$ và phân giác ngoài của góc $\angle B C A$. Chứng minh rằng $M N$ luôn đi qua một điểm cố định.

Lời giải. Tam giác $P B M$ cân tại $P$ nên bằng biến đổi góc, ta có

$\quad\quad\quad\quad\quad \angle P B M=\angle P M B \Rightarrow 2 \angle A B C-\angle M B C=\angle A C B+\angle M B C .$

Do đó $\angle A B C=2 \angle M B C$ nên $B M$ là tia phân giác của $\angle A B C$. Theo tính chất đường phân giác thì

$\quad\quad\quad\quad\quad\quad\quad\quad\quad\quad\quad\quad \frac{M C}{M A}=\frac{B C}{B A}=\frac{B C}{A C}$

 

Lại có $C N$ là phân giác ngoài của $\angle A C B$ nên ta cũng có $\frac{N A}{N B}=\frac{C A}{C B}$. Gọi $I$ là trung điểm của $B C$ thì $I$ là điểm cố định.

Xét tam giác $A B C$ với $I$ thuộc $B C, M$ thuộc $A C$ và $N$ thuộc $A B$ với

$\quad\quad\quad\quad\quad\quad\quad\quad\quad\quad\quad\quad \frac{I B}{I C} \cdot \frac{M C}{M A} \cdot \frac{N A}{N B}=1 \cdot \frac{B C}{A C} \cdot \frac{A C}{B C}=1$

thì theo định lý Menelaus đảo, ta có $M, N, I$ thẳng hàng.

Vậy $M N$ luôn đi qua điểm $I$ cố định.

Ngày thi thứ hai

Bài 5. Cho 2014 số thực $x_1, x_2, \ldots, x_{2014}$ thoả mãn các điều kiện

$\quad (i) x_1+x_2+\cdots+x_{2014}=0$,

$\quad (ii) x_1^2+x_2^2+\cdots+x_{2014}^2=2014$.

Tìm giá trị lớn nhất của biểu thức $P=x_1 x_2 \cdots x_{2014}$.

Lời giải. Rõ ràng có thể chọn giá trị các biến thích hợp để $P>0$ nên để tìm giá trị lớn nhất của $P$ thì ta chỉ xét các số $x_1, x_2, \ldots, x_{2014}$ đều khác 0 và số các số âm là chẵn. Không mất tính tổng quát, giả sử $x_1 \geq x_2 \geq \ldots \geq x_{2 m}>0>x_{2 m+1} \geq$ $\ldots \geq x_{2014}$. Đổi dấu các số $y_k=-x_k>0$ với $2 m+1 \leq k \leq 2014$. Khi đó ta viết lại

$\quad\quad\quad\quad\quad\quad \left\{\begin{array}{l}x_1+x_2+\cdots+x_{2 m}=y_1+y_2+\cdots+y_{2 n}=A \\ x_1^2+x_2^2+\cdots+x_{2 m}^2+y_1^2+y_2^2+\cdots+y_{2 n}^2=2014\end{array}\right.$

trong đó $m+n=1007$ (ngoài ra $m, n>0$ vì các số đã cho không thể toàn bộ là dương hoặc toàn bộ là âm). Theo bất đẳng thức Cauchy-Schwarz thì

$\quad\quad\quad\quad\quad\quad\quad\quad\quad\quad\quad\quad 2014 \geq \frac{A^2}{2 m}+\frac{A^2}{2 n} \text { nên } A^2 \leq 4 m n$

Lại theo bất đẳng thức AM-GM thì

$\quad\quad\quad\quad\quad\quad\quad\quad\quad P =\left(x_1 x_2 \ldots x_{2 m}\right)\left(y_1 y_2 \ldots y_{2 n}\right) \leq\left(\frac{A}{2 m}\right)^{2 m}\left(\frac{A}{2 n}\right)^{2 n} $

$\quad\quad\quad\quad\quad\quad\quad\quad\quad\quad =\frac{A^{2 m+2 n}}{2^{2 m+2 n} m^{2 m} n^{2 n}} \leq \frac{(4 m n)^{m+n}}{2^{2 m+2 n} m^{2 m} n^{2 n}}=\left(\frac{m}{n}\right)^{n-m}$

Do $m, n$ khác tính chẵn lẻ nên với vai trò bình đẳng của $m, n$, ta có thể giả sử $m<n$ nên $n-m \geq 1$ và $m \leq 503$. Khi đó, áp dụng bất đẳng thức Bernoulli thì

$\quad\quad\quad \left(\frac{n}{m}\right)^{n-m} \geq 1+\left(\frac{n}{m}-1\right)(n-m)=1+\frac{(n-m)^2}{m} \geq 1+\frac{1}{503}=\frac{504}{503}.$

Suy ra $P \leq\left(\frac{m}{n}\right)^{n-m} \leq \frac{503}{504}$. Đây chính là giá trị lớn nhất cần tìm, dấu bằng xảy ra khi $m=503, n=504$ và

$\quad\quad\quad x_1=x_2=\cdots=x_{1006}=\sqrt{\frac{504}{503}}, x_{1007}=x_{1008}=\cdots=x_{2014}=-\sqrt{\frac{503}{504}} .$

Bài 6. Cho dãy số $\left(u_n\right)$ xác định bởi $u_1=1, u_{n+1}=\frac{u_n}{\sqrt{u_n^2+1}+\sqrt{2}} \forall n \in \mathbb{N}^*$.

$\operatorname{Tính} \lim \frac{u_{n+1}}{u_n}$.

Lời giải. Xét hàm số $f(x)=\frac{x}{\sqrt{x^2+1}+\sqrt{2}}$ với $x \in \mathbb{R}$ thì

$\quad\quad\quad\quad\quad\quad\quad\quad f^{\prime}(x)=\frac{1+\sqrt{2+2 x^2}}{\sqrt{1+x^2}\left(\sqrt{2}+\sqrt{1+x^2}\right)^2}>0$

nên hàm này đồng biến trên $\mathbb{R}$. Dãy số đã cho được viết lại thành

$\quad\quad\quad\quad\quad\quad\quad\quad\quad\quad \left\{\begin{array}{l}u_1=1, \\ u_{n+1}=f\left(u_n\right), n \geq 1\end{array}\right.$

thì $u_1<u_2$ nên dễ dàng chứng minh quy nạp được rằng dãy này giảm.

Do dãy này bị chặn dưới bởi 0 nên nó có giới hạn, đặt giới hạn đó là $L \geq 0$. Trong biểu thức xác định dãy, cho $n \rightarrow+\infty$, ta được

$\quad\quad\quad\quad\quad\quad\quad\quad\quad\quad L=\frac{L}{\sqrt{L^2+1}+\sqrt{2}}$

nên $L=0$. Từ đó suy ra

Bài 7 . Cho $n$ là số nguyên dương và $A$ là tập con khác rỗng của $X={1,2, . ., n}$.

(a) Tính giá trị của tổng $S(A)=\sum_{E \subset X}(-1)^{|E \cap A|}$ trong đó $E$ lấy trên tất cả các tập hợp con của $X$, kể cả tập hợp rỗng.

(b) Cho $m \in \mathbb{N}^*$. Xét $m$ tập con khác rỗng của $X$ là $A_1, A_2, \ldots, A_m$ và $m$ số nguyên khác 0 là $a_1, a_2, \ldots, a_m$ sao cho $a_1+a_2+\cdots+a_m<0$. Chứng minh rằng tồn tại $E \subset X$ sao cho $\sum_{i=1}^m(-1)^{\left|E \cap A_i\right|} a_i>0$.

Lời giải. (a) Nếu $A=X$ thì

$\quad\quad\quad\quad\quad\quad\quad\quad\quad\quad\quad S(A)=\sum_{E \subset X}(-1)^{|E|}=C_n^0-C_n^1+C_n^2-\cdots+(-1)^n C_n^n=0 .$

Còn nếu $A \neq X$, do $S(A)$ chỉ phụ thuộc vào số phần tử của $A$ nên không mất tính tổng quát, ta giả sử rằng $A={1,2, \ldots, k}$ với $k<n$. Khi đó, ta có

$\quad\quad\quad\quad\quad\quad\quad S(A) =\sum_{E \subset X-{k}}(-1)^{|E \cap A|}+\sum_{E \subset X-{k}}(-1)^{|(E \cup{k}) \cap A|} $

$\quad\quad\quad\quad\quad\quad\quad\quad\quad\quad =\sum_{E \subset X-{k}}(-1)^{|E \cap A|}-\sum_{E \subset X-{k}}(-1)^{|E \cap A|}=0 .$

Vậy $S(A)=0, \forall A \subset X$.

(b) Đặt $f(E)=\sum_{i=1}^m(-1)^{\left|E \cap A_i\right|} a_i$. Giả sử $f(E) \leq 0, \forall E$. Mà ta cũng có

$\quad\quad\quad\quad\quad\quad\quad\quad\quad\quad \sum_{E \subset X} f(E)=\sum_{i=1}^m a_i S\left(A_i\right)=0$

Suy ra $f(E)=0, \forall E \subset X$, nhưng điều này là không thể vì $f(\varnothing)<0$. Vậy luôn tồn tại $E$ sao cho $f(E)>0$.

Bài 8. Cho tam giác $A B C$ nhọn có $H$ là trực tâm. $P$ là điểm di động bên trong tam giác $A B C$ sao cho $\angle B P C=\angle B H C$. Đường thẳng qua $B$ vuông góc $A B$ cắt $P C$ tại $M$, đường thẳng qua $C$ vuông góc $A C$ cắt $P B$ tại $N$. Gọi $I$ là trung điểm của $M N$. Chứng minh rằng $I$ luôn thuộc một đường thẳng cố định.

Lời giải. Vẽ đường kính $A A^{\prime}$ của đường tròn $(A B C)$. Vì $A^{\prime} B \perp A B$ nên $B, A^{\prime}, M$ thẳng hàng. Tương tự thì $C, A^{\prime}, N$ thẳng hàng. Giả sử $B A^{\prime}, C A^{\prime}$ cắt lại $(B H C)$ lần lượt tại $E, F$. Mặt khác

$\quad\quad\quad\angle N P M=180^{\circ}-\angle B H C=\angle A=180^{\circ}-\angle B A^{\prime} C=\angle M A^{\prime} N$

nên $P A^{\prime} M N$ là tứ giác nội tiếp.

Ta sẽ chứng minh trung điểm của $A^{\prime} F, A^{\prime} E, M N$ là thẳng hàng. Theo định lý Menelaus đảo thì điều nào tương đương với

$\quad\quad\quad\quad \frac{\overline{A^{\prime} F}}{\overline{A^{\prime} N}}=\frac{\overline{E A^{\prime}}}{\overline{E M}} \Leftrightarrow \frac{\overline{A^{\prime} F}}{\overline{\overline{A^{\prime} E}}}=-\frac{\overline{A^{\prime} N}}{\overline{E M}} \Leftrightarrow \frac{A^{\prime} B}{A^{\prime} C}=\frac{A^{\prime} N}{M E}\quad\quad(*) .$

Vì $\angle B N A^{\prime}=\angle C M E$ và $\angle N B A^{\prime}=\angle M C E$ nên hai tam giác $B N A^{\prime}, C M E$ dồng dạng với nhau. Do đó

$\quad\quad\quad\quad\quad\quad\quad\quad\quad\quad \frac{A^{\prime} N}{M E}=\frac{A^{\prime} B}{C E} .$

Mặt khác, bằng biến đổi góc, ta cũng có tam giác $C A^{\prime} E$ cân tại $C$ nên $C E=C A^{\prime}$. Ta có được

$\quad\quad\quad\quad\quad\quad\quad\quad\quad\quad \frac{A^{\prime} N}{M E}=\frac{A^{\prime} B}{A^{\prime} C} .$

Do đó, khẳng định $(*)$ là đúng. Vậy nên điểm $I$ luôn nằm trên đường trung bình của tam giác $A^{\prime} E F$ là đường cố định.

 

 

 

 

 

 

 

 

 

 

 

 

 

 

 

 

 

 

 

 

 

 

 

 

 

 

 

 

 

 

 

 

 

 

 

 

 

 

 

 

 

 

 

 

 

 

 

 

 

 

 

Đề thi và đáp án kì thi chọn đội tuyển thi Quốc gia trường Phổ thông Năng khiếu năm học 2011 – 2012

 

ĐỀ THI

Ngày thi thứ nhất

Bài 1. Cho các số $a, b, c>0$ thoả mãn $a b+b c+c a=1$. Chứng minh rằng:

$\quad\quad\quad\quad\quad\quad\quad\quad\frac{1}{3+2\left(a^2-b c\right)}+\frac{1}{3+2\left(b^2-c a\right)}+\frac{1}{3+2\left(c^2-a b\right)} \geq 1$

Bài 2. Có bao nhiêu bộ số nguyên dương $(x, y, z, t)$ thoả mãn

$\quad\quad\quad\quad\quad\quad\quad\quad\quad\quad 12<x<y<z<t \text { và } x+y+z+t=2011 ?$

Bài 3. Cho tam giác $A B C$ nội tiếp đường tròn $(O, R)$. Gọi $\left(\mathcal{C}_1\right)$ là đường tròn thay đổi luôn qua $B, C$ và lần lượt cắt các cạnh $A B, A C$ tại $M, N$ khác $B, C$.

(a) Chứng minh rằng $(A M N)$ luôn tiếp xúc với một đường cố định.

(b) Cho $B, C$ cố định, $B C=2 R$ và $A$ thay đổi trên $(O)$. Đường thẳng qua $A$ vuông góc $B C$ cắt $(O)$ tại $D$ và cắt $\left(\mathcal{C}_1\right)$ tại $E, F$. Chứng minh rằng nếu $A$ và $\left(\mathcal{C}_1\right)$ thay đổi sao cho $\frac{E F}{A D}=\frac{\sqrt{5}}{2}$ thì $(A M N)$ luôn tiếp xúc với một đường cố định.

Bài 4. Cho $p$ là số nguyên tố lẻ và đa thức $Q(x)=(p-1) x^p-x-1$. Xét dãy số $\left(a_n\right)$ thoả mãn

$\quad\quad\quad\quad\quad\quad\quad\quad a_0=\frac{p-1}{2}, a_n=a_{n-1}+Q\left(a_{n-1}\right) \forall n \in \mathbb{N}^* .$

(a) Chứng minh rằng với mọi số nguyên dương $n$ thì $\left(a_n, p\right)=1$.

(b) Chứng minh rằng với mọi số nguyên dương $n$ thì $Q\left(a_n\right) \equiv 0\left(\bmod p^n\right)$.

Ngày thi thứ hai

Bài 5. Cho dãy số $\left(u_n\right)$ thoả mãn $u_1=\frac{1}{6}$ và $u_{n+1}=u_n^2+\frac{2}{3} u_n \forall n \in \mathbb{N}^*$.

Tìm $\lim \frac{5 u_{n+1}^2-2 u_n^2 u_{n+1}+5 u_n u_{n+1}}{3 u_n^2+u_n u_{n+1}\left(4+u_n^2\right)}$.

Bài 6. Cho hàm số $f: \mathbb{N} \times \mathbb{N} \rightarrow \mathbb{N}$ thoả mãn $f(0,0)=0$ và:

$\quad\quad\quad\quad\quad\quad\quad\quad f(a, b)=\left\{\begin{array}{l}f\left(\left\lfloor\frac{a}{2}\right\rfloor,\left\lfloor\frac{b}{2}\right\rfloor\right) \text { khi } a+b \equiv 0 \quad(\bmod 2) \\ 1+f\left(\left\lfloor\frac{a}{2}\right\rfloor,\left\lfloor\frac{b}{2}\right\rfloor\right) \text { khi } a+b \equiv 1 \quad(\bmod 2)\end{array}\right.$

(a) Có bao nhiêu số tự nhiên $m \leq 2011$ sao cho $f(2011, m)=5$ ?

(b) Cho số lẻ $p$, cho $n \in \mathbb{N}\left(1<p<2^n\right)$ và $A$ là tập hợp gồm $p$ số tự nhiên không vượt quá $2^n-1$. Chứng minh rằng $\sum_{{a, b} \subset A} f(a, b) \leq n \cdot \frac{p^2-1}{4}$.

Bài 7. Cho tam giác $A B C$ nội tiếp đường tròn $(O)$ với $B, C$ cố định và $A$ thay đổi trên $(O)$. Đường trung trực $d$ của $B C$ cắt $A B, A C$ tại $M, N$. Gọi $P, Q$ lần lượt là các điểm đối xứng của $M, N$ qua $O . K$ là giao điểm của $B P$ và $C Q$.

(a) Chứng minh rằng $K$ luôn thuộc một đường tròn cố định.

(b) Kết luận trên còn đúng không khi $d$ là đường thẳng Euler của tam giác $A B C ?$

Bài 8. Với mọi số nguyên dương $n$, đặt $S_n=x^n+y^n+z^n$. Ta đã biết rằng $S_n=$ $P_n(s, t, p)$ với $s=x+y+z, t=x y+y z+z x, p=x y z$. Hãy tính tổng các hệ số của các đơn thức chứa $p$ trong $P_{2011}(s, t, p)$.

 

LỜI GIẢI

Bài 1. Cho các số $a, b, c>0$ thoả mãn $a b+b c+c a=1$. Chứng minh rằng:

$\quad\quad\quad\quad\quad\quad\quad\quad \frac{1}{3+2\left(a^2-b c\right)}+\frac{1}{3+2\left(b^2-c a\right)}+\frac{1}{3+2\left(c^2-a b\right)} \geq 1$

Lời giải. Đặt $a b=x ; b c=y ; c a=z$ thì ta có $x+y+z=1$. Khi đó áp dụng bất đẳng thức Cauchy-Schwarz, ta có

$\quad\quad\quad\quad\quad \sum_{c y c} \frac{1}{3+2\left(\frac{x z}{y}-y\right)} =\sum_{c y c} \frac{y^2}{3 y^2+2 x y z-2 y^3} $

$\quad\quad\quad\quad\quad\quad\quad\quad\quad\quad\quad\quad  \geq \frac{1}{3\left(x^2+y^2+z^2\right)+6 x y z-2\left(x^3+y^3+z^3\right)} .$

Ta đưa về chứng minh

$\quad\quad\quad\quad\quad\quad\quad\quad 3\left(x^2+y^2+z^2\right)+6 x y z-2\left(x^3+y^3+z^3\right) \leq 1 .$

Tuy nhiên đây lại là đẳng thức vì

$\quad\quad\quad\quad 3\left(x^2+y^2+z^2\right)+6 x y z-2\left(x^3+y^3+z^3\right) $

$\quad\quad\quad\quad\quad =3\left(x^2+y^2+z^2\right)-2(x+y+z)\left(x^2+y^2+z^2-x y-y z-z x\right) $

$\quad\quad\quad\quad\quad =3\left(x^2+y^2+z^2\right)-2\left(x^2+y^2+z^2\right)+2(x y+y z+z x) $

$\quad\quad\quad\quad\quad =(x+y+z)^2=1$

Đẳng thức xảy ra khi $x=y=z=1$ hay $a=b=c=\frac{1}{\sqrt{3}}$.

Bài 2. Có bao nhiêu bộ số nguyên dương $(x, y, z, t)$ thoả mãn $12<x<y<z<t$ và $x+y+z+t=2011 ?$

Lời giải. Đặt $x^{\prime}=x-12 ; y^{\prime}=y-12 ; z^{\prime}=z-12 ; t^{\prime}=t-12$. Phương trình đã cho tương đương với:

$\quad\quad\quad\quad\quad x^{\prime}+y^{\prime}+z^{\prime}+t^{\prime}=2011-48=1963 \text { với } 0 \leq x^{\prime}<y^{\prime}<z^{\prime}<t^{\prime}.$

Theo bài toán chia kẹo Euler thì nếu không có điều kiện thứ hai, số nghiệm của phương trình trên sẽ là $C_{1966}^3$. Ta sẽ trừ ra các trường hợp các số bị trùng nhau

  • Số bộ có 3 số giống nhau là $C_4^3 \cdot\left(1+\left\lfloor\frac{1963}{3}\right\rfloor\right)=2620=A$.
  • Số bộ có 2 số giống nhau là $C_4^2\left(\sum_{a=0}^{981}(1964-2 a)\right)=5791836=B$.

Do mỗi bộ nghiệm như trên chỉ tồn tại 1 cách sắp xếp $x, y, z, t$ thỏa mãn nên số bộ thoả mãn đề bài là

$\quad\quad\quad\quad\quad\quad\quad\quad \frac{C_{1966}^3-B+2 A}{4 !}=\frac{C_{1966}^3-5786596}{4 !} .$

Bài 3. Cho tam giác $A B C$ nội tiếp đường tròn $(O, R)$. Gọi $\left(\mathcal{C}_1\right)$ là đường tròn thay đổi luôn qua $B, C$ và lần lượt cắt các cạnh $A B, A C$ tại $M, N$ khác $B, C$.

(a) Chứng minh rằng $(A M N)$ luôn tiếp xúc với một đường cố định.

(b) Cho $B, C$ cố định, $B C=2 R$ và $A$ thay đổi trên $(O)$. Đường thẳng qua $A$ vuông góc $B C$ cắt $(O)$ tại $D$ và cắt $\left(\mathcal{C}_1\right)$ tại $E, F$. Chứng minh rằng nếu $A$ và $\left(\mathcal{C}_1\right)$ thay đổi sao cho $\frac{E F}{A D}=\frac{\sqrt{5}}{2}$ thì $(A M N)$ luôn tiếp xúc với một đường cố định.

Lời giải. (a) Gọi $d$ là đường thẳng qua $A$, song song với $B C$ và cắt đường tròn $(O)$ tại $T$. Bằng biến đổi góc, ta có

$\quad\quad\quad\quad\quad\quad\quad\quad \angle T A C=\angle A C B=\angle A M N .$

Suy ra $A T$ là tiếp tuyến của đường tròn $(A M N)$ nên ( $A M N)$ tiếp xúc với đường thẳng $d$ cố định.

(b) Gọi $H$ là hình chiếu của $A$ lên $B C$. Xét phương tích từ $A$ đến $\mathcal{C}_1$ thì

$\quad\quad\quad\quad\quad\quad\quad\quad A M \cdot A B=A N \cdot A C=A E \cdot A F \text {. }$

Ta có $H E \cdot H F=H B \cdot H C=H A^2$, mà $\frac{E F}{A D}=\frac{\sqrt{5}}{2}$ nên $H E+H F=\sqrt{5} A H$. Giải hệ này, ta có

$\quad\quad\quad\quad\quad\quad\quad\quad H E=\frac{\sqrt{5}-1}{2} A H \text { và } H F=\frac{\sqrt{5}+1}{2} A H .$

Suy ra $A E=A H-H E=\frac{3-\sqrt{5}}{2} A H$ và $A F=A H+H F=\frac{3+\sqrt{5}}{2} A H$. Từ đó ta được

$\quad\quad\quad\quad\quad\quad\quad\quad A E \cdot A F=\frac{3-\sqrt{5}}{2} A H \cdot \frac{3+\sqrt{5}}{2} A H=A H^2 \text {. }$

Vì thế nên $A H^2=A M \cdot A B=A N \cdot A C$, chứng tỏ $H M, H N$ lần lượt vuông góc với $A B, A C$. Suy ra $(A M N)$ có đường kính là $A H$ nên $(A M N)$ tiếp xúc với $B C$ là đường thẳng cố định.

Bài 4. Cho $p$ là số nguyên tố lẻ và đa thức $Q(x)=(p-1) x^p-x-1$. Xét dãy số $\left(a_n\right)$ thoả mãn

$\quad\quad\quad\quad\quad\quad\quad\quad a_0=\frac{p-1}{2}, a_n=a_{n-1}+Q\left(a_{n-1}\right) \forall n \in \mathbb{N}^* .$

(a) Chứng minh rằng với mọi số nguyên dương $n$ thì $\operatorname{gcd}\left(a_n, p\right)=1$.

(b) Chứng minh rằng với mọi số nguyên dương $n$ thì $Q\left(a_n\right) \equiv 0\left(\bmod p^n\right)$.

Lời giải. (a) Ta có

$\quad\quad\quad\quad a_0=\frac{p-1}{2} \text { và } a_1=a_0+(p-1) a_0^p-a_0-1=\frac{(p-1)^{p+1}}{2^p}-1$

không chia hết cho $p$.

Giả sử tồn tại $k$ nhỏ nhất sao cho $p \mid a_k$ thì $k \geq 2$. Ta có

$\quad\quad\quad\quad a_k=(p-1) a_{k-1}^p-1 \text { và } p-1 \equiv-1, a_{k-1}^p \equiv a_k \quad(\bmod p) .$

Suy ra $a_{k-1} \equiv-1(\bmod p)$ từ đó ta được $a_{k-2} \equiv 0(\bmod p)$, mâu thuẫn với tính nhỏ nhất của $k$. Vậy nên ta phải có $\operatorname{gcd}\left(a_n, p\right)=1$ với mọi $n$ nguyên dương.

(b) Ta có $Q(x)=(p-1) x^p-x-1 \equiv(-1) x-x-1=-2 x-1(\bmod p)$ với mọi $x$ nguyên nên

$\quad\quad\quad\quad Q\left(a_1\right) \equiv-2 a_1-1 \equiv-2(p-1) \frac{p-1}{2}+2-1=0 \quad(\bmod p)$

nên khẳng định đúng với $n=1$. Ta sẽ chứng minh bằng quy nạp.

$\quad\quad\quad\quad Q\left(a_{n+1}\right)=(p-1)\left(a_{n+1}^p-a_n^p\right)=(p-1) Q\left(a_n\right)\left(\sum_{i=1}^p a_n^{i-1} a_{n+1}^{p-i}\right).$

Giả sử rằng $p^n \mid Q\left(a_n\right)$ nên suy ra

$\quad\quad\quad\quad a_{n+1} \equiv a_n \quad(\bmod p) \Rightarrow \sum_{i=1}^p a_n^{i-1} a_{n+1}^{p-i} \equiv p a_n^{p-1} \equiv 0 \quad(\bmod p) .$

Như vậy $p^{n+1} \mid Q\left(a_{n+1}\right)$.Theo nguyên lí quy nạp thì ta có điều phải chứng minh.

Bài 5. Cho dãy số $\left(u_n\right)$ thoả mãn $u_1=\frac{1}{6}$ và $u_{n+1}=u_n^2+\frac{2}{3} u_n \forall n \in \mathbb{N}^*$.

Tìm $\lim \frac{5 u_{n+1}^2-2 u_n^2 u_{n+1}+5 u_n u_{n+1}}{3 u_n^2+u_n u_{n+1}\left(4+u_n^2\right)}$.

Lời giải. Trước hết, ta sẽ tìm giới hạn của dãy $\left(u_n\right)$. Bằng quy nạp, ta sẽ chứng minh rằng $0<u_n<\frac{1}{3}, \forall n$. Thật vậy,

  • Với $n=1$ thì khẳng định đúng.
  • Giả sử khẳng định đúng với $n=k>1$ thì $0<u_k<\frac{1}{3}$. Ta có:

$\quad\quad\quad\quad\quad\quad\quad\quad 0<u_{k+1}=u_k^2+\frac{2}{3} u_k<\frac{1}{9}+\frac{2}{3} \cdot \frac{1}{3}=\frac{1}{3}$

nên khẳng định cũng đúng với $n=k+1$.

Theo nguyên lí quy nạp, khẳng định được chứng minh. Xét hàm số $f(x)=x^2+$ $\frac{2}{3} x, x \in\left(0 ; \frac{2}{3}\right)$ thì $f^{\prime}(x)=2 x+\frac{2}{3}>0$ nên đây là hàm đồng biến. Dãy số đã cho chính là $u_1=\frac{1}{6}, u_{n+1}=f\left(u_n\right), n=1,2,3, \ldots$

Hơn nữa $u_2=\frac{1}{6^2}+\frac{2}{3} \cdot \frac{1}{6}=\frac{5}{36}<\frac{1}{6}$ nên đây là dãy giảm và bị chặn dưới nên có giới hạn. Gọi $l$ là giới hạn của dãy thì

$\quad\quad\quad\quad\quad\quad\quad\quad l=l^2+\frac{2}{3} l \Leftrightarrow l=0 \text { hay } l=\frac{1}{3} \text {. }$

Nhưng do dãy này giảm và theo chứng minh trên thì $0<u_n<\frac{1}{3}, \forall n$ nên giới hạn của dãy là 0 .

Theo công thức xác định dãy, ta có $\frac{u_{n+1}}{u_n}=u_n+\frac{2}{3}$. Do dãy $\lim u_n=0$ nên dãy tương ứng $\left(\frac{u_{n+1}}{u_n}\right)$ có giới hạn là $\frac{2}{3}$. Từ đó, ta tính được

$\quad\quad\quad\quad\quad\quad\quad\quad\quad\quad\quad\quad\quad\quad =\frac{5\left(\frac{2}{3}\right)^2-2 \cdot 0+5 \cdot \frac{2}{3}}{3+\frac{2}{3}\left(4+0^2\right)}=\frac{50}{51} .$

Bài 6 . Cho hàm số $f: \mathbb{N} \times \mathbb{N} \rightarrow \mathbb{N}$ thoả mãn $f(0,0)=0$ và:

$\quad\quad\quad\quad\quad\quad\quad\quad f(a, b)=\left\{\begin{array}{l}f\left(\left\lfloor\frac{a}{2}\right\rfloor,\left\lfloor\frac{b}{2}\right\rfloor\right) \text { khi } a+b \equiv 0 \quad(\bmod 2) \\ 1+f\left(\left\lfloor\frac{a}{2}\right\rfloor,\left\lfloor\frac{b}{2}\right\rfloor\right) \text { khi } a+b \equiv 1 \quad(\bmod 2)\end{array}\right.$

(a) Có bao nhiêu số tự nhiên $m \leq 2011$ sao cho $f(2011, m)=5$ ?

(b) Cho số lẻ $p$, cho $n \in \mathbb{N}\left(1<p<2^n\right)$ và $A$ là tập hợp gồm $p$ số tự nhiên không vượt quá $2^n-1$. Chứng minh rằng $\sum_{{a, b} \subset A} f(a, b) \leq n \cdot \frac{p^2-1}{4}$.

Lời giải. (a) Đổi số 2011 sang hệ nhị phân, ta có $2011=\overline{11111011011}(2)$. Khi đổi số $m$ sang hệ nhị phân, ta cũng có tương ứng $m=\overline{a_1 a_2 \ldots a_{11}(2)}$ (do $m \leq 11$ nên ta chỉ xét 11 chữ số).

Do công thức xác định của hàm, ta thấy $f(2011, m)$ chính bằng số vị trí trong dãy chữ số trên mà hai chữ số tại cùng vị trí là khác tính chẵn lẻ.

Trong 11 chữ số của $m$, ta chọn 5 vị trí để cho chúng khác tính chẵn lẻ với các chữ số của 2011 thì có $C_{11}^5=462$ cách. Mỗi cách chọn tính chẵn lẻ đó tương ứng với đúng một số $m$.

Tuy nhiên, ta phải trừ đi trường hợp đổi tính chẵn lẻ tại vị trí thứ 6 (và giữ nguyên từ $a_1 \rightarrow a_5$ ), tức là

$\quad\quad\quad\quad\quad\quad\quad\quad a_1=a_2=\cdots=a_6=1$

khi đó thì $m>2011$, không thỏa mãn. Ta sẽ đếm số cách chọn $m$ như thế. Trong 5 vị trí từ $a_7 \rightarrow a_{11}$, chọn ra 4 vị trí để đổi tính chẵn lẻ, có $C_5^4=5$ cách. Chú ý rằng số 0 ở vị trí thứ 9 không ảnh hưởng vì sau nó chỉ còn 2 vị trí, không đủ để thực hiện chọn ra 4 vị trí để đổi tính chẵn lẻ như trên.

Vậy nên số các số $m$ thỏa mãn là $462-5=457$.

(b) Đổi tất cả $p$ số của tập $A$ sang hệ nhị phân thì mỗi số sẽ có không quá $n$ chữ số và xếp vào bảng ô vuông kích thước $p \times n$. Mỗi dòng tương ứng với một số, và số nào không có đủ $n$ chữ số trong hệ nhị phân thì ta thêm 0 vào trước nó. Khi đó, tổng $\sum_{{a, b} \subset A} f(a, b)$ chính bằng tổng các cặp vị trí khác nhau trên mỗi cột.

$\quad\quad\quad\quad\quad\quad\quad\quad\quad\quad\quad\quad \begin{array}{|l|l|l|l|}\hline a_{1,1} & a_{1,2} & \cdots & a_{1, n} \\ \hline a_{2,1} & a_{2,2} & \cdots & a_{2, n} \\ \hline \cdots & \cdots & \cdots & \cdots \\ \hline a_{p, 1} & a_{p, 2} & \cdots & a_{p, n} \\ \hline\end{array}$

Xét cột thứ 1 , giả sử trên đó có $x$ số 0 và $y$ số 1 với $x+y=p$. Khi đó, số cặp chữ số khác nhau trên cột này sẽ là

$\quad\quad\quad\quad x y=\frac{1}{4}\left[(x+y)^2-(x-y)^2\right]=\frac{1}{4}\left[p^2-(x-y)^2\right] \leq \frac{p^2-1}{4}$

(do $x, y$ khác tính chẵn lẻ nên $|x-y| \geq 1$ ). Tương tự với các cột khác, số cặp chữ số khác nhau cũng không vượt quá $\frac{p^2-1}{4}$. Và do tính độc lập giữa các cột, ta có

$\quad\quad\quad\quad\quad\quad\quad\quad \sum_{{a, b} \subset A} f(a, b) \leq n \cdot \frac{p^2-1}{4} .$

Bài 7. Cho tam giác $A B C$ nội tiếp đường tròn $(O)$ với $B, C$ cố định và $A$ thay đổi trên $(O)$. Đường trung trực $d$ của $B C$ cắt $A B, A C$ tại $M, N$. Gọi $P, Q$ lần lượt là các điểm đối xứng của $M, N$ qua $O$. $K$ là giao điểm của $B P$ và $C Q$.

(a) Chứng minh rằng $K$ luôn thuộc một đường tròn cố định.

(b) Kết luận trên còn đúng không khi $d$ là đường thẳng Euler của tam giác $A B C ?$

Lời giải. Ta sẽ chứng minh bài toán tổng quát khi thay trung trực $B C$ lẫn đường thẳng Euler bởi đường thẳng $d$ bất kỳ đi qua $O$.

Kẻ đường kính $B B^{\prime}, C C^{\prime}$ của $(O)$ và giả sử $B^{\prime} N, C^{\prime} M$ cắt nhau ở $T$. Khi đó, vì $M, N, O$ thẳng hàng nên theo định lý Pascal đảo thì lục giác tạo bởi các đỉnh $A, B, C, B^{\prime}, C^{\prime}, T$ nội tiếp. Do đó, $T \in(O)$.

Ngoài ra, vì $\angle B T N=\angle B T B^{\prime}=90^{\circ}$ nên $T B \perp T N$, tương tự thì $T C \perp T M$.

Kẻ đường kính $T K^{\prime}$ của $(O)$ thì do $O$ là trung diểm chung của $T K^{\prime}, M P$ nên tứ giác $T M K^{\prime} P$ là hình bình hành. Suy ra $T M | K^{\prime} P$ nên $K^{\prime} P \perp T C$.

Mà tứ giác $C M C^{\prime} P$ cũng là hình bình hành nên $C P | C^{\prime} M$, mà $C^{\prime} M \perp T C$ nên $C P \perp T C$. Từ các điều này, ta suy ra $K^{\prime}, P, C$ thẳng hàng. Tương tự thì $K^{\prime}, Q, B$ thẳng hàng. Vì thế nên $K^{\prime} \equiv K$, hay $K$ luôn thuộc đường tròn $(O)$ cố định.

Nhận xét. Trong bài toán trên, $T$ chính là giao điểm của hai đường tròn đường kính $B N, C M$. Nếu gọi $S$ là giao điểm còn lại thì ta chứng minh được bằng phép nghịch đảo trực tâm $H$ rằng $S$ nằm trên đường tròn Euler của tam giác $A B C$.

Bài 8. Với mọi số nguyên dương $n$, đặt $S_n=x^n+y^n+z^n$. Ta đã biết rằng $S_n=P_n(s, t, p)$ với $s=x+y+z, t=x y+y z+z x, p=x y z$. Hãy tính tổng các hệ số của các đơn thức chứa $p$ trong $P_{2011}(s, t, p)$.

Lời giải. Theo định lý Viete thì $x, y, z$ là nghiệm của phương trình

$\quad\quad\quad\quad\quad\quad\quad\quad\quad a^3-s a^2+t a-p=0 .$

Để tính tổng hệ số của tất cả các đơn thức trong $P_{2011}$, ta xét $P_{2011}(1,1,1)$. Tương tự, tổng các hệ số của các đơn thức không chứa $p$ trong $P_{2011}$ là $P_{2011}(1,1,0)$. Do đó, ta cần tính

$\quad\quad\quad\quad\quad\quad\quad\quad M=P_{2011}(1,1,1)-P_{2011}(1,1,0) .$

Xét phương trình $a^3-a^2+a-1=0$ có ba nghiệm là $a=1, a=i$ và $a=-i$. Vì $P_{2011}(s, t, p)=x^n+y^n+z^n$ nên ta có

$\quad\quad\quad\quad\quad\quad\quad P(1,1,1)=1^{2011}+i^{2011}+(-i)^{2011}=1 .$

Tiếp tục xét $a^3-a^2+a=0$ có ba nghiệm là $a=0, a=\frac{1 \pm i \sqrt{3}}{2}$. Áp dụng công thức Moivre của lũy thừa số phức, ta tính được

$\quad\quad\quad\quad P(1,1,0) =0^{2011}+\left(\frac{1+i \sqrt{3}}{2}\right)^{2011}+\left(\frac{1-i \sqrt{3}}{2}\right)^{2011} $

$\quad\quad\quad\quad\quad\quad\quad\quad =\left(\cos \frac{\pi}{3}+i \sin \frac{\pi}{3}\right)^{2011}-\left(\cos \frac{2 \pi}{3}+i \sin \frac{2 \pi}{3}\right)^{2011} $

$\quad\quad\quad\quad\quad\quad\quad\quad =\left(\cos \frac{2011 \pi}{3}+i \sin \frac{2011 \pi}{3}\right)-\left(\cos \frac{4022 \pi}{3}+i \sin \frac{4022 \pi}{3}\right)=1$

Vì thế nên $M=0$.

 

 

 

 

 

 

 

 

 

 

 

 

 

 

 

 

 

 

 

 

 

 

 

 

 

 

 

 

 

 

 

 

 

 

 

 

 

 

 

 

 

 

 

 

 

 

 

 

 

 

 

 

 

 

 

 

 

 

 

 

 

ĐỀ THI CHỌN ĐỘI TUYỂN HSG QUỐC GIA CỦA TRƯỜNG PTNK NĂM 2016 – 2017

ĐỀ THI

Ngày thi thứ nhất

Bài 1. Tìm tất cả $a$ để dãy số $\left(u_n\right)$ hội tụ, biết $u_1=a$ và $\forall n \in \mathbb{N}^*$ thì:

$\quad\quad\quad\quad\quad\quad\quad\quad\quad\quad u_{n+1}=\left\{\begin{array}{l}2 u_n-1 \text { nếu } u_n>0 \\ -1 \text { nếu }-1 \leq u_n \leq 0 \\ u_n^2+4 u_n+2 \text { nếu } u_n<-1\end{array}\right.$

Bài 2. Tìm số nguyên dương $k$ nhỏ nhất để bất đẳng thức

$\quad\quad\quad\quad\quad\quad\quad\quad\quad\quad x^k y^k z^k\left(x^3+y^3+z^3\right) \leq 3$

luôn đúng với mọi số thực dương $x, y, z$ thoả mãn điều kiện $x+y+z=3$.

Bài 3. Cho hàm số $f: \mathbb{N}^* \rightarrow \mathbb{N}^*$ thoả mãn hai điều kiện sau:

$\quad\quad$ i) $f$ là hàm tăng thật sự trên $\mathbb{N}^*$.

$\quad\quad$ ii) $f(2 n)=2 f(n) \forall n \in \mathbb{N}^*$.

(a) Giả sử $f(1)=3$ và $p>3$ là số nguyên tố. Chứng minh rằng tồn tại số nguyên dương $n$ sao cho $f(n)$ chia hết cho $p$.

(b) Cho $q$ là số nguyên tố lẻ. Hãy xây dựng một hàm $f$ thoả mãn các điều kiện của bài toán mà $f(n)$ không chia hết cho $q$ với mọi $n$ nguyên dương.

Bài 4. Cho tam giác $A B C$ có góc $\angle B A C$ tù và $A H \perp B C(H$ nằm trên $B C)$. Điểm $M$ thay đổi trên cạnh $A B$. Dựng điểm $N$ sao cho $\Delta B M N \sim \triangle H C A$, với $H$ và $N$ nằm khác phía đối với đường thẳng $A B$.

(a) Gọi $C M$ cắt đường tròn ngoại tiếp tam giác $B M N$ tại $K$. Chứng minh rằng $N K$ luôn đi qua một điểm cố định.

(b) Gọi $N H$ cắt $A C$ tại $P$. Dựng điểm $Q$ sao cho $\triangle H P Q \sim \triangle H N M$, với $Q$ và $M$ nằm khác phía đối với đường thẳng $N P$. Chứng minh rằng $Q$ luôn thuộc một đường thẳng cố định.

Ngày thi thứ hai

Bài 5. Với mỗi số nguyên dương $n$, tồn tại duy nhất số tự nhiên $a$ thoả mãn điều kiện $a^2 \leq n<(a+1)^2$. Đặt $\Delta_n=n-a^2$.

(a) Tìm giá trị nhỏ nhất của $\Delta_n$ khi $n$ thay đổi và luôn thoả mãn $n=15 m^2$ với $m$ là số nguyên dương.

(b) Cho $p, q$ là các số nguyên dương và $d=5(4 p+3) q^2$. Chứng minh rằng $\Delta_d \geq 5$.

Bài 6. Với các số nguyên $a, b, c, d$ thoả mãn $1 \leq a<b<c<d$, ký hiệu:

$T(a, b, c, d)=[(x, y, z, t) \subset \mathbb{N}^* \mid 1 \leq x<y<z<t, x \leq a, y \leq b, z \leq c, t \leq d]$

(a) Tình số phần tử của $T(1,4,6,7)$.

(b) Cho $a=1$ và $b \geq 4$. Gọi $d_1$ là số phần tử của $T(a, b, c, d)$ chứa 1 và không chứa $2 ; d_2$ là số phần tử chứa 1,2 và không chứa $3 ; d_3$ là số phần tử chứa $1,2,3$ và không chứa 4 . Chứng minh rằng $d_1 \geq 2 d_2-d_3$. Đẳng thức xảy ra khi nào?

Bài 7. Trong một hệ thống máy tính, một máy tính bất kỳ có kết nối trực tiếp với ít nhất $30 \%$ máy tính khác của hệ thống. Hệ thống này có một chương trình cảnh báo và ngăn chặn khá tốt, do đó khi một máy tính bị virus, nó chỉ có đủ thời gian lây cho các máy tính được kết nối trực tiếp với nó. Chứng minh rằng dù vậy, kẻ tấn công vẫn có thể chọn hai máy tính của hệ thống mà nếu thả virus vào hai máy đó, ít nhất $50 \%$ máy tính của hệ thống sẽ bị nhiễm virus.

Bài 8. Cho tam giác $A B C$ nhọn. Đường tròn $(I)$ có tâm $I$ thuộc cạnh $B C$ và tiếp xúc với các cạnh $A B, A C$ lần lượt tại $E, F$. Lấy $M, N$ bên trong tứ giác $B C E F$ sao cho $E F N M$ nội tiếp $(I)$ và các đường thẳng $M N, E F, B C$ dồng quy. Gọi $M F$ cắt $N E$ tại $P, A P$ cắt $B C$ tại $D$.

(a) Chứng minh rằng $A, D, E, F$ cùng thuộc một đường tròn.

(b) Lấy trên các đường thẳng $B N, C M$ các điểm $H, K$ sao cho $\angle A C H=$ $\angle A B K=90^{\circ}$. Gọi $T$ là trung điểm $H K$. Chứng minh rằng $T B=T C$.

 

LỜI GIẢI

Ngày thi thứ nhất

Bài 1. Tìm tất cả $a$ để dãy số $\left(u_n\right)$ hội tụ, biết $u_1=a$ và $\forall n \in \mathbb{N}^*$ thì:

$\quad\quad\quad\quad\quad\quad\quad\quad\quad\quad u_{n+1}=\left\{\begin{array}{l}2 u_n-1 \text { nếu } u_n>0, \\ -1 \text { nếu }-1 \leq u_n \leq 0, \\ u_n^2+4 u_n+2 \text { nếu } u_n<-1\end{array}\right.$

Lời giải. Có các trường hợp sau cần xem xét:

  • Nếu $a>1$, bằng quy nạp đơn giản, ta có $u_n>1 \forall n \in \mathbb{N}^*$ và

$\quad\quad\quad\quad\quad\quad\quad\quad\quad\quad u_n=2^{n-1}(a-1)+1, \forall n \in \mathbb{N}^* .$

Do $a>1$, cho $n \rightarrow+\infty$ thì $u_n \rightarrow+\infty$. Từ đó $\left(u_n\right)$ không hội tụ.

  • Nếu $a=1$ thì $u_n=1 \forall n \in \mathbb{N}^*$ hay $\left(u_n\right)$ hội tụ về 1 .

  • Nếu $0<a<1$, ta sẽ chứng minh rằng $\left(u_n\right)$ có ít nhất một số hạng không dương. Thật vậy, giả sử $u_n>0 \forall n \in \mathbb{N}^*$ thì theo trường hợp đầu tiên, ta có:

$\quad\quad\quad\quad\quad\quad\quad\quad\quad\quad u_n=2^{n-1}(a-1)+1 \forall n \in \mathbb{N}^*$

Do $a>1$, cho $n \rightarrow+\infty$ thì $u_n \rightarrow-\infty$, trái với việc $u_n>0 \forall n, \in \mathbb{N}^*$.

Từ đó điều giả sử là sai hay phải tồn tại $k \in \mathbb{N}^*\text { sao cho } u_k>0 \text { và } u_{k+1} \leq 0$. Với cách chọn chỉ số $\text{k}$ như vậy, ta có:

$\quad\quad\quad\quad\quad\quad\quad\quad\quad\quad -1 \leq 2 u_k-1=u_{k+1} \leq 0$

Khi đó $u_{k+2}=0$. Bằng quy nạp thì $u_n=-1 \forall n \in \mathbb{N}^*, n \geq k+2$. Điều này dễn đến $\left(u_n\right)$ hội tụ về $-1$.

  • Nếu $-1 \leq a \leq 0$, từ giả thiết thì $u_2=-1$. Bằng quy nạp thì $u_n=-1 \forall n \in$ $\mathbb{N}^*, n \geq 2$ hay $\left(u_n\right)$ hội tụ về $-1$.

  • Nếu $-2<a<-1$, ta có:

$\quad\quad\quad\quad\quad\quad\quad\quad\quad\quad u_2-u_1=a^2+3 a+2=(a+2)(a+1)<0$

Khi đó thì $u_2<u_1<-1$. Lại có $u_2=(a+2)^2-2 \geq-2$ nên $-2<u_2<-1$.

Bằng quy nạp, ta có $\left(u_n\right)$ là dãy giảm và $-2<u_n<-1$ nên $\left(u_n\right)$ hội tụ.

  • Nếu $-2-\sqrt{3} \leq a \leq-2$ thì $u_2=a^2-4 a+2$ và dễ có được:

$\quad\quad\quad\quad\quad\quad\quad\quad\quad\quad\quad\quad -1 \leq a^2-4 a+2 \leq 1$

Theo các trường hợp đã xét, dãy số $\left(u_n\right)$ hội tụ.

  • Nếu $a<-2-\sqrt{3}$, bằng vài tính toán, ta có $u^2=a^2-4 a+2>1$.

Theo trường hợp đầu tiên, dãy số $\left(u_n\right)$ không hội tụ.

Vậy dãy số $\left(u_n\right)$ hội tụ khi và chỉ khi $-2-\sqrt{3} \leq a \leq 1$.

Bài 2. Tìm số nguyên dương $k$ nhỏ nhất để bất đẳng thức

$\quad\quad\quad\quad\quad\quad\quad\quad\quad\quad x^k y^k z^k\left(x^3+y^3+z^3\right) \leq 3$

luôn đúng với mọi số thực dương $x, y, z$ thoả mãn điều kiện $x+y+z=3$.

Lời giải. Ta sẽ chứng minh rằng $k=3$ là số nguyên dương nhỏ nhất thoả mãn bài toán. Trước hết, chọn $x=y=\frac{3}{4}, z=\frac{3}{2}$ thì ta phải có:

$\quad\quad\quad\quad\quad\quad\quad\quad \left(\frac{3}{4}\right)^{2 k} \cdot\left(\frac{3}{2}\right)^k\left(2 \cdot\left(\frac{3}{4}\right)^3+\left(\frac{3}{2}\right)^3\right) \leq 3$

Dễ thấy đánh giá trên chỉ đúng nếu $k \geq 3$. Ta đưa về chứng minh rằng:

$\quad\quad\quad\quad\quad\quad\quad\quad\quad\quad x^3 y^3 z^3\left(x^3+y^3+z^3\right) \leq 3 .$

Không mất tính tổng quát, giả sử $x \geq y \geq z$ thì $z \leq 1$. Ta có:

$\quad\quad\quad\quad\quad x^3+y^3=(x+y)^3-3 x y(x+y)=(3-z)^3-3 x y(x+y) \text { hay } $

$\quad\quad\quad\quad\quad (3-z)^3+z^3 \leq \frac{3}{x^3 y^3 z^3}+3 x y(x+y)$

Khai triển và thu gọn, bất đẳng thức trở thành:

$\quad\quad\quad\quad\quad\quad\quad\quad\quad\quad 3 z^2-9 z+9 \leq \frac{1}{x^3 y^3 z^3}+x^2 y+x y^2$

Theo bất đẳng thức AM-GM, ta có vế phải của bất đẳng thức trên sẽ không nhỏ hơn $\frac{3}{z}$. Từ đây ta chỉ cần chứng minh rằng

$\quad\quad\quad\quad\quad\quad\quad 3 z^2-9 z+9 \leq \frac{3}{z} \text { hay } 3(z-1)^3 \leq 0 \text {, đúng. }$

Vậy $k=3$ là hằng số nguyên dương nhỏ nhất thoả mãn bài toán.

Nhận xét. Dưới đây là các cách xử lý khác cho bất đẳng thức ứng với $k=3$ ở trên.

Cách 1. Không mất tính tổng quát ta giả sử $x \leq y \leq z$. Khi đó luôn tồn tại $m>n \geq 0$ sao cho $x=m-n, y=m+n$. Khi đó

$\quad\quad\quad\quad\quad\quad\quad\quad\quad\quad z=3-2 m ; m=\frac{x+y}{2} \leq 1$

Xét hàm số

$f(n)=(m-n)^3(m+n)^3 z^3\left[z^3+(m-n)^3+(m+n)^3\right]=z^3\left(m^2-n^2\right)^3\left(z^3+2 m^3+6 m n^2\right)$

thì

$\quad\quad\quad\quad\quad f^{\prime}(n)=z^3\left(m^2-n^2\right)^2\left(-6 n z^3-48 m n^3\right) \leq 0$

nên

$\quad\quad\quad\quad\quad f(n) \leq f(0)=m^6 z^3\left(z^3+2 m^3\right)=m^6(3-2 m)^3\left((3-2 m)^3+2 m^3\right)$

Xét hàm số

$\quad\quad\quad\quad\quad\quad\quad\quad\quad\quad g(m)=m^6(3-2 m)^3\left[(3-2 m)^3+2 m^3\right]$

thì

$\quad\quad\quad\quad\quad g^{\prime}(m)=18 m^5(3-2 m)^2(m-1)\left[(m-1)\left(8 m^2-37 m+26\right)-1\right] \geq 0 .$

Vậy nên $g(m) \leq g(1)=3$, bài toán được giải quyết.

Cách 2. Không mất tính tổng quát, ta giả sử $z$ là số lớn nhất trong ba số $x, y, z$. Đặt $t=\frac{x+y}{2}$ và $f(x, y, z)=x^3 y^3 z^3\left(x^3+y^3+z^3\right)$. Ta sẽ chứng minh $f(x, y, z) \leq$ $f(t, t, z)$. Ta có

$\quad\quad\quad\quad f(t, t, z)-f(x, y, z)=z^3\left[t^6\left(2 t^3+z^3\right)-x^3 y^3\left(x^3+y^3+z^3\right)\right] .$

Mặt khác,

$t^6\left(2 t^3+z^3\right)-x^3 y^3\left(x^3+y^3+z^3\right)=z^3\left(t^6-x^3 y^3\right)+2 t^9-x^3 y^3(x+y)\left(x^2+y^2-x y\right) $

$=z^3\left(t^6-x^3 y^3\right)+2 t^9-2 t x^3 y^3\left(4 t^2-3 x y\right) \geq t^3\left(t^6-x^3 y^3\right)+2 t^9-2 t x^3 y^3\left(4 t^2-3 x y\right) $

$=3 t\left(t^2-x y\right)\left[t^6+x y\left(2 x y+t^2\right)\left(t^2-x y\right)\right] \geq 0 .$

Vậy nên

$\quad\quad\quad\quad\quad f(x, y, z) \leq f(t, t, z)=f(t, t, 3-2 t)=t^6(3-2 t)^3\left[2 t^3+(3-2 t)^3\right]$

Ta chỉ cần chứng minh

$\quad\quad\quad\quad\quad\quad\quad\quad\quad\quad t^6(3-2 t)^3\left[2 t^3+(3-2 t)^3\right] \leq 3$

Đến đây thực hiện như cách 1 ở trên.

Bài 3. Cho hàm số $f: \mathbb{N}^* \rightarrow \mathbb{N}^*$ thoả mãn hai điều kiện sau:

i) $f$ là hàm tăng thật sự trên $\mathbb{N}^*$.

ii) $f(2 n)=2 f(n) \forall n \in \mathbb{N}^*$.

(a) Giả sử $f(1)=3$ và $p>3$ là số nguyên tố. Chứng minh rằng tồn tại số nguyên dương $n$ sao cho $f(n)$ chia hết cho $p$.

(b) Cho $q$ là số nguyên tố lẻ. Hãy xây dựng một hàm $f$ thoả mãn các điều kiện của bài toán mà $f(n)$ không chia hết cho $q$ với mọi $n$ nguyên dương.

Lời giải. (a) Đặt $A=[f(n+1)-f(n) \mid n \in \mathbb{N}^*].$

Vì $\text { f là hàm số tăng thực sự trên } \mathbb{N}^* \text { nên } A \subset \mathbb{N}^*$.

Khi đó phải tồn tại $k=\min A \text { và tồn tại } n \in \mathbb{N}^* \text { để } k=f(n+1)-f(n)$. Khi đó:

$\quad\quad\quad\quad\quad\quad\quad\quad\quad\quad f(2 n+2)-f(2 n)=2 f(n+1)-2 f(n)=2 k .$

Lại có $f(2 n+2)-f(2 n+1), f(2 n+1)-f(2 n) \geq k$ nên

$\quad\quad\quad\quad\quad\quad\quad\quad\quad\quad f(2 n+2)-f(2 n+1)+f(2 n+1)-f(2 n) \geq 2 k .$

Từ đây ta phải có $f(2 n+2)-f(2 n+1)=f(2 n+1)-f(2 n)=k$. Bằng quy nạp theo $m$, ta chứng minh được

$\quad\quad\quad\quad\quad\quad\quad\quad\quad\quad f\left(2^m n+t\right)=2^m f(n)+t k \forall t, m \in \mathbb{N}, t \leq m .$

Lại có $f(1)=3, f(2)=6$ nên $k \leq 3<p$ hay $(k, p)=1$.

Xét $p$ số nguyên dương sau:

$\quad\quad\quad\quad\quad\quad\quad\quad\quad\quad f\left(2^p n\right), f\left(2^p n+1\right), f\left(2^p n+2\right), \ldots, f\left(2^p n+p-1\right)$

lập thành một cấp số cộng có công sai $k$ nên là một hệ thặng dư đầy đủ modulo $p$. Từ đó phải tồn tại một số hạng chia hết cho $p$.

(b) Ta xây dựng một hàm số $f$ với các điều kiện như sau:

$\quad\quad$ i) $f(1)=2^a>q\left(a \in \mathbb{N}^*\right.$,

$\quad\quad$ ii) $f(2 n)=2 f(n) \forall n \in \mathbb{N}^*$,

$\quad\quad$ iii) $f(2 n+1)=f(2 n)+q \forall n \in \mathbb{N}^*$.

Ta chứng minh rằng hàm số $f$ vừa xây dựng thỏa mãn bài toán.

Trước hết ta chứng minh rằng $f$ là hàm tăng thực sự, cụ thể là:

$\quad\quad\quad\quad\quad\quad\quad\quad\quad\quad f(n+1)-f(n) \geq q \forall n \in \mathbb{N}^* .$

Với $n=1$, ta có $f(2)-f(1)=2.2^a-2^a=2^a>q$. Giả sử khẳng định cần chứng minh đúng đến $n=k$. Xét các khả năng sau:

  • Nếu $k$ là số chẵn, ta có $f(k+1)=f(k)+q$ thỏa mãn yêu cầu.

  • Nếu $k$ là số lẻ, ta có:

$\quad\quad\quad\quad f(k+1)=2 f\left(\frac{k+1}{2}\right) \geq 2\left(f\left(\frac{k-1}{2}\right)+q\right)=f(k-1)+2 q .$

Lại có $f(k)=f(k-1)+q$ nên $f(k+1) \geq f(k)+q$.

Theo nguyên lý quy nạp, ta có $f(n+1)-f(n) \geq q \forall n \in \mathbb{N}^*$.

Bây giờ ta chứng minh rằng không tồn tại $n$ để $q \mid f(n)$. Trước hết thì $f(1)=2^a$ không chia hết cho $q$. Giả sử điều này đúng đến $n=k$. Xét các khả năng sau:

  • Nếu $k$ chẵn thì $f(k+1)=f(k)+q$ không chia hết cho $q$.

  • Nếu $k$ lẻ thì $f(k+1)=2 f\left(\frac{k+1}{2}\right)$ không chia hết cho $q$.

Theo nguyên lý quy nạp, $f(n)$ không chia hết cho $q$ với mọi $n \in \mathbb{N}^*$. Các điều kiện đã được kiểm tra đầy đủ.

Bài 4. Cho tam giác $A B C$ có góc $\angle B A C$ tù và $A H \perp B C(H$ nằm trên $B C$ ). Điểm $M$ thay đổi trên cạnh $A B$. Dựng điểm $N$ sao cho $\Delta B M N \sim \triangle H C A$, với $H$ và $N$ nằm khác phía đối với đường thẳng $A B$.

(a) Gọi $C M$ cắt đường tròn ngoại tiếp tam giác $B M N$ tại $K$. Chứng minh rằng $N K$ luôn đi qua một điểm cố định.

(b) Gọi $N H$ cắt $A C$ tại $P$. Dựng điểm $Q$ sao cho $\triangle H P Q \sim \Delta H N M$, với $Q$ và $M$ nằm khác phía đối với đường thẳng $N P$. Chứng minh rằng $Q$ luôn thuộc một đường thẳng cố định.

Lời giải. (a) Xét điểm $X$ trên $A C$ sao cho $\angle X B C=90^{\circ}$ và $K^{\prime}$ là giao điểm của $N X$ và $C M$. Ta có $\Delta B M N \sim \triangle B C X$ (cùng hướng). Từ đó có một phép vị tự quay tâm $B$ biến $M \mapsto N, C \mapsto X$.

Giả sử $C M$ cắt $B X$ tại $K^{\prime}$ thì $K^{\prime}$ thuộc đường tròn ngoại tiếp tam giác $B M N$. Từ đó $K^{\prime} \equiv K$ nên $N K$ luôn đi qua điểm $X$ cố định.

(b) Xét phép vị tự tâm $H$ biến

$\quad\quad\quad\quad\quad\quad\quad\quad\quad\quad N \mapsto P, M \mapsto Q, B \mapsto F .$

Ta có $\Delta B M N \sim \triangle F Q P$. Khi đó

$\quad\quad\quad\quad\quad\quad\quad\quad\quad\quad \angle F Q P=\angle B M N=\angle A C B=\angle F C P$

nên tứ giác $C F P Q$ nội tiếp. Từ đây dẫn đến

$\quad\quad\quad\quad\quad\quad\quad\quad\quad\quad \angle Q C P=\angle Q F P=\angle M B N=90^{\circ} .$

Vậy $Q$ thuộc đường thẳng qua $C$ vuông góc với $A C$, là đường thẳng cố định.

 

Ngày thi thứ hai

Bài 5. Với mỗi số nguyên dương $n$, tồn tại duy nhất số tự nhiên $a$ thoả mãn điều kiện $a^2 \leq n<(a+1)^2$. Đặt $\Delta_n=n-a^2$.

(a) Tìm giá trị nhỏ nhất của $\Delta_n$ khi $n$ thay đổi và luôn thoả mãn $n=15 m^2$ với $m$ là số nguyên dương.

(b) Cho $p, q$ là các số nguyên dương và $d=5(4 p+3) q^2$. Chứng minh rằng $\Delta_d \geq 5$.

Lời giải. (a) Ta cần tìm $\Delta_n$ nhỏ nhất để phương trình $15 m^2-a^2=\Delta_n$ có nghiệm nguyên dương. Nhận thấy $15-3^2=6$ nên $\min \Delta_n \leq 6$. Ta chứng minh rằng phương trình trên không có nghiệm nguyên dương với $\Delta_n<6$.

Ta có $3 \mid a^2+\Delta_n$. Suy ra $3 \mid \Delta_n$ hoặc $3 \mid \Delta_n+1$. Mặt khác $5 \mid a^2+\Delta_n$ nên $\Delta_n$ chia 5 chỉ có thể dư 0,1 hoặc 4 .

Từ đó nếu tồn tại $n$ để $\Delta_n<6$ thỏa mãn bài toán thì $\Delta_n=5$. Giả sử rằng tồn tại $n$ như thế, ta có $15 m^2-a^2=5$ hay $5 \mid a$. Đặt $a=5 s\left(s \in \mathbb{N}^*\right)$, ta có:

$\quad\quad\quad\quad\quad\quad\quad\quad\quad\quad\quad\quad 3 m^2-5 s^2=1 \text {. }$

Từ đó thì

$\quad\quad\quad\quad 3\left(m^2+s^2\right) \equiv 1 \quad(\bmod 8)$ hay $m^2+s^2 \equiv 3 \quad(\bmod 8)$

Điều này vô lý do $m^2$ chia 8 dư $0,1,4$. Vậy $\Delta_n$ nhỏ nhất là 6 .

(b) Ta có

$\quad\quad\quad\quad\quad\quad\quad\quad\quad\quad 5(4 p+3) q^2-a^2=\Delta_d .$

Do $a^2$ chia 5 dư $0,1,4$ nên $\Delta_d$ chia 5 dư $0,1,4$. Giả sử rằng có bộ số để $\Delta_d<5$. Xét các khả năng sau:

  • Nếu $\Delta_d=0$ thì $5(4 p+3) q^2=a^2$. Xét bộ số $(q, a)$ với $q+a$ nhỏ nhất. Từ phương trình trên, ta có $a^2+q^2 \equiv 0(\bmod 4)$ hay $a \equiv q \equiv 0(\bmod 2)$.

Đặt $a=2 a_1$ và $q=2 q_1$ với $a_1, q_1 \in \mathbb{N}^*$ thì bộ số $\left(q_1, a_1\right)$ cũng thoả mãn điều kiện $5(4 p+3) q_1^2=a_1^2$. Hơn nữa $q_1+a_1<q+a$, mâu thuẫn.

  • Nếu $\Delta_d=1$, ta có $a^2+1=5(4 p+3) q^2$. Do $5(4 p+3) \equiv 3(\bmod 4)$ nên số này tồn tại một ước nguyên tố $r \equiv 3(\bmod 4)$.

Do đó $a^2+1 \equiv 0(\bmod r)$ hay $r \mid 1$, vô lý.

  • Nếu $\Delta_d=4$, chứng minh tương tự, ta cũng có điều mâu thuẫn.

Vậy ta phải có $\Delta_d \geq 5$.

Bài 6. Với các số nguyên $a, b, c, d$ thoả mãn $1 \leq a<b<c<d$, ký hiệu: $T(a, b, c, d)=[(x, y, z, t) \subset \mathbb{N}^* \mid 1 \leq x<y<z<t, x \leq a, y \leq b, z \leq c, t \leq d]$.

(a) Tính số phần tử của $T(1,4,6,7)$.

(b) Cho $a=1$ và $b \geq 4$. Gọi $d_1$ là số phần tử của $T(a, b, c, d)$ chứa 1 và không chứa $2 ; d_2$ là số phần tử chứa 1,2 và không chứa $3 ; d_3$ là số phần tử chứa $1,2,3$ và không chứa 4 . Chứng minh rằng $d_1 \geq 2 d_2-d_3$. Đẳng thức xảy ra khi nào ?

Lời giải. (a) Với $T(1,4,6,7)$, ta có $x \leq 1$ nên $x=1$. Khi đó ta có $2 \leq y \leq 4$ hay $y \in{2,3,4}$. Xét các khả năng sau:

  • Nếu $y=2$ thì $3 \leq z \leq 6$. Với mỗi giá trị của $z$, ta có thể thu được $7-z$ giá trị của $t$ nên ta có 10 bộ số.

  • Nếu $y=3$, tương tự ta có 6 bộ số.

  • Nếu $y=4$, tương tự ta có 3 bộ số.

Vậy có tất cả 19 bộ số trong $T(1,4,6,7)$.

(b) Đặt các tập hợp sau:

$\quad\quad\quad\quad\quad\quad\quad \left\{\begin{array}{l}T_1={(1, y, z, t) \mid 3 \leq y \leq b, y<z \leq c, z<t \leq d} \\ T_2={(1,2, z, t) \mid 4 \leq z \leq c, z<t \leq d} \\ T_3={(1,2,3, t) \mid 5 \leq t \leq d}\end{array}\right.$

Ta có $d_3=\left|T_3\right|=d-4$ và

$\quad\quad\quad\quad\quad\quad\quad\quad\quad\quad d_2=\sum_{z=4}^c(d-z)=(c-3) d+\frac{(c+4)(c-3)}{2}$

Tiếp theo ta tính $d_1=\left|T_1\right|$. Vì $b \geq 4$ nên $y \geq 3$. Xét các khả năng sau

  • Nếu $y=3$ thì $T(1,3, z, t)=d_2$.

  • Nếu $y=4$ thì $T(1,4, z, t)=\sum_{z=5}^c(d-z)=(c-4) d-\frac{(c+5)(c-4)}{2}$.

Từ đó $d_1 \geq d_2+(c-4) d-\frac{(c+5)(c-4)}{2}$. Do đó, kết hợp với việc tính được giá trị của $d_2$, khi cộng theo vế thì $d_1+d_3-2 d_2 \geq 0$.

Vậy $d_1 \geq 2 d_2-d_3$. Đẳng thức xảy ra khi và chỉ khi $b=4$.

Nhận xét. Ngoài lời giải khá “đại số” phía trên, có một lời giải khác cho ý sau của bài toán sử dụng song ánh:

  • Điểm mấu chốt là phân rã $T_1, T_2, T_3$ thành các nhóm thích hợp và thiết lập được đơn ánh giữa chúng. Với các tập $T_1, T_2, T_3$ định nghĩa như trên, ta viết $T_1$ thành $A \cup B \cup C$ có giao đôi một khác rỗng, trong đó

$\quad\quad\quad\quad\quad\quad \left\{\begin{array}{l}A={(1,3,4, t) \mid 5 \leq t \leq d} \\ B={(1,3, z, t) \mid 5 \leq z \leq c, z<t \leq d} \\ C={(1, y, z, t) \mid 4 \leq y \leq b, y<z \leq c, z<t \leq d}\end{array}\right.$

  • Dễ kiểm chứng rằng có song ánh từ $A$ vào $T_3$ nên $|A|=\left|T_3\right|=d_3$.

  • Xét $D={(1,4, z, t) \mid 5 \leq z \leq c, z<t \leq d}$. Dễ kiểm chứng rằng $D \subset C$ và có song ánh từ $D$ vào $B$ nên $|D|=|B|$.

  • Ta có $A \cup B={(1,3, z, t) \mid 4 \leq z \leq c, z<t \leq d}$. Dễ kiểm chứng rằng có song ánh từ $A \cup B$ vào $T_2$ nên $|A \cup B|=\left|T_2\right|=d_2$. Chú ý rằng $A \cap B=\varnothing$ nên $|A|+|B|=d_2$ hay $|B|=d_2-d_3$. Từ đó ta có:

$\quad\quad\quad\quad\quad d_1=|A|+|B|+|C| \geq|A|+|B|+|D|=d_3+2|B|$

Vậy $d_1 \geq d_3+2\left(d_2-d_3\right)=2 d_2-d_3$. Đẳng thức xảy ra khi và chỉ khi $b=4$.

Bài 7. Trong một hệ thống máy tính, một máy tính bất kỳ có kết nối trực tiếp với ít nhất $30 \%$ máy tính khác của hệ thống. Hệ thống này có một chương trình cảnh báo và ngăn chặn khá tốt, do đó khi một máy tính bị virus, nó chỉ có đủ thời gian lây cho các máy tính được kết nối trực tiếp với nó. Chứng minh rằng dù vậy, kẻ tấn công vẫn có thể chọn hai máy tính của hệ thống mà nếu thả virus vào hai máy đó, ít nhất $50 \%$ máy tính của hệ thống sẽ bị nhiễm virus.

Lời giải Trước hết ta chứng minh bổ đề sau: Xét một tập con $S$ bất kỳ của tập các máy tính $X$, khi đó tồn tại 1 máy tính của hệ thống kết nối trực tiếp với ít nhất $30 \%$ máy tính của $S$.

Thật vậy, xét các cặp $(s, x)$ với $s \in S, x \in X$ và $(s, x)$ kết nối trực tiếp với nhau. Khi đó, nếu tính theo $s$ thì số cặp như vậy sẽ không ít hơn $\frac{3}{10}|S||X|$. Do đó nếu tính theo $x$ thì sẽ phải tồn tại máy tính $x$ kết nối trực tiếp với ít nhất $\frac{3}{10}|S|$.

Quay trở lại bài toán,

Giả sử hệ thống có $n$ máy tính. Xét máy tính $A$ bất kỳ. Gọi $S$ là tập hợp các máy tính không kết nối trực tiếp với $A$. Nếu $S=\varnothing$ thì kết quả bài toán là hiển nhiên. Nếu $S \neq \varnothing$ thì theo bổ đề, tồn tại máy tính $B$ kết nối trực tiếp với ít nhất $30 \%$ máy tính trong $S$. Ta chứng minh hai máy tính $A$ và $B$ thỏa mãn yêu cầu bài toán.

Thật vậy, giả sử $A$ kết nối trực tiếp với $k$ máy tính khác. Khi đó, theo cách chọn, $A$ và $B$ sẽ kết nối trực tiếp với ít nhất

$\quad\quad\quad\quad\quad k+0,3(n-k)=0,7 k+0,3 n \geq 0,7 \cdot 0,3 n+0,3 n=0,51 n .$

Từ đây ta có được kết luận của bài toán.

Bài 8 . Cho tam giác $A B C$ nhọn. Đường tròn $(I)$ có tâm $I$ thuộc cạnh $B C$ và tiếp xúc với các cạnh $A B, A C$ lần lượt tại $E, F$. Lấy $M, N$ bên trong tứ giác $B C E F$ sao cho $E F N M$ nội tiếp $(I)$ và các đường thẳng $M N, E F, B C$ đồng quy. Gọi $M F$ cắt $N E$ tại $P, A P$ cắt $B C$ tại $D$.

(a) Chứng minh rằng $A, D, E, F$ cùng thuộc một đường tròn.

(b) Lấy trên các đường thẳng $B N, C M$ các điểm $H, K$ sao cho $\angle A C H=$ $\angle A B K=90^{\circ}$. Gọi $T$ là trung điểm $H K$. Chứng minh rằng $T B=T C$.

Lời giải. (a) Ta sẽ chứng minh rằng $A D \perp B C$. Gọi $X$ là điểm đồng quy của $E F, M N, B C$. Do $A E, A F$ tiếp xúc với $(I)$ nên $E F$ là đường đối cực của $A$ đối với (I). Ta có $X \in E F$ nên theo định lý La Hire, điểm $A$ sẽ nằm trên đường đối cực của $X$ đối với đường tròn $(I)$.

Lại có $P$ là giao điểm của $E N, F M$ nên $P$ nằm trên đường đối cực của $X$ đối với $(I)$. Vì thế nên $A P$ là đường đối cực của $X$ đối với $(I)$ hay $A P \perp B C$. Do đó

$\quad\quad\quad\quad\quad\quad\quad\quad\quad\quad \angle A D I=\angle A E I=\angle A F I=90^{\circ} .$

Vậy $A, D, E, F$ cùng thuộc một đường tròn.

(b) Gọi $S$ là giao điểm của $B N, C M$. Xét hai tam giác $P E F, S B C$ có $P E$ cắt $S B$ tại $N, P F$ cắt $S C$ tại $M, E F$ cắt $B C$ tại $X$ và $X, M, N$ thẳng hàng. Theo định lý Desargues thì $P S, E B, F C$ đồng quy. Mặt khác $E B$ cắt $F C$ tại $A$ nên $A, P, S$ thẳng hàng, dẫn đến $S \in A D$.

Tiếp theo ta sẽ chứng minh rằng $\angle B A K=\angle C A H$. Áp dụng định lý Ceva dạng lượng giác cho tam giác $A B C$ với:

  • Các đường thẳng $A D, B H, C K$ đồng quy:

$\quad\quad\quad\quad\quad\quad\quad\quad\quad\quad \frac{\sin \angle D A B}{\sin \angle D A C} \cdot \frac{\sin \angle H B C}{\sin \angle H B A} \cdot \frac{\sin \angle K C A}{\sin \angle K C B}=1$

  • Các đường thẳng $A H, B H, C H$ đồng quy:

$\quad\quad\quad\quad\quad\quad\quad\quad\quad\quad \frac{\sin \angle H A B}{\sin \angle H A C} \cdot \frac{\sin \angle H B C}{\sin \angle H B A} \cdot \frac{\sin \angle H C A}{\sin \angle H C B}=1$

  • Các đường thẳng $A K, B K, C K$ đồng quy:

$\quad\quad\quad\quad\quad\quad\quad\quad\quad\quad \frac{\sin \angle K A B}{\sin \angle K A C} \cdot \frac{\sin \angle K B C}{\sin \angle K B A} \cdot \frac{\sin \angle K C A}{\sin \angle K C B}=1$

Chú ý rằng do các góc vuông và góc bù nhau nên ta có

$\quad\quad\quad\quad\quad\quad\quad\quad\quad\quad \frac{\sin \angle H A C}{\sin \angle H A B}=\frac{\sin \angle K A B}{\sin \angle K A C}$

Từ đó sử dụng công thức cộng cho mẫu thức và biến đổi thì:

$\quad\quad\quad\quad\quad\quad\quad\quad\quad\quad \tan \angle H A C=\tan \angle K A B$

Dẫn đến $\angle H A C=\angle K A B$. Cuối cùng, ta sẽ chứng minh $T B=T C$.

Gọi $U, V$ lần lượt là trung điểm của các đoạn $A K, A H$. Ta có:

$\quad\quad\quad\quad\quad\quad\quad\quad\quad\quad U B=\frac{A K}{2}=V T, U T=\frac{A H}{2}=V C .$

Đồng thời, ta cũng có:

$\quad\quad\quad\quad\quad\quad \angle B U T=\angle B U A-\angle A U T=\angle A V C-\angle A V T=\angle T V C$

Do đó $\Delta B U T=\Delta T V C$ (c.g.c), vậy nên $T B=T C$.

Nhận xét. Để chứng minh $\angle H A C=\angle K A B$, cũng là mấu chốt của lời giải trên, ta có thể dùng bổ đề sau:

Cho tam giác $A B C$ có hai điểm $P, Q$ sao cho $A P, A Q$ đẳng giác trong góc $A$. Gọi $X$ là giao điểm của $B P, C Q$ và $Y$ là giao điểm của $B Q, C P$. Khi đó, ta cũng có $A X, A Y$ đẳng giác trong góc $A$.

 

 

 

 

 

 

 

 

 

 

 

 

 

 

 

 

 

 

 

 

 

 

 

 

 

 

 

 

 

 

 

 

 

 

 

 

 

 

 

 

 

 

 

 

 

 

 

 

 

 

 

 

 

 

 

 

 

 

 

 

 

Đề thi và đáp án kì thi chọn đội tuyển thi Quốc gia trường Phổ thông Năng khiếu năm học 2010 – 2011

ĐỀ THI

Ngày thi thứ nhất

Bài 1. Giải hệ phương trình sau:

$\quad\quad\quad\quad\quad\quad\quad\quad\quad\quad \left\{\begin{array}{l}\frac{5(x+y)}{x+y+6 x y}+\frac{6(x+z)}{x+z+5 x z}=4 \\ \frac{6(y+z)}{z+y+4 z y}+\frac{4(x+y)}{x+y+6 x y}=5 \\ \frac{4(x+z)}{x+y+5 x z}+\frac{5(y+z)}{y+z+4 y z}=6\end{array}\right.$

Bài 2. Tìm tất cả các hàm số $f: \mathbb{R} \rightarrow \mathbb{R}$ thỏa mãn:

$\quad\quad\quad\quad\quad\quad\quad\quad\quad\quad  f(|x|+y+f(y+f(y)))=3 y+|f(x)|, \forall x, y \in \mathbb{R}$

Bài 3. Cho $p$ là số nguyên tố lẻ và $n=2 p+r$ với $r \in{0,1,2, \ldots, p-1}$. Đặt $X={1,2, \ldots, n}$. Ánh xạ $f: X \rightarrow X$ được gọi là có tính chất $\mathcal{P}$ nếu $f$ không phải là ánh xạ đồng nhất và $f(f(\ldots(f(k)) \ldots)$ ) $=k$ (ánh xạ hợp $p$ lần) với mọi $k \in X$.

Đặt $A_f={k \in X \mid f(k)=k}$.

a) Chứng minh rằng nếu $f$ có tính chất $\mathcal{P}$ thì $\left|A_f\right| \equiv r(\bmod p)$.

b) Gọi $d$ là số các ánh xạ có tính chất $\mathcal{P}$. Chứng minh rằng $d$ không là ước của $n$ !.

(Kí hiệu $|A|$ chỉ số lượng các phần tử của tập hợp $A$.)

Bài 4. Cho tam giác $A B C$ nội tiếp đường tròn $(O)$ có $A$ cố định và $B, C$ thay đổi trên $(O)$ sao cho $B C$ luôn song song với một đường thẳng cố định. Các tiếp tuyến của $(O)$ tại $B$ và $C$ cắt nhau tại $K$. Gọi $M$ là trung điểm của $B C, N$ là giao điểm của $A M$ với $(O)$. Chứng minh rằng đường thẳng $K N$ luôn đi qua một điểm cố định.

Ngày thi thứ hai

Bài 5. Chứng minh rằng nếu $a, b, c$ là độ dài ba cạnh của một tam giác thì:

$\quad\quad\quad\quad\quad\quad\quad\quad\quad\quad (2 a+2 b-c)(2 b+2 c-a)(2 c+2 a-b)>25 a b c$

Bài 6. Cho dãy số $\left(u_n\right)$ thoả mãn $u_1=\sqrt{2}$ và $u_{n+1}=\frac{2 u_n^2+5 u_n+5}{2 u_n+4}, \forall n \geq 1$. Tìm $\lim \frac{u_n^2-3 u_n+5}{3 n^2+4 n-1}$.

Bài 7. Xét số tự nhiên $n>1$. Bắt đầu từ bộ số $1,2, \ldots, 2 n-1,2 n$, ta thực hiện phép biến đổi sau: Chọn hai số $a, b$ sao cho $a-b>1$, xoá hai số này và thay thế bởi hai số $a-1, b+1$. Với bộ số mới, ta lại tiếp tục thực hiện phép biến đổi tương tự’

a) Chứng minh rằng ta sẽ đạt đến trạng thái dừng, tức là không thể tiếp tục thực hiện phép biến đổi như vậy được nữa.

b) Gọi $k$ là số lần phép biến đổi cần thực hiện để đạt đến trạng thái dừng. Tìm giá trị nhỏ nhất và lớn nhất của $k$.

Bài 8. Cho đường tròn $\left(\gamma_1\right)$ đường kính $A B$ và đường tròn $\left(\gamma_2\right)$ tâm $A$ cắt $\left(\gamma_1\right)$ tại $C, D$. Điểm $M$ thay đổi trên cung $C D$ (nằm bên trong $\left(\gamma_1\right)$ ) của $\left(\gamma_2\right)$. Gọi $B M$ cắt $\left(\gamma_1\right)$ tại $N$ khác $M$ và $B$. Tìm giá trị nhỏ nhất của $\frac{N D+N C}{M N}$.

 

LỜI GIẢI

Ngày thi thứ nhất

Bài 1. Giải hệ phương trình

$\quad\quad\quad\quad\quad\quad\quad\quad\quad\quad \left\{\begin{array}{l}\frac{5(x+y)}{x+y+6 x y}+\frac{6(x+z)}{x+z+5 x z}=4 \\ \frac{6(y+z)}{z+y+4 z y}+\frac{4(x+y)}{x+y+6 x y}=5 \\ \frac{4(x+z)}{x+y+5 x z}+\frac{5(y+z)}{y+z+4 y z}=6\end{array} .\right.$

Lời giải. Đặt $u=\frac{x+y}{x+y+6 x y}, v=\frac{y+z}{y+z+4 y z}, w=\frac{z+x}{z+x+5 z x}$ thì ta có hệ

$\quad\quad\quad\quad\quad\quad\quad\quad\quad\quad \left\{\begin{array} { l }{ 5 u + 6 w = 4 } \\ { 6 v + 4 u = 5 } \\ { 4 w + 5 v = 6 }\end{array} \Leftrightarrow \left\{\begin{array}{l}8 u=1 \\ 4 v=3 \\ 16 w=9\end{array} .\right.\right.$

Suy ra

$\quad\quad\quad\quad\quad\quad\quad\quad\quad\quad \left\{\begin{array} { l }{ 7 ( x + y ) = 6 x y } \\ { 3 ( y + z ) = 1 2 y z } \\ { 7 ( z + x ) = 4 5 z x }\end{array} \Leftrightarrow \left\{\begin{array}{l}a+b=\frac{6}{7} \\ b+c=12 \\ c+a=\frac{45}{7}\end{array}\right.\right.$

trong đó $a=\frac{1}{x}, b=\frac{1}{y}, c=\frac{1}{z}$. Giải hệ trên, ta thu được $a=-\frac{33}{14}, b=\frac{45}{14}, c=\frac{123}{14}$ nên $(x, y, z)=\left(-\frac{14}{33}, \frac{14}{45}, \frac{14}{123}\right)$.

Bài 2. Tìm tất cả các hàm số $f: \mathbb{R} \rightarrow \mathbb{R}$ thỏa mãn:

$\quad\quad\quad\quad\quad\quad f(|x|+y+f(y+f(y)))=3 y+|f(x)|, \forall x, y \in \mathbb{R}$

Lời giải. Dễ thấy $f$ toàn ánh. Giả sử $f(a)=0$ và thay $x=0, y=a$, ta có

$\quad\quad\quad\quad\quad\quad\quad\quad\quad\quad 0=3 a+|f(0)|$

Suy ra $a$ tồn tại duy nhất và $a=-\frac{1}{3}|f(0)| \leq 0$. Lại thay $x=y=a$, ta có $f(0)=3 a \leq 0$. Lại thay $x=-a, y=a$ thì chú ý rằng $|-a|+a=0$, ta có $f(0)=3 a+|f(-a)|$ nên $f(-a)=0$, điều này kéo theo $a=-a$ hay $a=0$ (do tính duy nhất ở trên).

Thay $y=0$ thì $f(|x|)=|f(x)|$ nên $f(x) \geq 0, \forall x \geq 0$. Xét $x>0$ và $y=-\frac{f(x)}{3}$, ta có $f\left(x-\frac{f(x)}{3}+f\left(-\frac{f(x)}{3}+f\left(-\frac{f(x)}{3}\right)\right)\right)=0$ nên

$\quad\quad\quad\quad\quad\quad\quad\quad\quad\quad -\frac{f(x)}{3}+f\left(-\frac{f(x)}{3}+f\left(-\frac{f(x)}{3}\right)\right)=-x$

với mọi $x>0$. Trong đề bài, thay $x=0$ thì $f(y+f(y+f(y)))=3 y$. Thay $y \rightarrow-\frac{f(x)}{3}$ thì $f\left(-\frac{f(x)}{3}+f\left(-\frac{f(x)}{3}+f\left(-\frac{f(x)}{3}\right)\right)\right)=-f(x)$. So sánh hai đẳng thức trên, ta có $f(-x)=-f(x), \forall x>0$ nên $f$ là hàm số lẻ.

Từ tính chất hàm số lẻ, ta có $f\left(\frac{f(x)}{3}+f\left(\frac{f(x)}{3}+f\left(\frac{f(x)}{3}\right)\right)\right)=f(x)$ với mọi $x>0$. Trong đề bài, xét $x \geq 0$ và $y \rightarrow \frac{f(y)}{3}$, ta có

$\quad\quad\quad\quad\quad\quad\quad\quad\quad\quad f\left(x+\frac{f(y)}{3}+f\left(\frac{f(y)}{3}+f\left(\frac{f(y)}{3}\right)\right)\right)=f(y)+f(x)$

hay $f(x+y)=f(x)+f(y)$ với mọi $x, y>0$. Vì $f$ cộng tính trên $\mathbb{R}^{+}$nên ta có $f(x)=a x, \forall x>0$. Lại do tính chất hàm lẻ, ta suy ra $f(x)=a x, \forall x \in \mathbb{R}$. Thay vào đề bài, ta có $a=1$.

Vậy tất cả các hàm số cần tìm là $f(x)=x$.

Bài 3. Cho $p$ là số nguyên tố lẻ và $n=2 p+r$ với $r \in{0,1,2, \ldots, p-1}$. Đặt $X={1,2, \ldots, n}$. Ánh xạ $f: X \rightarrow X$ được gọi là có tính chất $\mathcal{P}$ nếu $f$ không phải là ánh xạ đồng nhất và $f(f(\ldots(f(k)) \ldots)$ ) $=k$ (ánh xạ hợp $p$ lần) với mọi $k \in X$.

Đặt $A_f={k \in X \mid f(k)=k}$.

a) Chứng minh rằng nếu $f$ có tính chất $\mathcal{P}$ thì $\left|A_f\right| \equiv r(\bmod p)$.

b) Gọi $d$ là số các ánh xạ có tính chất $\mathcal{P}$. Chứng minh rằng $d$ không là ước của $n$ !.

(Kí hiệu $|A|$ chỉ số lượng các phần tử của tập hợp $A$.)

Lời giải. a) Ta có

$\quad\quad\quad\quad\quad\quad\quad\quad\quad\quad \left|A_f\right| \equiv r \quad(\bmod p) \Leftrightarrow\left|X \backslash A_f\right| \text { chia hết cho } p \text {. }$

Điều này tương đương số phần tử của tập hợp $B={k \in X \mid f(k) \neq k}$ là bội của $p$. Đặt $f_m(x)$ là ánh xạ hợp $m$ lần. Xét $x \in B$ thì cũng có các số $f(x), f_2(x), \ldots, f_{p-1}(x) \in$ B. Thật vậy,

Giả sử tồn tại $1<m<p$ sao cho $f_m(x)=x$ với số $x \in B$ nào đó, ta chọn $m$ là số nhỏ nhất như thế. Vì $p$ nguyên tố lẻ nên $p$ không chia hết cho $m$. Do vậy tồn tại số $t$ sao cho $0<p-t m<m$. Lại có

$\quad\quad\quad\quad\quad\quad\quad\quad\quad\quad f_m(x)=x \Rightarrow f_{t m}(x)=x \Rightarrow f_{p-t m}(x)=f_p(x)=x$

(mâu thuẫn với tính nhỏ nhất của $m$ ). Vì thế nên với mọi $m$ mà $1<m<p$ thì $f_m(x) \neq x$. Từ đó suy ra với mọi $1<k<l<p$ thì $f_k(x) \neq f_l(x)$, tức là $x, f(x), f_2(x), \ldots, f_{p-1}(x)$ là $p$ số khác nhau thuộc $B$.

Xét số $y \in B$ và $y$ khác tất cả $p$ số ở trên. Khi đó, ta cũng sẽ có $y$ sinh ra một bộ $p$ số phân biệt mới. Giả sử rằng có $f_i(x)=f_j(y)$ với $i<j$ nào đó thì sẽ có $f_{p+i-j}(x)=f_p(y)=y$, mâu thuẫn. Suy ra trong $B$ sẽ có 1 hoặc 2 bộ $p$ số rời nhau, chứng tỏ rằng số phần tử của $B$ chia hết cho $p$. Suy ra điều phải chứng minh.

(b) Từ đây ta thấy rằng để đếm số ánh xạ $f$ có tính chất $\mathcal{P}$, trước hết, ta chọn ra $r$ hoặc $p+r$ vị trí cố định. Ta xét hai trường hợp như sau:

  1. Nếu $\left|A_f\right|=p+r$ thì có $C_n^{p+r}$ cách chọn ra các số này, còn lại $p$ số thì $f$ phải là song ánh trên tập con đó. Do đó trong trường hợp này có $p ! C_n^{p+r}$ cách.
  2. Nếu $\left|A_f\right|=r$ thì tương tự trên, ta cũng đếm được $(p !)^2 C_n^r C_{2 p}^p$.

Từ đó suy ra số ánh xạ tính chất $\mathcal{P}$ là

$\quad\quad\quad\quad\quad\quad\quad\quad\quad\quad d=p ! C_n^{p+r}+(p !)^2 C_n^r C_{2 p}^p$

Ta sẽ chứng minh số này không là ước của $n$ !. Ta viết số $d$ dưới dạng khai triển

$\quad\quad\quad\quad\quad\quad\quad\quad\quad\quad d=p ! \frac{n !}{(p+r) ! p !}+(p !)^2 \frac{n !}{r !(2 p) !} \cdot \frac{(2 p) !}{(p !)^2}=\frac{n !}{(p+r) !}+\frac{n !}{r !} .$

Đặt $(p+r) !=k \cdot(r !)^2$ với $k=\frac{(p+r) !}{(r !)^2}=\frac{p !}{r !} \cdot \frac{(p+r) !}{p ! r !}=\frac{p !}{r !} C_{p+r}^r \in \mathbb{Z}$. Khi đó, ta viết lại

$\quad\quad\quad\quad\quad\quad\quad\quad\quad\quad \frac{n !}{d}=\frac{r !(p+r) !}{r !+(p+r) !}=\frac{k \cdot(r !)^3}{(1+k \cdot r !) \cdot r !}=\frac{k \cdot(r !)^2}{k \cdot r !+1} .$

Dễ thấy số này không thể nguyên vì $k \cdot r !+1$ nguyên tố cùng nhau với $k \cdot(r !)^2$. Từ đó ta có $d$ không là ước của $n$ !.

Nhận xét. Bài này nếu tổng quát $n=k q+r$ thì kết quả câu a vẫn đúng. Tuy nhiên, câu b biến đổi sẽ phức tạp hơn nhiều.

Bài 4. Cho tam giác $A B C$ nội tiếp đường tròn $(O)$ có $A$ cố định và $B, C$ thay đổi trên $(O)$ sao cho $B C$ luôn song song với một đường thẳng cố định. Các tiếp tuyến của $(O)$ tại $B$ và $C$ cắt nhau tại $K$. Gọi $M$ là trung điểm của $B C, N$ là giao điểm của $A M$ với $(O)$. Chứng minh rằng đường thẳng $K N$ luôn đi qua một điểm cố định.

Lời giải. Giả sử $K N$ cắt $(O)$ tại $I$ thì tứ giác $B N C I$ điều hòa.

Do đó $A(B C, N I)=-1$, mà $A N$ chia đôi $B C$ nên $A I | B C$, tức là $A I$ có phương cố định. Từ đó ta thấy $I$ là điểm cố định cần tìm.

Ngày thi thứ hai

Bài 5. Chứng minh rằng nếu $a, b, c$ là độ dài ba cạnh của một tam giác thì:

$\quad\quad\quad\quad\quad\quad\quad\quad\quad\quad (2 a+2 b-c)(2 b+2 c-a)(2 c+2 a-b)>25 a b c .$

Lời giải. Đặt $a+b-c=x, b+c-a=y, c+a-b=z$ thì $x, y, z>0$. Ta đưa về bất đẳng thức

$\quad\quad\quad\quad\quad\quad\quad\quad\quad\quad \left(4 \cdot \frac{x}{y+z}+1\right)\left(4 \cdot \frac{y}{z+x}+1\right)\left(4 \cdot \frac{z}{x+y}+1\right)>25 .$

Không mất tính tổng quát, giả sử $0<x \leq y \leq z$. Đặt $S=x+y+z$. Ta đưa về

$\quad\quad\quad\quad\quad\quad\quad\quad\quad (S+3 x)(S+3 y)(S+3 z)>25(S-x)(S-y)(S-z) .$

Khai triển và rút gọn, ta được

$\quad\quad\quad\quad\quad\quad\quad\quad\quad\quad\quad\quad S^3-4 S(x y+y z+z x)+13 x y z>0 .$

Chú ý rằng

$\quad\quad\quad\quad S^3-4 S(x y+y z+z x)=S\left(S^2-4(x y+y z+z x)\right)=S\left((x+y-z)^2-4 x y\right)$

nên ta đưa về $S(x+y-z)^2+x y(13 z-4 S)>0$. Bất đẳng thức cuối đúng vì $13 c-4 S=9 z-4(x+y)>0$.

Bài 6. Cho dãy số $\left(u_n\right)$ thoả mãn $u_1=\sqrt{2}$ và $u_{n+1}=\frac{2 u_n^2+5 u_n+5}{2 u_n+4}, \forall n \geq 1$. Tìm $\lim \frac{u_n^2-3 u_n+5}{3 n^2+4 n-1}$.

Lời giải. Ta thấy rằng $u_n>0, \forall n$ và $u_{n+1}-u_n=\frac{u_n+5}{2\left(u_n+2\right)}>0$ nên dãy tăng. Giả sử dãy bị chặn trên thì nó hội tụ về $L>0$, suy ra

$\quad\quad\quad\quad\quad\quad\quad\quad\quad L=\frac{2 L^2+5 L+5}{2 L+4} \Leftrightarrow L=-5,$

vô lý. Suy ra $\lim _{n \rightarrow+\infty} u_n=+\infty$. Từ đó, ta được

nên theo định lý Stolz, ta suy ra $\lim _{n \rightarrow+\infty} \frac{u_n}{n}=\frac{1}{2}$ và $\lim _{n \rightarrow+\infty} \frac{u_n}{n^2}=0$. Do đó, trong biểu thức cần tính giới hạn, chia tử và mẫu cho $n^2$ rồi áp dụng kết quả trên, ta có

$\quad\quad\quad\quad\quad \lim _{n \rightarrow+\infty} \frac{u_n^2-3 u_n+5}{3 n^2+4 n-1}=\lim _{n \rightarrow+\infty} \frac{\left(\frac{u_n}{n}\right)^2-\frac{3 u_n-5}{n^2}}{3+\frac{4}{n}-\frac{1}{n^2}}=\left(\frac{1}{2}\right)^2 \cdot \frac{1}{3}=\frac{1}{12}$

Bài 7. Xét số tự nhiên $n>1$. Bắt đầu từ bộ số $1,2, \ldots, 2 n-1,2 n$, ta thực hiện phép biến đổi sau: Chọn hai số $a, b$ sao cho $a-b>1$, xoá hai số này và thay thế bởi hai số $a-1, b+1$. Với bộ số mới, ta lại tiếp tục thực hiện phép biến đổi tương tự.

a) Chứng minh rằng ta sẽ đạt đến trạng thái dừng, tức là không thể tiếp tục thực hiện phép biến đổi như vậy được nữa.

b) Gọi $k$ là số lần phép biến đổi cần thực hiện để đạt đến trạng thái dừng. Tìm giá trị nhỏ nhất và lớn nhất của $k$.

Lời giải. (a) Xét đại lượng $S$ là tổng bình phương các số thu được sau mỗi thao tác biến đổi.

Ta thấy rằng từ $(a, b)$ với $a-b>1$, ta đưa về bộ $(a-1, b+1)$ thì tổng trên thay đổi một lượng là $a^2+b^2-(a-1)^2-(b+1)^2=2(a+b-1)>0$. Do đó, tổng $S$ giảm ngặt, và rõ ràng $S$ phải luôn dương nên thao tác trên chỉ thực hiện được trong hữu hạn lần.

(b) Rõ ràng tổng trên không đổi khi không còn cặp số $a, b$ nào mà $a-b>1$. Điều này đồng nghĩa với việc các số thu được trong trạng thái cuối chỉ nhận hai giá trị liên tiếp nào đó. Ta thấy rằng tổng các số đã cho luôn không đổi và là $1+2+\cdots+2 n=n(2 n+1)$

Giả sử cuối cùng, ta có $x$ số $m$ và $y$ số $m+1$ thì

$\quad\quad\quad\quad\quad\quad\quad\quad\quad\quad\quad\quad\left\{\begin{array}{l}x+y=2 n \\ m x+(m+1) y=n(2 n+1)\end{array}\right.$

Suy ra $2 m n+y=2 n^2+n \Rightarrow n \mid y$. Tuy nhiên, nếu $y \in{0,2 n}$ thì vô lý vì vế phải không chia hết cho $2 n$. Do đó $x=y=n$ và $m=n$, tức là ở trạng thái cuối, ta còn $n$ số $n$ và $n+1$.

  • Tổng bình phương của chúng là $S=n \cdot n^2+n \cdot(n+1)^2=n\left(2 n^2+2 n+1\right)$.
  • Tổng bình phương ban đầu là $S_0=1^2+2^2+\cdots+(2 n)^2=\frac{n(2 n+1)(4 n+1)}{3}$.

Suy ra $S_0-S=\frac{2}{3}\left(n^3-n\right)$.

(b) Để thực hiện được nhiều lần nhất thì giá trị giảm đi ở mỗi lần phải ít nhất. Theo câu a) thì giá trị đó sẽ là $2(a+b-1) \geq 2$.

Suy ra số lần nhiều nhất sẽ là $\frac{1}{3}\left(n^3-n\right)$. Để thực hiện được điều này, ta sẽ cố gắng trong mỗi thao tác tạo ra nhiều giá trị nhất có thể và đồng thời làm giảm số lượng các giá trị ở hai biên đi. Từ đó ta được $k_{\max }=\frac{1}{3}\left(n^3-n\right)$.

Để thực hiện được ít lần nhất, ta sử dụng ý tưởng tham lam, mỗi lần, ta sẽ chọn các cặp số nằm về hai phía của $n, n+1$. Khi đó, giá trị của các số $1,2, \ldots, n-1$ sẽ dần dần được tăng lên, trong khi giá trị của các số $n+2, n+3, \ldots, 2 n$ dần dần sẽ giảm đi. Tổng khoảng cách từ các số nhỏ hơn $n$ đến $n$ là $1+2+\cdots+n-1=\frac{n(n-1)}{2}$. Tương tự thì tổng khoảng cách các số lớn hơn $n+1$ đến $n+1$ cũng là $\frac{n(n-1)}{2}$. Ta thấy mỗi lần thao tác thì các số này sẽ thu hẹp khoảng cách đúng 2 đơn vị nên số lần thao tác tối thiểu phải là $\frac{1}{2}\left(\frac{n(n-1)}{2}+\frac{n(n-1)}{2}\right)=\frac{n(n-1)}{2}$.

Để đạt được giá trị này, mỗi lần, ta chỉ cần chọn các cặp số có dạng $(t, 2 n+1-t)$ với $1 \leq t \leq n-1$ là được. Suy ra $k_{\min }=\frac{n(n-1)}{2}$.

Bài 8. Cho đường tròn $\left(\gamma_1\right)$ đường kính $A B$ và đường tròn $\left(\gamma_2\right)$ tâm $A$ cắt $\left(\gamma_1\right)$ tại $C, D$. Điểm $M$ thay đổi trên cung $C D$ (nằm bên trong $\left(\gamma_1\right)$ ) của $\left(\gamma_2\right)$. Gọi $B M$ cắt $\left(\gamma_1\right)$ tại $N$ khác $M$ và $B$. Tìm giá trị nhỏ nhất của $\frac{N D+N C}{M N}$.

Lời giải. Theo định lý Ptolemy cho tứ giác $B C N D$ nội tiếp trong $\gamma_1$ thì

$\quad\quad\quad\quad\quad\quad\quad\quad\quad\quad\quad B C \cdot N D+B D \cdot N C=B N \cdot C D .$

Vì $A C=A D$ nên $B C=B D=m$ và $C D=n$ là các giá trị cố định.

Ta có

$\quad\quad\quad\quad\quad\quad\quad\quad m(N C+N D)=n \cdot B N \Rightarrow N C+N D=\frac{n}{m} \cdot B N .$

Suy ra $\frac{N C+N D}{M N}=\frac{n}{m} \cdot \frac{B N}{M N}$. Ta đưa về tìm giá trị nhỏ nhất của $\frac{B N}{M N}$. Xét phương tích từ $B$ đến $\gamma_2$ thì $B M \cdot B N=B K \cdot B A=c$ là hằng số nên$(B N-M N) B N=c$. Do đó $\frac{M N}{B N}=1-\frac{c}{B N^2}$ nên

$\quad\quad\quad\quad\quad\quad\frac{B N}{M N} \min \Leftrightarrow \frac{M N}{B N} \max \Leftrightarrow \frac{c}{B N^2} \min \Leftrightarrow B N^2 \max .$

Dễ thấy $\max B N=A B$, xảy ra khi $N \equiv A$ hay $M \equiv K$. Khi đó

$\quad\quad\quad\quad\quad\quad\quad\quad\quad\quad\quad\frac{N C+N D}{M N}=\frac{A C+A D}{A K}=2$

chính là giá trị nhỏ nhất cần tìm.

 

 

 

 

 

 

 

 

 

 

 

 

 

 

 

 

 

 

 

 

 

 

 

 

 

 

 

 

 

 

 

 

 

 

 

 

 

 

 

 

 

 

 

 

 

 

 

 

 

 

 

 

 

 

 

 

 

 

 

 

 

 

 

 

 

 

 

 

 

 

 

 

 

 

 

 

 

 

 

 

 

 

 

 

 

 

 

 

 

 

 

 

 

 

 

 

 

Đề thi và đáp án kì thi chọn đội tuyển thi Quốc gia trường Phổ thông Năng khiếu năm học 2009 – 2010

ĐỀ THI

Ngày thi thứ nhất

Bài 1. Cho $a, b, c$ là các số thực để đa thức $P(x)=x^4+ax^3+b x^2+c x+1$ có ít nhất một nghiệm thực. Tìm tất cả các bộ $(a, b, c)$ để $a^2+b^2+c^2$ đạt giá trị nhỏ nhất.

Bài 2. Cho $A={1,2, \ldots, 2 n}$. Một tập con của $A$ được gọi là tốt nếu như có đúng 2 phần tử $x, y$ và đồng thời $|x-y| \in[1, n]$. Tìm số các tập hợp $[A_1, A_2, \ldots, A_n]$ để $A_i$ là tập con tốt của $A$ với $1 \leq i \leq n$ và $\bigcup_{i=1}^n A_i=A$.

Bài 3. Tìm tất cả các hàm số $f: \mathbb{N}^* \rightarrow \mathbb{N}^*$ thoả mãn các điều kiện sau:

$\quad\quad(i) f $ là hàm số tăng thật sự trên $\mathbb{N}^*$.

$\quad\quad(ii) f(f(n))=4 n+9 \forall n \in \mathbb{N}^*$.

$\quad\quad(iii) f(f(n)-n)=2 n+9 \forall n \in \mathbb{N}^*$.

Bài 4. Cho đường tròn tâm $O$ và dây cung $A B$ cố định khác đường kính. Một điểm $P$ thay đổi trên cung lớn $A B$. Gọi $I$ là trung điểm của $A B$. Lấy các điểm $M, N$ trên các tia $P A, P B$ sao cho $\angle P M I=\angle P N I=\angle A P B$.

(a) Chứng minh rằng đường cao từ $P$ của tam giác $P M N$ luôn đi qua một điểm cố định.

(b) Chứng minh rằng đường thẳng Euler của tam giác $P M N$ luôn đi qua một điểm cố định.

Ngày thi thứ hai

Bài 5. Cho $a, b, c$ là các số thực dương. Giải hệ phương trình sau:

$\quad\quad\quad\quad\quad\quad\quad\quad\quad\quad\left\{\begin{array}{l}a x-a b y+\frac{1}{x y}=b c^2 \\ a b z-b c^2 x+\frac{1}{x z}=a . \\ b c^2 y-a z+\frac{1}{y z}=a b\end{array}\right.$

Bài 6. Cho dãy số $\left(a_n\right)$ xác định bởi $a_1=a, a_{n+1}=\left(a_1+\cdots+a_n-2\right)^2 \forall n \in \mathbb{N}^*$. Đặt $S_n=a_1+a_2+\cdots+a_n$. Tìm tất cả các giá trị $a$ để dãy số $\left(S_n\right)$ hội tụ.

Bài 7. Tìm tất cả các số nguyên dương $k$ để phương trình sau có nghiệm nguyên dương $(x, y)$ :

$$\quad\quad x^2+y^2+x+y=k x y$

Bài 8. Cho tam giác $A B C$ nội tiếp đường tròn $(O)$. Gọi $I, I_1, I_2, I_3$ lần lượt là tâm đường tròn nội tiếp và tâm đường tròn bàng tiếp các đỉnh $A, B, C$ của tam giác $A B C$. Dường tròn ngoại tiếp tam giác $I I_2 I_3$ cắt $(O)$ tại hai điểm $M_1, N_1$. Gọi $J_1$ là giao điểm của $A I$ và $(O)$. Ký hiệu $d_1$ là đường thẳng qua $J_1$ và vuông góc với $M_1 N_1$. Xác định các đường thẳng $d_2, d_3$ tương tự. Chứng minh rằng $d_1, d_2, d_3$ dồng quy.

 

LỜI GIẢI

Ngày thi thứ nhất

Bài 1. Cho $a, b, c$ là các số thực để đa thức $P(x)=x^4+a x+3+b x^2+c x+1$ có ít nhất một nghiệm thực. Tìm tất cả các bộ $(a, b, c)$ để $a^2+b^2+c^2$ đạt giá trị nhỏ nhất.

Lời giải. Gọi $x_0$ là một nghiệm của $P(x)$ (dễ thấy $x_0 \neq 0$ ). Do $P\left(x_0\right)=0$ nên ta có

$\quad\quad\quad\quad\quad\quad\quad\quad\quad\quad-\left(x_0^4+1\right)=a x_0^3+b x_0^2+c x_0 .$

Sử dụng bất đẳng thức Cauchy-Schwarz, ta có

$\quad\quad\quad\quad\quad\left(x_0^4+1\right)^2=\left(a x_0^3+b x_0^2+c x_0\right)^2 \leq\left(a^2+b^2+c^2\right)\left(x_0^6+x_0^4+x_0^2\right) .$

Đặt $t=x_0^2>0$. Từ đánh giá trên, ta suy ra

$\quad\quad\quad\quad\quad\quad\quad\quad\quad\quad a^2+b^2+c^2 \geq \frac{\left(t^2+1\right)^2}{t^3+t^2+t}=\frac{\left(t^2+1\right)^2}{t\left(t^2+t+1\right)}$

Mà theo bất đẳng thức AM-GM thì

$\quad\quad\quad\quad\quad\quad\quad\quad\quad\quad t \leq \frac{t^2+1}{2} \text { và } t^2+t+1 \leq t^2+\frac{t^2+1}{2}=\frac{3}{2}\left(t^2+1\right) .$

Do đó

$\quad\quad\quad\quad\quad\quad\quad\quad\quad\quad\frac{\left(t^2+1\right)^2}{t\left(t^2+t+1\right)} \geq \frac{4}{3}, \text { nên } a^2+b^2+c^2 \geq \frac{4}{3}$

Đẳng thức xảy ra khi và chỉ khi

$\quad\quad\quad\quad\quad\quad\quad\quad\quad\quad\left\{\begin{array}{l}x_0^4+a x_0^3+b x_0^2+c x_0+1=0 \\ x_0^2=1 \\ \frac{a}{x_0^3}=\frac{b}{x_0^2}=\frac{c}{x_0}\end{array}\right.$

Giải hệ này, ta thu được $a=b=c=-\frac{2}{3}$ hoặc $a=-b=c=\frac{2}{3}$.

Bài 2. Cho $A=[1,2, \ldots, 2 n]$. Một tập con của $A$ được gọi là tốt nếu như có đúng 2 phần tử $x, y$ và đồng thời $|x-y| \in[1, n]$. Tìm số các tập hợp $[A_1, A_2, \ldots, A_n]$ để $A_i$ là tập con tốt của $A$ với $1 \leq i \leq n$ và $\bigcup_{i=1}^n A_i=A$.

Lời giải . Gọi $u_n, n \in{1,2, \ldots, n}$ là số các tập hợp $[A_1, A_2, \ldots, A_n]$ thỏa mãn yêu cầu đề bài, đồng thời hai phần tử $n$ và $n+1$ không đi cùng nhau trong bất kì tập $A_i$ nào. Ta chia các số $1,2, \ldots, 2 n$ vào một bảng $2 \times n$ như sau

$\quad\quad\quad\quad\quad\quad\quad\quad\quad\quad\quad\quad\quad\quad\begin{array}{|c|c|c|c|}\hline 1 & 2 & \ldots & n \\ \hline n+1 & n+2 & \ldots & 2 n \\ \hline\end{array}$

Khi đó, mỗi cách chọn được liệt kê trong $u_n$ tương ứng với một cách chọn từ bảng trên các cặp gồm hai số ở cùng một cột hoặc hai số liên tiếp nhau trên cùng một hàng. Xét $u_{n+1}$, vì phần tử $2(n+1)$ chỉ có thể đi cùng với $n+1$ hoặc $2 n+1$ trong cùng một tập $A_i$ nào đó nên ta xét hai khả năng sau.

  • $2(n+1)$ và $n+1$ cùng thuộc một tập $A_i$, giả sử là $A_{n+1}$.

$\quad\quad\quad\quad\quad\quad\quad\quad\quad\quad\quad\quad\begin{array}{|c|c|l|c|}\hline 1 & 2 & \ldots & n+1 \\ \hline n+1 & n+2 & \ldots & 2(n+1) \\ \hline\end{array}$

Lúc này, mỗi cách chọn một bộ $[A_1, A_2, \ldots, A_n]$ ứng với một cách chọn các cặp số gồm các số ở cùng một cột hoặc ở cạnh nhau trong cùng một hàng từ một bảng $2 \times n$. Theo định nghĩa của ta số cách chọn như thế là $u_n$. Vậy trong trường hợp này có $u_n$ cách chọn.

  • $2(n+1)$ và $2 n+1$ cùng thuộc một tập $A_i$, giả sử đó là $A_{n+1}$.

$\quad\quad\quad\quad\quad\quad\quad\quad\quad\quad\quad\quad\begin{array}{|c|c|l|c|c|}\hline 1 & 2 & \ldots & n & n+1 \\ \hline n+1 & n+2 & \ldots & 2 n+1 & 2(n+1) \\ \hline\end{array}$

Ta thấy $n+1$ chỉ có thể đi cùng với $2(n+1)$ (trường hợp $n+1$ đi cùng với $n+2$ không được xét trong $\left.u_{n+1}\right)$ nhưng $2(n+1)$ đã đi cùng với $2 n+1$ nên $n+1$ phải đi cùng với $n$ trong cùng một tập $A_i$ nào đó, giả sử là $A_n$. Lập luận tương tự trường hợp trên, ta suy ra số cách chọn các tập ${A_1, A_2, \ldots, A_n-1}$ là $u_{n-1}$.

Theo quy tắc cộng, ta có $u_{n+1}=u_n+u_{n-1}$. Mặt khác, $u_1=1$ và $u_2=2$ nên ta tìm được công thức tổng quát của $u_n$ là

$\quad\quad\quad\quad\quad\quad\quad\quad\quad\quad\quad\quad u_n=\frac{1}{\sqrt{5}}\left[\left(\frac{1+\sqrt{5}}{2}\right)^{n+1}-\left(\frac{1-\sqrt{5}}{2}\right)^{n+1}\right]$

Xét trường hợp sinh ra bộ $[A_1, A_2, \ldots, A_n]$ có $n$ và $n+1$ đi cùng trong một tập $A_i$ nào đó.

$\quad\quad\quad\quad\quad\quad\quad\quad\quad\quad\quad\quad\begin{array}{|c|c|l|c|}\hline 1 & 2 & \ldots & n \\ \hline n+1 & n+2 & \ldots & 2 n \\ \hline\end{array}$

Rõ ràng 1 chỉ có thể đi cùng với 2 hoặc $n+1$ nhưng $n+1$ đã đi cùng $n$ nên 1 chỉ có thể đi cùng với 2 . Tiếp theo, $n+2$ có thể đi cùng với $2, n+1$ hay $n+3$ nhưng 2 đã đi với 1 còn $n+1$ đã đi với $n$ nên $n+2$ phải đi với $n+3$.

Tiếp tục, 3 có thể đi cùng 2,4 hay $n+3$ nhưng 2 đã đi với 1 còn $n+3$ đã đi với $n+2$ nên 3 phải đi với 4 . Tiếp tục lý luận như trên, ta suy ra $A_i$ phải có dạng

$\quad\quad\quad [1,2],[3,4], \ldots,[n-2, n-1],[n, n+1],[n+1, n+2], \ldots,[2 n-1,2 n]$

Từ đó suy ra trường hợp này chỉ cho ta duy nhất một bộ $[A_1, A_2, \ldots A_n]$ nếu $n$ lẻ và không có bộ nào nếu $n$ chẵn.

Vậy số các bộ $[A_1, A_2, \ldots A_n]$ thỏa mãn đề bài là

$\quad\quad\quad\quad\quad\quad\quad\quad\begin{cases}\frac{1}{\sqrt{5}}\left[\left(\frac{1+\sqrt{5}}{2}\right)^{n+1}-\left(\frac{1-\sqrt{5}}{2}\right)^{n+1}\right] & , n=1,n \text { chẳn. } \\ 1+\frac{1}{\sqrt{5}}\left[\left(\frac{1+\sqrt{5}}{2}\right)^{n+1}-\left(\frac{1-\sqrt{5}}{2}\right)^{n+1}\right] & , n>1, n \text { lẻ’ }\end{cases}$

Bài 3. Tìm tất cả các hàm số $f: \mathbb{N}^* \rightarrow \mathbb{N}^*$ thoả mãn các điều kiện sau:

$\quad\quad(i) f $ là hàm số tăng thật sự trên $\mathbb{N}^*$.

$\quad\quad(ii) f(f(n))=4 n+9 \forall n \in \mathbb{N}^*$.

$\quad\quad(iii) f(f(n)-n)=2 n+9 \forall n \in \mathbb{N}^*$.

Lời giải. Vì $f: \mathbb{N}^* \longrightarrow \mathbb{N}^*$ tăng ngặt nên

$\quad\quad\quad\quad\quad\quad\quad\quad\quad\quad f(a)-f(b) \geq a-b, \forall a, b \in \mathbb{N}^*, a>b .$

Theo điều kiện (iii), ta có

$2=2(n+1)+9-(2 n+9) =f(f(n+1)-(n+1))-f(f(n)-n) $

$\quad\quad\quad\quad\quad\quad\quad\quad\quad\quad\quad\quad \geq f(n+1)-(n+1)-[f(n)-n] $

$\quad\quad\quad\quad\quad\quad\quad\quad\quad\quad\quad\quad =f(n+1)-f(n)-1 .$

Do đó $f(n+1)-f(n) \leq 3$

với mọi $n \in \mathbb{N}^*$, tức $f(n+1)-f(n) \in\ {1,2,3}$

với mọi $n \in \mathbb{N}^*$. Ta xét các trường hợp sau

  • Giả sử tồn tại $n \in \mathbb{N}^*$ sao cho $f(n+1)-f(n)=1$ thì

$\quad\quad\quad\quad\quad\quad\quad\quad\quad\quad f(n+1)-(n+1)=f(n)-n,$

suy ra $2(n+1)+9=f(f(n+1)-(n+1))=f(f(n)-n))=2 n+9$, vô lí.

  • Giả sử tồn tại $n \in \mathbb{N}^*$ sao cho $f(n+1)-f(n)=3$ thì

$\quad\quad\quad\quad\quad\quad\quad\quad\quad\quad f(n+1)-(n+1)=f(n)-n+2$

Ta lại có

$\quad\quad\quad\quad\quad\quad\quad\quad\quad\quad f(f(n+1)-(n+1))-f(f(n)-n)=2$

Do đó, nếu $f(n)>n$ thì đặt $t=f(n)-n \in \mathbb{N}^*$, ta suy ra $f(t+2)=t+2$.

Mà $f$ tăng ngặt nên $f(t+1)-f(t)=1$, mâu thuẫn.

Vậy $f(n) \leq n$ với mọi $n \in \mathbb{N}^*$ suy ra

$f(n)=n$ với mọi $n \in \mathbb{N}^*$.

Khi đó ta lại có $n=f(n)=f(f(n))=4 n+9,$ tức $n=-3$, vô lí.

Như vậy, $f(n+1)-f(n) \notin{1,3}$ với mọi $n \in \mathbb{N}^*$, nên

$\quad\quad\quad\quad\quad\quad\quad\quad\quad\quad f(n+1)-f(n)=2 \text {, với mọi } n \in \mathbb{N}^* \text {. }$

Ta suy ra $f(n)=2 n+k$. Thay vào đề bài, ta thu được $k=3$. Vậy $f(n)=2 n+3$ là nghiệm duy nhất của phương trình.

Bài 4. Cho đường tròn tâm $O$ và dây cung $A B$ cố định khác đường kính. Một điểm $P$ thay đổi trên cung lớn $A B$. Gọi $I$ là trung điểm của $A B$. Lấy các điểm $M, N$ trên các tia $P A, P B$ sao cho $\angle P M I=\angle P N I=\angle A P B$.

(a) Chứng minh rằng đường cao từ $P$ của tam giác $P M N$ luôn đi qua một điểm cố định.

(b) Chứng minh rằng đường thẳng Euler của tam giác $P M N$ luôn đi qua một điểm cố định.

Lời giải. (a) Kí hiệu $X=M I \cap P B, Y=N I \cap P A$. Ta có $\angle P M I=\angle P N I=$ $\angle A P B$ nên các tam giác $P M X$ và $P N Y$ cân tại $X, Y$. Từ đó suy ra

$\angle P X M=\angle P Y N=180^{\circ}-2 \angle A P B,$

suy ra $M, N, X, Y$ đồng viên. Gọi $S$ là tâm đường tròn ngoại tiếp tam giác $A O B$ thì $S$ cố định. Ta có $\angle I S B=180^{\circ}-\angle A O B=180^{\circ}-2 \angle A P B=\angle P X M$. Tương tự, ta suy ra $\angle I S A=\angle P Y N$. Do đó $I, S, X, B$ đồng viên và $I, S, Y, A$ đồng viên. Suy ra

$\angle S X B=\angle S Y A=\angle S I B=90^{\circ} .$

Suy ra $I S$ là đường kính của đường tròn ngoại tiếp tam giác $P X Y$. Mặt khác, $M, N, X, Y$ dồng viên nên nên $M N$ và $X Y$ đối song nhau trong $\angle A P B$, tức $I S \perp$ $M N$. Nói cách khác, đường cao từ $P$ của tam giác $P M N$ đi qua điểm $S$ cố định.

(b) Trước tiên, ta chứng minh bổ đề sau.

BỔ ĐỀ. Cho tam giác $A B C$ và đường tròn $(\omega)$ đi qua hai điểm $B, C$ và cắt các canh $A B, A C$ tại $X, Y$. Gọi $X X^{\prime}, Y Y^{\prime}$ là các đường cao của tam giác $A X Y$. Gọi $B B^{\prime}, C C^{\prime}$ là các đường cao của tam giác $A B C$. Gọi $H, H^{\prime}$ là các trục tâm của tam giác $A B C$ và tam giác $A X Y$. Kí hiệu $I \equiv B Y \cap C X$. Khi đó $H, I, H^{\prime}$ thẳng hàng.

Chứng minh. Ta có $X, Y, X^{\prime}, Y^{\prime}$ đồng viên nên $\overline{H^{\prime} X} \cdot \overline{H^{\prime} X^{\prime}}=\overline{H^{\prime} Y} \cdot \overline{H^{\prime} Y^{\prime}}$, tức là

$P_{H^{\prime} /[B Y]}=P_{H^{\prime} /[C X]},$

trong đó $[U V]$ là đường tròn đường kính $U V$. Ta có $B, C, B^{\prime}, C^{\prime}$ đồng viên nên $\overline{H B} \cdot \overline{H B^{\prime}}=\overline{H C} \cdot \overline{H C^{\prime}}$, tức

$P_{H /[B Y]}=P_{H /[C X]} .$

Cuối cùng $B, C, X, Y$ đồng viên nên $\overline{I B} \cdot \overline{I Y}=\overline{I C} \cdot \overline{I X}$, tức

$P_{I /[B Y]}=P_{I /[C Y]} .$

Suy ra $H, I, H^{\prime}$ thẳng hàng vì cùng thuộc trục đẳng phương của $[B Y]$ và $[C X]$.

Trở lại bài toán,

Gọi $H, O^{\prime}$ lần lượt là trực tâm và tâm đường tròn ngoại tiếp của tam giác $P M N$. Ta có $O^{\prime} P=O^{\prime} M$ và $X P=X M$ nên $X O^{\prime}$ là đường trung trực của $P M$, suy ra $X O^{\prime} \perp P Y$. Tương tự ta cũng có $Y O^{\prime} \perp P X$.

Vì thế nên $O^{\prime}$ cũng chính là trực tâm của tam giác $P X Y$. Áp dụng bổ đề cho tam giác $P X Y$ với $(\omega) \equiv(M N X Y)$ thì ta có $O^{\prime}, H, I \equiv Y N \cap M X$ thẳng hàng. Hay nói cách khác, đường thẳng Euler $O^{\prime} H$ của tam giác $P M N$ đi qua điểm $I$ cố định. Bài toán được giải quyết.

Ngày thi thứ hai

Bài 5. Cho $a, b, c$ là các số thực dương. Giải hệ phương trình sau:

$\quad\quad\quad\quad\quad\quad\quad\quad\quad\quad\left\{\begin{array}{l}a x-a b y+\frac{1}{x y}=b c^2 \\ a b z-b c^2 x+\frac{1}{x z}=a \\ b c^2 y-a z+\frac{1}{y z}=a b\end{array}\right.$

Lời giải. Đặt $(m, n, p)=\left(a, a b, b c^2\right)$. Khi đó $m, n, p>0$. Hệ phương trình trở thành

$\quad\quad\quad\quad\quad\quad\quad\quad\quad\quad\left\{\begin{array}{l}m x-n y+\frac{1}{x y}=p \\ n z-p x+\frac{1}{z x}=m \\ p y-m z+\frac{1}{y z}=n,\end{array}\right.$

tương đương

$\quad\quad\quad\quad\quad\quad\quad\quad\quad\quad\left\{\begin{aligned}m x-n y-p &=-\frac{1}{x y}, \\ -m+n z-p x &=-\frac{1}{z x}, \\ -m z+p y-n &=-\frac{1}{y z} .\end{aligned}\right.$

Xem hệ trên là hệ phương trình tuyến tính theo ẩn $m, n, p$, ta có

$\quad\quad D=\left|\begin{array}{ccc}x & -y & -1 \\ -1 & z & -x \\ -z & -1 & y\end{array}\right|=x y z-x y z-1-x^2-y^2-z^2=-1-\left(x^2+y^2+z^2\right) \neq 0$

$\quad\quad\quad\quad\quad\quad\quad \quad\quad D_m=\left|\begin{array}{ccc}x & -\frac{1}{x y} & -1 \\ -1 & -\frac{1}{z x} & -x \\ -z & -\frac{1}{y z} & y\end{array}\right|=-\frac{1+x^2+y^2+z^2}{z x} .$

Tương tự, ta cũng tính được

$\quad\quad\quad\quad\quad\quad D_n=-\frac{1+x^2+y^2+z^2}{y z} \text {, và } D_p=-\frac{1+x^2+y^2+z^2}{x y} .$

Do đó

$\quad\quad\quad\quad\quad\quad\quad (m, n, p)=\left(\frac{D_m}{D}, \frac{D_n}{D}, \frac{D_p}{D}\right)=\left(\frac{1}{z x}, \frac{1}{y z}, \frac{1}{x y}\right) .$

Thay $(m, n, p)=\left(a, a b, b c^2\right)$, ta được

$\quad\quad\quad\quad\quad\quad\quad\quad\quad\quad x y=\frac{1}{b c^2}, y z=\frac{1}{a b}, z x=\frac{1}{a} .$

Nhân ba phương trình trên vế theo vế rồi lấy căn bậc hai, ta được $x y z=\pm \frac{1}{a b c}$.

  • Với $x y z=\frac{1}{a b c}$, ta suy ra $x=\frac{1}{c}, y=\frac{1}{b c}, z=\frac{c}{a}$.
  • Với $x y z=-\frac{1}{a b c}$, ta suy ra $x=-\frac{1}{c}, y=-\frac{1}{b c}, z=-\frac{c}{a}$.

Vậy hệ có 2 nghiệm là $\left(\frac{1}{c}, \frac{1}{b c}, \frac{c}{a}\right)$ và $\left(-\frac{1}{c},-\frac{1}{b c},-\frac{c}{a}\right)$.

Bài 6. Cho dãy số $\left(a_n\right)$ xác định bởi $a_1=a, a_{n+1}=\left(a_1+\cdots+a_n-2\right)^2 \forall n \in \mathbb{N}^*$.

Đặt $S_n=a_1+a_2+\cdots+a_n$. Tìm tất cả các giá trị $a$ để dãy số $\left(S_n\right)$ hội tụ.

Lời giải. Từ giả thiết suy ra $S_{n+1}-S_n=\left(S_n-2\right)^2$. Do đó dãy $\left(S_n\right)$ được xác định như sau

$\quad\quad\quad\quad\quad\quad\quad\quad\quad\quad\left\{\begin{array}{l}S_1=a, \\ S_{n+1}=f\left(S_n\right)=S_n^2-3 S_n+4 .\end{array}\right.$

Hơn nữa $f^{\prime}(x)=0$ có nghiệm duy nhất $x=\frac{2}{3}$ nên ta có thể vẽ bảng biến thiên, khảo sát được hàm số này. Từ đó, nhờ việc các trường hợp của $a$, ta thấy

  • Nếu $a>2$. Giả sử $[S_n]$ có giới hạn $L$ thì ta phải có $L=f(L)$ nên $L \in{1,2}$. Mà $\left(S_n\right)$ không giảm nên $L \geq a>2$, mâu thuẫn. Vậy nếu $a>2$ thì $\left(S_n\right)$ không hội tụ.
  • Nếu $a<1$ thì suy ra $S_2=f(a)>2$. Quay về trường hợp 1 , ta suy ra $\left(S_n\right)$ không hội tụ.
  • Nếu $1 \leq a \leq 2$ thì từ bảng biến thiên, ta có $\frac{7}{4} \leq S_n \leq 2$ với mọi $n \in \mathbb{N}^*$. Từ đó $\left(S_n\right)$ không giảm và bị chặn nên $\left(S_n\right)$ hội tụ.

Vậy các giá trị của $a$ thỏa mãn đề bài là $a \in[1,2]$.

Bài 7. Tìm tất cả các số nguyên dương $k$ để phương trình sau có nghiệm nguyên dương $(x, y)$ :

$\quad\quad\quad\quad\quad\quad\quad\quad\quad\quad\quad x^2+y^2+x+y=k x y .$

Lời giải. Không mất tính tổng quát, giả sử $x \geq y$. Xét giá trị $k$ sao cho phương trình đã cho có nghiệm nguyên dương. Trong các nghiệm ấy, gọi $\left(x_0, y_0\right)$ là nghiệm sao cho $x_0 \geq y_0 \geq 0$ và $x_0$ nhỏ nhất. Xét tam thức

$\quad\quad\quad\quad\quad\quad\quad\quad\quad f(x)=x^2-\left(k y_0-1\right) x+y_0^2+y_0 .$

Khi đó $f\left(x_0\right)=0$. Theo định lí Viette, $f(x)$ còn một nghiệm khác là $x_0^{\prime}=k y_0-1-x_0$. Tuy nhiên, theo cách chọn $\left(x_0, y_0\right)$ thì ta có $x_0^{\prime} \geq x_0 \geq y_0$ nên $y_0$ nằm ngoài hai khoảng nghiệm của tam thức bậc hai $f(x)$. Mà hệ số cao nhất của $f(x)$ dương nên $f\left(y_0\right) \geq 0$. Do $f\left(y_0\right) \geq 2 y_0^2+2 y_0-k y_0^2$ nên ta có

$\quad\quad\quad\quad\quad\quad\quad\quad\quad\quad\quad k \leq 2+\frac{2}{y_0} \leq 4 .$

Suy ra $k \in{1,2,3,4}$.

  • Nếu $k=1$ thì phương trình có dạng $x^2+y^2+x+y=x y$, tương đương với

$\quad\quad\quad\quad\quad\quad\quad\quad\left(x-\frac{y}{2}\right)^2+\frac{3}{4} y^2+x+y=0$ (vô lí vì $\left.x, y>0\right)$.

  • Nếu $k=2$ thì phương trình có dạng $x^2+y^2+x+y=2 x y$, tương đương với

$\quad\quad\quad\quad\quad\quad\quad\quad\quad\quad(x-y)^2+x+y=0$ (vô lí vì $x, y>0$ ).

  • Nếu $k=3$ thì phương trình có nghiệm $(x, y)=(2,2)$.
  • Nếu $k=4$ thì phương trình có nghiệm $(x, y)=(1,1)$.

Vậy các giá trị cần tìm là $k=3, k=4$.

Bài 8. Cho tam giác $A B C$ nội tiếp đường tròn $(O)$. Gọi $I, I_1, I_2, I_3$ lần lượt là tâm đường tròn nội tiếp và tâm đường tròn bàng tiếp các đỉnh $A, B, C$ của tam giác $A B C$. Đường tròn ngoại tiếp tam giác $I_2 I_3$ cắt $(O)$ tại hai điểm $M_1, N_1$. Gọi $J_1$ là giao điểm của $A I$ và $(O)$. Ký hiệu $d_1$ là đường thẳng qua $J_1$ và vuông góc với $M_1 N_1$. Xác định các đường thẳng $d_2, d_3$ tương tự. Chứng minh rằng $d_1, d_2, d_3$ đồng quy.

Lời giải. Gọi $\left(O^{\prime}\right)$ là đường tròn ngoại tiếp tam giác $I_1 I_2 I_3$ và $\left(O_1\right)$ là đường tròn ngoại tiếp tam giác $\left(I I_2 I_3\right)$. Ta có $A I \perp I_2 I_3, B I \perp I_3 I_1$ nên $I$ là trực tâm tam giác $I_1 I_2 I_3$ và $(O)$ là đường tròn Euler của tam giác $I_1 I_2 I_3$ nên $O$ là trung điểm của $I O^{\prime}$. Mặt khác thì

$\angle I_2 O_1 I_3=2\left(180^{\circ}-\angle I_2 I I_3\right)=2 \angle I_2 I_1 I_3=I_2 O^{\prime} I_3 .$

Do đó $O^{\prime}$ đối xứng với $O_1$ qua $I_2 I_3$, suy ra $\overrightarrow{O^{\prime} O_1}=\overrightarrow{I_1 I}$, tức $A I O_1 O^{\prime}$ là hình bình hành. Mà $O$ là trung điểm $I O^{\prime}$ nên $O$ cũng là trung điểm của $I_1 O_1$. Hơn nữa, $O O_1 \perp M_1 N_1$ (đường nối tâm vuông góc với dây cung) nên $I_1 O \perp M_1 N_1$.

Mặt khác, $J_1$ là trung điểm của $I I_1$ (đường tròn Euler đi qua trung điểm của đoạn thẳng nối trực tâm của tam giác với đỉnh của tam giác ấy) nên phép vị tự tâm $I$, tỉ số $k=\frac{1}{2}$ biến $I_1 O_1$ thành $d_1$. Do đó $d_1$ đi qua trung điểm $S$ của $O I$. Tương tự, ta suy ra $d_2, d_3$ cũng đi qua $S$, tức $d_1, d_2, d_3$ đồng quy.

Nhận xét. Bài toán thực chất là việc đổi mô hình từ một tính chất quen thuộc liên quan đến trực tâm, chân đường cao sang mô hình ba tâm bàng tiếp. Vì thế, đôi khi việc chuyển đổi giữa các mô hình giúp cho bài toán sáng sủa, dễ xử lý hơn.

 

 

 

 

 

 

 

 

 

 

 

 

 

 

 

 

 

 

 

 

 

 

 

 

 

 

 

 

 

 

 

 

 

 

 

 

 

 

 

 

 

 

 

 

 

 

 

 

 

 

 

 

 

 

 

 

 

 

 

 

 

 

 

 

 

 

 

 

 

 

 

 

 

 

 

 

 

 

 

 

 

 

Đề thi và đáp án kì thi chọn đội tuyển thi Quốc gia trường Phổ thông Năng khiếu năm học 2008 – 2009

ĐỀ THI

Ngày thi thứ nhất

Bài 1.

(a) Chứng minh rằng tồn tại số $n$ chẵn, $n>2008$ sao cho $2009 n-49$ là số chính phương.

(b) Chứng minh rằng không tồn tại số nguyên $m$ sao cho $2009 m-147$ là số chính phương.

Bài 2.

(a) Tìm số các số tự nhiên có 3 chữ số khác nhau và chia hết cho 6 .

(b) Tìm số các số tự nhiên có $n$ chữ số lập từ các chữ số ${1,2,3,4,5}$ và chia hết cho 3 .

Bài 3. Cho tam giác $A B C$ có $A$ cố định và $B, C$ thay đổi trên đường thẳng $d$ cố định sao cho nếu gọi $A^{\prime}$ là hình chiếu của $A$ lên $d$ thì $\overline{A^{\prime} B} \cdot \overline{A^{\prime} C}<0$ và không đổi. Gọi $M$ là hình chiếu của $A^{\prime}$ lên $A C$.

(a) Chứng minh rằng tâm $I$ đường tròn ngoại tiếp tam giác $B C M$ thuộc một đường thẳng cố định.

(b) Gọi $N$ là hình chiếu của $A^{\prime}$ lên $A B$ và $K$ là giao điểm hai tiếp tuyến của $\left(A^{\prime} M N\right)$ tại $M, N$. Chứng minh rằng $K$ thuộc một đường thẳng cố định.

Bài 4. Cho $f(x)=x^2+a x+b$ là tam thức bậc hai với $a, b \in \mathbb{R}$ và $f(f(x))=0$ có 4 nghiệm thực phân biệt. Biết rằng tổng của 2 nghiệm nào đó trong số 4 nghiệm đã nêu bằng $-1$. Chứng minh rằng $b \leq-\frac{1}{4}$.

Ngày thi thứ hai

Bài 5. Giả sử đa thức $P(x)=(x+1)^p(x-3)^q=x^n+a_1 x^{n-1}+a_2 x^{n-2}+\cdots+a_n$, trong đó $p, q$ là các số nguyên dương. Chứng minh rằng nếu như $a_1=a_2$ thì $3 n$ là một số chính phương.

Bài 6.

(a) Cho $a, b, c$ là các số thực dương. Chứng minh rằng ta có bất đẳng thức

$\quad\quad\quad\quad\quad\quad\quad\quad\quad\quad\quad\frac{a^2+b^2+c^2}{a b+b c+c a}+\frac{8 a b c}{(a+b)(b+c)(c+a)} \geq 2 .$

(b) Chứng minh rằng tồn tại các số thực dương $a, b, c$ sao cho

$\quad\quad\quad\quad\quad\quad\quad\quad\quad\quad\quad\frac{a b+b c+c a}{a^2+b^2+c^2}+\frac{(a+b)(b+c)(c+a)}{8 a b c}<2 .$

Bài 7. Cho góc $O x y$ và một điểm $P$ nằm bên trong nó. Gọi $\gamma$ là đường tròn thay đổi nhưng luôn qua $O$ và $P$. Giả sử $\gamma$ cắt $O x, O y$ tại $M, N$. Tìm quỹ tích trọng tâm $G$ và trực tâm $H$ của tam giác $O M N$.

Bài 8. Với mỗi số nguyên dương gọi là tổng các chữ số của $n$.

(a) Chứng minh rằng $n=999, n=2999$ không thể biểu diễn thành tổng $a+b$ mà $S(a)=S(b)$.

(b) Chứng minh rằng với mọi $n$ mà $999<n<2999$ thì điều kiện trên được thỏa mãn.

LỜI GIẢI

Ngày thi thứ nhất

Bài 1.

(a) Chứng minh rằng tồn tại số $n$ chẵn, $n>2008$ sao cho $2009 n-49$ là số chính phương.

(b) Chứng minh rằng không tồn tại số nguyên $m$ sao cho $2009 m-147$ là số chính phương.

Lời giải. Chú ý rằng $2009=49 \cdot 41=7^2 \cdot 41$ nên yêu cầu bài toán tương đương với việc chứng minh

  1. Tồn tại số $n$ chẵn, $n>2008$ sao cho $41 n-1$ là số chính phương.
  2. Không tồn tại số nguyên $m$ sao cho $41 m-3$ là số chính phương.

(a) Trước hết, ta đi tìm một số $a$ sao cho $a^2+1$ chia hết cho 41 . Điều này có thể được thực hiện bằng cách thử tuần tự. Ta dễ dàng tìm được $a=9$ thỏa mãn. Từ đây, ta thấy các số $(82 k+9)^2+1$ là số chẵn và chia hết cho 41 . Bây giờ chỉ cần chọn.

$\quad\quad\quad\quad\quad\quad\quad\quad\quad\quad\quad n=\frac{(82 k+9)^2+1}{41}$

với $k$ đủ lớn là ta tìm được số $n$ thỏa mãn điều kiện đề bài.

(b) Giả sử tồn tại $m$ sao cho $41 m-3=a^2$. Khi đó ta có $-3 \equiv a^2(\bmod 41)$. Từ đó theo định lý Fermat nhỏ thì

$\quad\quad\quad\quad\quad\quad\quad\quad\quad\quad\quad (-3)^{20} \equiv a^{40} \equiv 1 \quad(\bmod 41) .$

Nhưng mặt khác, ta lại có $(-3)^4 \equiv-1(\bmod 41)$, suy ra

$\quad\quad\quad\quad\quad\quad\quad\quad\quad\quad\quad (-3)^{20} \equiv(-1)^5 \equiv-1 \quad(\bmod 41) .$

Do đó $1 \equiv-1(\bmod 41)$, vô lý. Do đó điều đã giả sử là sai, tức là không tồn tại số nguyên $m$ sao cho $41 m-3$ là số chính phương.

Bài 2.

(a) Tìm số các số tự nhiên có 3 chữ số khác nhau và chia hết cho 6 .

(b) Tìm số các số tự nhiên có $n$ chữ số lập từ các chữ số ${1,2,3,4,5}$ và chia hết cho 3 .

Lời giải. (a) Ta sẽ tìm các số có dạng $\overline{a b c}$ sao cho $c$ chẵn và $3 \mid a+b+c$. Ta có các trường hợp sau

  • Nếu $c=0$ thì $a+b \in{3,6,9,12,15,18}$ nên có $2+4+8+6+4+0=24$.
  • Nếu $c=2$ thì $a+b \in{1,4,7,10,13,16}$ nên có $1+3+5+6+6+2=23$.
  • Nếu $c=4$ thì $a+b \in{2,5,8,11,14,17}$ nên có $1+3+7+6+4+2=23$.
  • Nếu $c=6$ thì $a+b \in{3,6,9,12,15,18}$ nên có $3+4+7+6+2+0=22$.
  • Nếu $c=8$ thì $a+b \in{1,4,7,10,13,16}$ nên có $1+3+7+6+4+2=23$.

Do đó, có tất cả $24+23+23+22+23=115$ số thỏa mãn đề bài.

(b) Ta xét khai triển tương ứng

$\quad\quad\quad\quad\quad\quad\quad\quad\quad\quad\quad P(x)=\left(x+x^2+x^3+x^4+x^5\right)^n=\sum_{k=n}^{5 n} a_k x^k$

Số các số chia hết cho ba cần tìm chính bằng tổng các số hạng của khai triển trên có dạng $x^{3 k}$, giả sử tổng đó là $A$. Xét ba nghiệm phức của phương trình $t^3=1$ là $t=1, t=\varepsilon, t=\varepsilon^2$. Ta có:

$\quad\quad\quad\quad\quad\quad\quad\quad\quad\quad\quad P(1)=5^n, P(\varepsilon)=\left(\varepsilon+\varepsilon^2+\varepsilon^3+\varepsilon+\varepsilon^2\right)^n =(-1)^n $

$\quad\quad\quad\quad\quad\quad\quad\quad\quad\quad\quad P\left(\varepsilon^2\right)=\left(\varepsilon^2+\varepsilon+1+\varepsilon^2+\varepsilon\right)^n =(-1)^n$

Hơn nữa, dễ dàng thấy rằng

$\quad\quad\quad\quad\quad\quad\quad\quad\quad\quad\quad P(1)+P(\varepsilon)+P\left(\varepsilon^2\right)=\sum_{k=n}^{5 n} a_k\left(1+\varepsilon^k+\varepsilon^{2 k}\right) $

Nếu $3 \mid k$ thì $P(1)+P(\varepsilon)+P\left(\varepsilon^2\right)=3 A$. Nếu $k$ không chia hết cho 3 thì $1+\varepsilon^k+\varepsilon^{2 k}=$ $1+\varepsilon+e^2=0$ nên các biểu thức còn lại trong tổng $P(1)+P(\varepsilon)+P\left(\varepsilon^2\right)$ đều bằng 0. Vậy nên ta tính được

$\quad\quad\quad\quad\quad\quad\quad\quad\quad\quad\quad P(1)+P(\varepsilon)+P\left(\varepsilon^2\right)=3 A \Leftrightarrow A=\frac{5^n+2(-1)^n}{3}$

Đó cũng chính là số các số cần tìm.

Bài 3. Cho tam giác $A B C$ có $A$ cố định và $B, C$ thay đổi trên đường thẳng $d$ cố định sao cho nếu gọi $A^{\prime}$ là hình chiếu của $A$ lên $d$ thì $\overline{A^{\prime} B} \cdot \overline{A^{\prime} C}<0$ và không đổi. Gọi $M$ là hình chiếu của $A^{\prime}$ lên $A C$.

(a) Chứng minh rằng tâm $I$ đường tròn ngoại tiếp tam giác $B C M$ thuộc một đường thẳng cố định.

(b) Gọi $N$ là hình chiếu của $A^{\prime}$ lên $A B$ và $K$ là giao điểm hai tiếp tuyến của $\left(A^{\prime} M N\right)$ tại $M, N$. Chứng minh rằng $K$ thuộc một đường thẳng cố định.

Lời giải. (a) Đặt $\overline{A^{\prime} B} \cdot \overline{A^{\prime} C}=-k^2$. Từ $E$ hạ $I E \perp A A^{\prime}$. Gọi $N^{\prime}, P$ lần lượt là giao điểm của $(B M N)$ với $A A^{\prime}$. Ta có:

$\quad\quad\quad\quad\quad\quad\quad\quad\overline{A M} \cdot \overline{A B}=\overline{A N^{\prime}} \cdot \overline{A P}=A A^{\prime 2}=\left(\overline{A A^{\prime}}+\overline{A^{\prime} N^{\prime}}\right)\left(\overline{A A^{\prime}}+\overline{A^{\prime} P}\right)$

Do đó,

$\quad\quad\quad\quad\quad\quad\quad\quad\quad\quad\quad \overline{A A^{\prime}}\left(\overline{A^{\prime} N^{\prime}}+\overline{A^{\prime} P}\right)=-\overline{A^{\prime} N^{\prime}} \cdot \overline{A^{\prime} P}=k^2$

Ta thu dược

$\quad\quad\quad\quad\quad\quad\quad\quad\quad\quad\quad\quad\quad\quad\quad 2 \overline{A^{\prime} E}=\frac{k^2}{\overline{A A^{\prime}}}$

Suy ra $E$ cố định. Vậy điểm $I$ chạy trên đường thẳng qua $E$ và vuông góc với $A A^{\prime}$ cố định.

(b) Gọi $F$ là trung điểm của $A A^{\prime}$, ta có $F$ là tâm đường tròn ngoại tiếp tứ giác $A N A^{\prime} M$. Gọi $Z$ là giao điểm của $M N$ và $A A^{\prime}$, ta có

$\quad\quad\quad\quad\quad\quad\quad\quad\quad\quad\quad \overline{Z A} \cdot \overline{Z A^{\prime}}=\overline{Z M} \cdot \overline{Z N^{\prime}}=-k^2 \text {. }$

Suy ra $Z$ cố định.

Bây giờ, từ $K$ hạ $K Y$ vuông góc với $A A^{\prime}$. Ta có $F, M, N, K, Y$ cùng nằm trên một đường tròn, suy ra

$\quad\quad\quad\quad\quad\quad\quad\quad\quad\quad\quad \overline{Z F} \cdot \overline{Z Y}=\overline{Z M} \cdot \overline{Z N^{\prime}}=-k^2 .$

Từ đây dễ thấy $Y$ cố định. Vậy $K$ di động trên đường thẳng qua $Y$ vuông góc với $A A^{\prime}$ cố định.

Bài 4. Cho $f(x)=x^2+a x+b$ là tam thức bậc hai với $a, b \in \mathbb{R}$ và $f(f(x))=0$ có 4 nghiệm thực phân biệt. Biết rằng tổng của 2 nghiệm nào đó trong số 4 nghiệm đã nêu bằng $-1$. Chứng minh rằng $b \leq-\frac{1}{4}$.

Lời giải. Trước hết, dễ thấy $f(x)=0$ phải có 2 nghiệm phân biệt, đặt là $c_1<c_2$. Gọi $x_1, x_2$ là 2 trong số 4 nghiệm có tổng bằng $-1$. Theo định lý Viete thì: $c_1+c_2=$ $-a, c_1 c_2=b$. Ngoài ra,

$\quad\quad\quad\quad\quad\quad\quad\quad\quad\quad\quad f(f(x))=0 \Leftrightarrow f(x)=c_1 \vee f(x)=c_2 .$

Ta xét các trường hợp sau

  • Nếu $x_1, x_2$ là nghiệm của cùng một phương trình trong hai phương trình trên, theo định lý Viete thì $-a=x_1+x_2=-1$ nên $a=1$. Do $f(x)=c_1, f(x)=c_2$ đều phải có 2 nghiệm phân biệt nên $\Delta_1>0, \Delta_2>0$, tức là $1-4\left(b-c_1\right)>$ $0,1-4\left(b-c_2\right)>0$. Cộng lại, ta có

$\quad\quad\quad\quad\quad\quad\quad\quad\quad\quad\quad 2-4\left(2 b-c_1-c_2\right)>0 \Leftrightarrow 1-2(2 b+1)>0 \Leftrightarrow b<-\frac{1}{4} .$

  • Nếu $x_1, x_2$ là nghiệm của hai phương trình thì $x_1^2+a x_1+b=c_1, x_2^2+a x_2+b=c_2$. Cộng lại, ta có $x_1^2+x_2^2+a\left(x_1+x_2\right)+2 b=c_1+c_2 \Leftrightarrow x_1^2+x_2^2+2 b=0$. Do đó $b=-\frac{x_1^2+x_2^2}{2} \leq-\frac{\left(x_1+x_2\right)^2}{4}=-\frac{1}{4}$.

Trong mọi trường hợp, ta luôn có điều phải chứng minh.

Ngày thi thứ hai

Bài 5. Giả sử $P(x)=(x+1)^p(x-3)^q=x^n+a_1 x^{n-1}+a_2 x^{n-2}+\cdots+a_n$, trong đó $p, q$ là các số nguyên dương. Chứng minh rằng nếu $a_1=a_2$ thì $3 n$ là một số chính phương.

Lời giải. Ta có

$P(x)=(x+1)^p(x-3)^q=\left(x^p+C_p^1 x^{p-1}+C_p^2 x^{p-2}+\cdots\right)\left(x^q-3 C_q^1 x^{q-1}+9 C_q^2 x^{q-2}+\cdots\right)$

Từ đó suy ra

$\quad\quad\quad\quad\quad\quad\quad\quad\quad\quad\quad a_1=C_p^1-3 C_q^1 \text { và } a_2=C_p^2+9 C_q^2-3 C_p^1 C_q^1$

Như vậy $a_1=a_2$ khi và chỉ khi

$\quad\quad\quad\quad\quad\quad\quad\quad\quad\quad\quad p-3 q=\frac{p(p-1)}{2}+\frac{9 q(q-1)}{2}-3 p q$

hay

$\quad\quad\quad\quad\quad\quad 2 p-6 q=p^2-p+9 q^2-9 q-6 p q \text { tức là } 3 n=3(p+q)=(p-3 q)^2 \text {. }$

Suy ra $3 n$ là số chính phương. Ta có điều phải chứng minh.

Bài 6.

(a) Cho $a, b, c$ là các số thực dương. Chứng minh rằng ta có bất đẳng thức

$\quad\quad\quad\quad\quad\quad\quad\quad\quad\quad\quad \frac{a^2+b^2+c^2}{a b+b c+c a}+\frac{8 a b c}{(a+b)(b+c)(c+a)} \geq 2 .$

(b) Chứng minh rằng tồn tại các số thực dương $a, b, c$ sao cho

$\quad\quad\quad\quad\quad\quad\quad\quad\quad\quad\quad \frac{a b+b c+c a}{a^2+b^2+c^2}+\frac{(a+b)(b+c)(c+a)}{8 a b c}<2 .$

Lời giải. (a) Không mất tính tổng quát, ta có thể giả sử $c=\min {a, b, c}$. Khi đó, với chú ý rằng $a^2+b^2+c^2 \geq a b+b c+c a$, ta có

$\quad\quad\quad\quad\quad\quad\quad\quad\quad\quad \frac{a^2+b^2+c^2}{a b+b c+c a} =1+\frac{a^2+b^2+c^2-a b-b c-c a}{a b+b c+c a} $

$\quad\quad\quad\quad\quad\quad\quad\quad\quad\quad\quad\quad\quad\quad \geq 1+\frac{a^2+b^2+c^2-a b-b c-c a}{a b+b c+c a+c^2} $

$\quad\quad\quad\quad\quad\quad\quad\quad\quad\quad\quad\quad\quad\quad =\frac{a^2+b^2+2 c^2}{a b+b c+c a+c^2} $

$\quad\quad\quad\quad\quad\quad\quad\quad\quad\quad\quad\quad\quad\quad =\frac{a^2+b^2+2 c^2}{(a+c)(b+c)}$

Do đó, ta chỉ cần chứng minh.

$\quad\quad\quad \frac{a^2+b^2+2 c^2}{(a+c)(b+c)}+\frac{8 a b c}{(a+b)(b+c)(c+a)} \geq 2$

Bất đẳng thức này tương đương với

$a^3+b^3+a^2 b+b^2 a+2 c^2 a+2 c^2 b+8 a b c \geq 2\left(a^2 b+a^2 c+b^2 a+b^2 c+c^2 a+c^2 b+2 a b c\right)$

hay

$a^3+b^3+4 a b c \geq a^2 b+b^2 a+2 a^2 c+2 b^2 c . \Leftrightarrow(a-b)^2(a+b-2 c) \geq 0 .$

Do $c=min (a, b, c)$ nên bất đẳng thức cuối hiển nhiên đúng.

Phép chứng minh hoàn tất. Đẳng thức xảy ra khi và chỉ khi $a=b=c$.

(b) Kiểm tra trực tiếp, ta thấy bộ $(a, b, c)=(2,1,1)$ thỏa mãn đề bài. Điều này cho thấy rằng nếu nghịch đảo cả hai phân số trong vế trái của câu a) thì bài toán không còn đúng nữa.

Bài 7. Cho góc $O x y$ và một điểm $P$ nằm bên trong nó. Gọi $\gamma$ là đường tròn thay đổi nhưng luôn qua $O$ và $P$. Giả sử $\gamma$ cắt $O x, O y$ tại $M, N$. Tìm quỹ tích trọng tâm $G$ và trực tâm $H$ của tam giác $O M N$.

Lời giải. (a) Quĩ tích trọng tâm G của tam giác $O M N$

Gọi $I$ là trung điểm $M N$. Ta có ${G}=V_O^{\frac{2}{3}}({I})$. Ta sẽ tìm quỹ tích điểm $I$.

Phần thuận. Gọi $X$ là giao điểm thứ hai của $(I M P)$ với $O x, Y$ là giao điểm thứ hai của $(I N P)$ với $O y$. Ta có:

$\quad\quad\quad\quad\quad\quad\quad\quad\quad\quad\quad \angle X I P=\angle X M P=\angle P N Y=180^{\circ}-\angle P I Y$

do đó $X, Y, Z$ thẳng hàng. Mặt khác,

$\quad\quad\quad\quad\quad\quad\quad\quad\quad\quad\quad \angle I X P=\angle N M P=\angle P O N$

nên $\angle I X P$ không đổi. Tương tự ta cũng có $\angle X Y P$ không đổi mà $P$ cố định suy ra $X, Y$ cố định vậy $I$ nằm trên đường thẳng $X Y$ cố định.

Phần đảo. Lấy $X, Y$ lần lượt thuộc tia $O x, O y$ sao cho $\angle P X Y=\angle P O y$ và $\angle P Y X=$ $\angle P O x$. Lấy $I \in X Y$ ta sẽ chứng minh tồn tại $M \in O x, N \in O y$ sao cho $(O M N)$ đi qua $P$. Thật vậy,

Gọi $M$ là giao điểm thứ hai của $(I X P)$ và $O x, N$ là giao điểm thứ hai của $(I Y P)$ và $O y$. Ta có:

$\quad\quad\quad\quad\quad\quad\quad\quad\quad\quad\quad \angle X M P=\angle X I P=\angle P N Y$

nên tứ giác $O M P N$ nội tiếp. Ta có điều phải chứng minh.

(b) Quỹ tích trục tâm H của tam giác OMN.

Phần thuận. Gọi $T$ là trung điểm $O P . X, Y$ là hình chiếu của $P$ lên $O x, O y . K$ là trực tâm tam giác $O X Y . I$ là tâm đường tròn $(O M N)$. Ta có

$\quad\quad\quad\quad\quad\quad\quad\quad\quad\frac{O K}{O T}=\frac{O H}{O I}(=2|\cos \angle x O y|) \text { và } \angle I O T=\angle H O K$

nên $\triangle I O T \sim \triangle H O K$. Mà $\angle I T O=90^{\circ}$ nên $\angle H K O=90^{\circ}$. Vậy $H$ thuộc đường thẳng qua $K$ vuông góc với $O K$ (Chú ý $K$ cố định).

Phần đảo. Lấy $H$ thuộc đường thẳng qua $K$ vuông góc với $O K$, trong đó $K$ là trực tâm tam giác $O X Y$ và $X, Y$ là hình chiếu của $P$ lên $O x, O y$. Ta sẽ chứng minh tồn tại $M \in O x, N \in O y$ sao cho $(O M N)$ đi qua $P$ và $\triangle O M N$ nhận $H$ là trực tâm. Thật vậy, gọi $T$ là trung điểm $O P$ và dựng $\triangle O T I \sim \triangle O K H$. Ta có

$\quad\quad\quad\quad\quad\quad\quad\quad\quad\quad\quad \angle O T I=\angle O K H=90^{\circ}$

$\quad\quad\quad\quad\quad\quad\quad\quad\quad\quad\quad \angle I O T=\angle H O K$

nên $I$ nằm trên trung trực của $O P$. Do đó nếu vẽ $(T, T O)$ thì đường tròn này đi qua $P$ và cắt $O x, O y$ tại $M, N$. Ta có $K, T$ là trực tâm và tâm đường tròn $(O X Y)$ nên $\angle K O y=\angle T O x$. Ta được

$\quad\quad\quad\quad\quad\quad\quad\quad\quad\quad\quad \angle I O y=\angle H O x$

Lại có $\triangle O T I \sim \triangle O K H$ nên $\frac{O H}{O I}=\frac{O T}{O K}=2|\cos \angle x O y|$. Ta suy ra $H$ là trực tâm tam giác $O M N$

Bài 8. Với mỗi số nguyên dương gọi là tổng các chữ số của $n$.

(a) Chứng minh rằng $n=999, n=2999$ không thể biểu diễn thành tổng $a+b$ mà $S(a)=S(b)$.

(b) Chứng minh rằng với mọi $n$ mà $999<n<2999$ thì điều kiện trên được thỏa mãn.

Lời giải. Ta sẽ giải bài toán tồng quát sau: Tồn tại các số $a, b$ thỏa mãn điều kiện khi và chỉ khi số $n>1$ không có dạng $n=\overline{m 999 \ldots 9}$ với $0 \leq m \leq 8$ và $S(n)$ lẻ.

Thật vậy, xét $n=\overline{d_1 d_2 d_3 \ldots d_k}$ với $k$ là số các chữ số của $n$.

Ta có 2 trường hợp cần xét như sau:

  1. Nếu $S(n)$ chẵn (tương đương với có chẵn chữ số lẻ trong trong $n$ ), ta thực hiện như sau:
  • Nếu $d_i$ chẵn thì tách thành $d_i=\frac{d_i}{2}+\frac{d_i}{2}$ và chữ số ở hàng tương ứng của $a, b$ sẽ là 2 số này.
  • Nếu $d_i$ lẻ thì tách thành $d_i=\frac{d_i-1}{2}+\frac{d_i+1}{2}$ thì $\frac{d_i+1}{2}-\frac{d_i-1}{2}=1$ và như thế, ta luân phiên thay đổi các số lớn nhỏ để ghép vào $a, b$ để đảm bảo có $S(a)=S(b)$.

Do có chẵn số $d_i$ lẻ như thế nên quá trình trên thực hiện được và trong trường hợp này, tồn tại số $a, b$ thỏa mãn.

  1. Nếu $S(n)$ lẻ (tương đương với có lẻ chữ số lẻ trong trong $n$ ). Nếu $n$ có dạng $m 999 \ldots 9$ với $0 \leq m \leq 8$ thì rõ ràng khi tách ra thành 2 phần, các phép tính tồng phía sau để thu được các sồ 9 là không có nhớ và chúng có dạng $d_i+d_i^{\prime}=9$. Khi đó, ta có

$\quad\quad\quad\quad\quad\quad\quad S(a)+S(b)=S(n) \text { mà }\left\{\begin{array}{l}S(a)=S(b) \\ S(n) \equiv 1(\bmod 2)\end{array}\right.$

Điều mâu thuân trên cho thấy trường hợp này không tồn tại cách tách $n$ thành $a, b$ thỏa mãn. Nếu $n$ không có dạng trên thì dễ thấy tồn tại $d_i \neq 0, d_{i+1} \neq 9$.

Ta viết lại $n$ như sau:

$n=\overline{d_1 d_2 d_3 \ldots d_i d_{i+1} \ldots d_k}=\overline{d_1 d_2 d_{3 \ldots}\left(d_i-1\right) 9 \ldots d_k}+\left(d_{i+1}+1\right) 10^{k-d_i}$

Đặt $a=\overline{d_1 d_2 d_3 \ldots\left(d_i-1\right) 9 \ldots d_k}, b=\left(d_{i+1}+1\right) 10^{k-d_i}$ thì $S(a), S(b)$ cùng tính chẵn lẻ và $a+b=n$. Ta có thể giả sử $S(a)>S(b)$, trường hợp còn lại chứng minh tương tự. Nếu chọn một vị trí $t$ mà $d_t \neq 0$ thì có thể đổi các số $a, b$ thành

$a^{\prime}=\overline{d_1 d_2 d_3 \ldots\left(d_i-1\right) 9 \ldots\left(d_t-1\right) \ldots d_k}, y=\left(d_{i+1}+1\right) 10^{k-i}+10^{k-t}$

với $k \neq i$. Khi đó

$\quad\quad\quad \quad\quad\quad S\left(a^{\prime}\right)-S\left(b^{\prime}\right)=S(a)-1-(S(b)+1)=-2 .$

Cứ như vậy, ta thực hiện liên tiếp đến khi nào chênh lệch giữa hai tổng các chữ số bằng 0 thì dừng lại (tồn tại thời điểm như vậy vì ban đầu chúng cùng tính chẵn lẻ nên hiệu của chúng là số chẵn và mỗi lần thực hiện quá trình trên thì hiệu giảm đi 2 đơn vị). Các số $a, b$ lúc đó sẽ thỏa mãn đề bài và cũng tồn tại cách tách.

Bài toán được giải quyết hoàn toàn.